Dies ist ein Archiv der Seite Wikipedia:Auskunft. Es enthält alle Abschnitte, die in der Kalender-Woche 03 im Jahr 2016 begonnen wurden.

Möchtest du in einer neuen Diskussion auf dieses Archiv verweisen? Nutze folgenden Link und ersetze ________ durch den Namen des Abschnittes. Die Nummer im Inhaltsverzeichnis gehört nicht dazu:

[[Wikipedia:Auskunft/Archiv/2016/Woche 03#________]]


← vorherige Woche Gesamtarchiv nächste Woche →

gmail-Virus ?

Ein merkwürdiges Phänomen:

Ich erhalte von einer mir bekannten Person XYZ eine Mail von der Adresse    XYZ@gmail.com.
Ich antworte auf die Mail (mit Reply) und in der Adressatenzeile steht nun    XYZ@mail.com (das "g" ist weg). Ich bekomme keine Nachricht, dass diese Mail nicht zugestellt werden kann.
In einer älteren Mail derselben Person (von vor etwa 4 Monaten) passiert das jetzt nicht (wenn ich diese ältere Mail nun beantworten will).
Bei Antwort an andere mir bekannten Personen (Beispiel: DEF@gmail.com) passiert das nicht.
Was passiert da?
Wo ist das Virus und wo kann man darüber nachlesen? Play It Again, SPAM (Diskussion) 19:54, 19. Jan. 2016 (CET)
der Absender hat ein REPLY-TO in seinen Mail-Header aufgenommen und dort seine @mail.com-Adresse eingetragen?!? --gdo 19:59, 19. Jan. 2016 (CET)
Yep. Schau dir mal den Quelltext oder die Kopfzeilen der Mail an (weiß nicht, wie das in deinem Mailprogramm geht) – dort dürfte ein Eintrag „Reply-To“ mit der mail.com-Adresse sein. Das hat der Absender in seinem Mailprofil so eingegeben und sich dabei wahrscheinlich vertippt. Das braucht man aber sowieso nur anzugeben, wenn die Antwort an eine andere Adresse gehen soll. Wenn kein Reply-To eingetragen ist, wird die Antwort an die Absenderadresse geschickt. – Fazit: Nicht gleich ein Virus vermuten, wenn es auch ein Bedienfehler sein kann. --Kreuzschnabel 21:28, 19. Jan. 2016 (CET)
Danke. Die Antworten waren hilfreich. Ich werde mit dem einen Sender Kontakt aufnehmen und ihn dazu bringen, dort nachzusehen und zu korrigieren. Play It Again, SPAM (Diskussion) 22:31, 19. Jan. 2016 (CET)
Archivierung dieses Abschnittes wurde gewünscht von: Play It Again, SPAM (Diskussion) 22:31, 19. Jan. 2016 (CET)

Website gehackt?

Wenn ich die Website im Artikel EHC Grindelwald aufrufe, sehe ich lauter japanische Schriftzeichen. Kann es sein, dass die Seite gehackt wurde? --Leyo 23:07, 19. Jan. 2016 (CET)

Denic.ch, demnach ist der Besitzer der Domain tatsächlich ein Japaner, evtl hat er kommerzielle Absichten, da demnach schon seit 2002 registriert--in dubio Zweifel? 23:26, 19. Jan. 2016 (CET)
Seltsam, dann hat der Verein wohl seine Website eingestellt. Über eine allfällige Auflösung habe ich jedenfalls nichts gefunden. --Leyo 23:35, 19. Jan. 2016 (CET)
laut der vereinseigenen Facebook-Präsenz scheint dies die offizielle Domain zu sein (imho wohl eher im Aufbau)--in dubio Zweifel? 23:45, 19. Jan. 2016 (CET)
Vielen Dank. Bei Google scheint die neue Website jedenfalls ein ziemlich schlechtes Ranking zu haben. Naja, vielleicht verbessert sich das ja jetzt durch die Korrektur im Artikel. ;-) --Leyo 00:04, 20. Jan. 2016 (CET)
Archivierung dieses Abschnittes wurde gewünscht von: Leyo 00:04, 20. Jan. 2016 (CET)

Vergleich der großen IT-Firmen?

Gibt es eigentlich irgendwo einen Vergleich zwischen den großen IT-Firmen? Welche Chancen und Bediengungen sie den Mitarbeitern bieten? Welche Projekte sie anstreben, auf welchen Märkten sie aktiv sind? Welche Ideen immer wieder vertreten werden? (nicht signierter Beitrag von 188.101.70.113 (Diskussion) 00:57, 18. Jan. 2016 (CET))

Erst mal solltest du definieren was du unter großen IT-Firmen verstehst. Dann in welchem Bereich, Deutschland? USA? Europa? Asien oder Weltweit? Außerdem ist die Bandbreite was alles IT ist doch enorm. Wie willst du z.B. Cray mit SAP vergleichen? --Mauerquadrant (Diskussion) 03:41, 18. Jan. 2016 (CET)
So funktioniert das nicht :) Du arbeitest dort in einem Projekt oder einer Abteilung. Und da können innerhalb der Firma komplett unterschiedliche Philosophien und Vorgehensweisen herrschen. Eine Abteilung setzt auf open source die andere macht nur mit Standartprodukten rum, in einer Abteilung verdienst du klasse und machst interessante Sachen in der anderen bist du nur schlecht bezahlter Datenschubser. Solche Vergleiche wie du dir erhoffst kann es nicht geben. Höchstens sehr eingeschränkt wie etwa ein Programmierer verdient im Durchschnitt bei Firma A sounsoviel und bei Firma B ... . --84.149.232.44 14:14, 18. Jan. 2016 (CET)

Warum Split und nicht Sand als Winterstreumittel

(Alle Jahre wieder trifft Berlin im Winter die globale Erwärmung mit Dauerfrost, Eis und Schnee). Seit Jahren kommt kein Sand mehr zum Einsatz, sondern es werden scharfkantige Scherben gestreut. Die im Artikel genannten Eigenschaften treffen auf Splitt nicht zu: "nicht so rasch in die Eisdecke gedrückt", "werden nicht so schnell von Neuschnee versteckt" (gilt nur, wenn der Neuschnee nicht mehr als 3 mm beträgt....). Das sollte im Artikel abgeändert werden! Zusätzlich kommt jedoch, dass sich Winterdienste regelmäßig die Bereinigung sparen und einfach ("den guten") Splitt rüberschütten und das Splitt aufgrund seiner Scherben-/Splittereigenschaft ein hohes Verletzungs- und Beschädigungsrisiko (insbesondere Fahrradreifen!, Bekleidung, Innenbodenbeläge) aufweist. Zusätzlich werden Splitkörner von schmelzenden und wieder überfrierenden Wasser/Kondensat umfasst und erhöhen(!) die Glättegefahr. Warum also Splitt? Billiger als Sand? An den gegenüber Sand besseren Eigenschaften liegt es nicht.--Wikiseidank (Diskussion) 10:44, 18. Jan. 2016 (CET)

Was im Artikel steht, ist richtig und selbstverständlich: „dass sie aufgrund der Größe […] nicht so schnell [wie die kleinen Sandkörner] von Neuschnee verdeckt werden“. „Aufgrund der Korngröße werden sie nicht so rasch in die Eisdecke gedrückt wie Sand.“ (Hervorhebung durch mich.) Außerdem steht dort: „Sand ist weniger witterungsbeständig als Granulate und kommt daher seltener zum Einsatz.“ --BlackEyedLion (Diskussion) 11:22, 18. Jan. 2016 (CET)
(BK) :WP: "Sand ist weniger witterungsbeständig als Granulate und kommt daher seltener zum Einsatz." "Nach dem Winter werden diese Streugranulate im Zuge der Straßenreinigung eingesammelt und für den Straßenbau verwendet." Play It Again, SPAM (Diskussion) 11:24, 18. Jan. 2016 (CET)
Splitt ist scharfkantig und größer als 2 mm, Sand ist abgerundet und kleiner als 2 mm. Kleine Körper versinken schneller, abgerundete Körper rollen über Eis. Von Splitt braucht man weniger Masse, allerdings ist er auch erheblich teurer. Die scharfen Kanten schneiden sich in Eis und bremsen deshalb besser als Sand. Fahrradreifen verkraften Splitt auch fast ohne Beschädigung, allerdings nur Winterreifen. --Pölkkyposkisolisti 11:38, 18. Jan. 2016 (CET)
es gibt auch Spikes-Reifen für Fahrraäder, hilft vielleicht auch!? MfG Arieswings (Diskussion) 11:43, 18. Jan. 2016 (CET)
Das sind ja die Wintereifen. Sie haben sehr kleine Lamellen, die gegen den Splitt nachgeben. Foto folgt. --Pölkkyposkisolisti 11:53, 18. Jan. 2016 (CET)
Splitt wird mitnichten fuer den Strassenbau verwendet, sondern als Sondermuell deponiert: [1], [2], [3]... --Nurmalschnell (Diskussion) 12:44, 18. Jan. 2016 (CET)
Splitt ist unabdingbar für den Straßenbau. Ob man dafür jedoch die Reste vom Winter nimmt, wage ich zu bezweifeln, die Reinigung wäre zu teuer. Was passiert, wenn man statt Splitt einfach Kies nimmt, hat man in der DDR gesehen, überall zerdrückte Straßen mit Spurrillen usw. Das mit "Sondermüll" ist Panikmache. --Pölkkyposkisolisti 12:49, 18. Jan. 2016 (CET)
Laut "Die Welt" manchmal Weiterverwendung, manchmal Müllkippe (aber von Sondermüll steht auch da nichts). [4] --Eike (Diskussion) 12:56, 18. Jan. 2016 (CET)
Fahrrad Winterreifen heißt Spikes, was keine Alternative in der Stadt ist, bei wechselnden Untergründen bspw. Kopfsteinpflaster/Zufahrten und Eisen im Boden: Schiene, Gullideckel. Nur ein Modell von Conti ohne Spikes, dessen Lamellen den meinigen nicht unähnlich sind. "Fahrradreifen mit sehr kleinen Lamellen, die gegen Splitt nachgeben"? Wie ist das gemeint, Slicks hätten ja gar keine Lamellen?. 5 mm Pannenschutz + Pannenschutzeinlage helfen bei mir nicht ausreichend gegen Splitt. Winterreifen sind gegen Winter (Schnee, Eis) nicht gegen Splitt! Was denn nun: "Sand nicht witterungsbeständig" (löst sich auf) oder "Sand/Kies macht die Straßen kaputt". Bei Winterstreu geht es aber nicht um Straßen. Da wird in Berlin kein Splitt raufgemacht. Nur auf Fußwegen, Radwegen (warum auch immer?), Kombis und Schulhöfen(!).--Wikiseidank (Diskussion) 13:08, 18. Jan. 2016 (CET)
Geh- und Radwege gehören (wie die Fahrbahn) zur Straße. Dass der gebrauchte Splitt Sondermüll ist und für den Straßenbau verwendet wird, schließt sich nicht aus; Sondermüll kann im Straßenbau verwendet werden. --BlackEyedLion (Diskussion) 13:39, 18. Jan. 2016 (CET)

Eine Schreibweise kann nur richtig sein. Google schlägt Bonell vor. Vielleicht der Name des Erfinders/einer Firma? Gruss --Nightflyer (Diskussion) 20:50, 18. Jan. 2016 (CET)

(BK) Amazon / en:WP / weitere Referenzen im Web verwenden durchgehend die Schreibweise "Bonnell". Ein Stapel von US Patenten welche im Prinzip identische Federn verwenden gehen knappe 100 Jahre zurück und ein Erfinder namens Thomas Bonnell taucht (inter alia) 1910 auf. Einen Thomas Bonnell gibts im US Census 1920, aber das mag natürlich ein anderer sein und der US Census ist kostenpflichtig. Sonst konnte ich über die Person nichts herausfinden. --Cookatoo.ergo.ZooM (Diskussion) 21:14, 18. Jan. 2016 (CET)
Im US Census 1920 gibt es sogar fast ein Dutzend Thomas Bonnells; leider konnte ich die von dir erwähnten Patente nicht finden. PDD 23:32, 18. Jan. 2016 (CET)
+1 zu Bonnell: ... sagt das Web. Auch GoogleBooks ist auf der Bonnell-Seite. Da man aber auch rel. häufig Bonell findet (auch GB), sollte dies in Klammern dazugesetzt werden. Play It Again, SPAM (Diskussion) 09:20, 19. Jan. 2016 (CET)
Danke. Wo soll die Klammer hin? Oder einen Hinweis auf Falschschreibung? Wer weiss, wie, bitte erledigen :-) GRuss --Nightflyer (Diskussion) 23:23, 19. Jan. 2016 (CET)
Done. --Neitram 
Archivierung dieses Abschnittes wurde gewünscht von: Nightflyer (Diskussion) 20:54, 20. Jan. 2016 (CET)

WhatsApp lebenslang oder nicht?

Vorhin, um halb neun bekam ich die Nachricht von WhatsApp "aus Kulanz habe[n] wir lebenslangen Service für deinen Account gewährt". Zwanzig Minuten später eine neue Nachricht: "neues Ablaufdatum: 04.07.2016". Was soll das? Wollen sie es nun kostenlos machen oder nicht? --  etrophil44 21:40, 19. Jan. 2016 (CET)

Es wurde verschiedentlich über die Presse bekanntgegeben, dass Whatsapp fürderhin kostenlos ist, weil der Anbieter seine Einnahmen auf anderem Wege, nämlich über Firmenkunden, generieren will. --Rôtkæppchen₆₈ 21:55, 19. Jan. 2016 (CET)
war wohl ein Versehen, siehe hier--in dubio Zweifel? 11:13, 20. Jan. 2016 (CET)
An Deiner Stelle würde ich versuchen bis zum 04.07.2016 alles Wichtige zu regeln und mit allen verstrittenen Verwandten deinen Frieden zu machen ;) VG Thogru (Diskussion) 11:50, 20. Jan. 2016 (CET)
Jo, in dubio, jetzt ist bei mir auch wieder lebenslang  Vorlage:Smiley/Wartung/;)  --  etrophil44 11:56, 20. Jan. 2016 (CET)
Archivierung dieses Abschnittes wurde gewünscht von: --  etrophil44 11:56, 20. Jan. 2016 (CET)

Kontolöschungen

habe aus [V]ersehen 3 Konten aktiviert und nur bei 2 kann ich mich einloggen. Wie kann ich Konto löschen. --AG-GÜNTHER (Diskussion) 09:09, 20. Jan. 2016 (CET)

Hinweis: Für Fragen speziell zur Wikipedia gibt es ein eigenes Diskussionsforum: Fragen zur Wikipedia.
Gar nicht. Wenn ich mir deine Beiträge (hier und die kaputten Anfragen Wikipedia:Benutzernamen ändern) so anschaue, erwarte ich ehrlich gesagt auch nichts Gutes. --Magnus (Diskussion) für Neulinge 09:16, 20. Jan. 2016 (CET)
Dein alter Benutzer wurde mittlerweile in Benutzer:Neuer Benutzername AlAlAl umbenannt. Beachte, dass die Wörter „Neuer Benutzername“ antragsgemäß Bestandteil des Benutzernamens sind. --Rôtkæppchen₆₈ 09:45, 20. Jan. 2016 (CET)
Gute Güte, was für ein Chaos. Bitte alle Benutzer sperren... --Magnus (Diskussion) für Neulinge
Archivierung dieses Abschnittes wurde gewünscht von: Kann weg, siehe Benutzer:Pentachlorphenol/Panda und Co. --Rôtkæppchen₆₈ 00:23, 21. Jan. 2016 (CET)

Betrugsmasche?

Wollte mir über Amazon ein Smartphone bestellen und sehe, dass ein anderer Verkäufer dies statt für ca. 560 Euro für 400 anbietet. Allerdings stehen widersprüchliche Aussagen im Shop. "Gebraucht - Wie neu" und "EUR 400,00 + Versand (derzeit nicht verfügbar)", "Versand aus Deutschland - Internationale und Inlandsversandkosten". Diese Angaben scheinen aus dem Shopsystem von Amaon zu kommen (Kenne mich damit aber nicht wirklich aus). Per Hand ergänzt steht da "NEU - Bitte schreiben sie an: xxxxxxx [bèi] GMX DE wenn sie kaufen möchten ." Ich hab ihn angeschrieben und bekomme folgende Antwort "Hallo Das Sony Xperia Z5 Smartphone (5,2 Zoll (13,2 cm) Touch-Display, 32 GB interner Speicher, Android 5.1) schwarz ist neu, Deutschland Model. Das Produkt ist im Neuzustand, mit internationale Garantie, Eingang, alle Hersteller mitgelieferte Zubehör kommt. Der Preis beträgt Eur 400.00 inkl. MwSt die Versandkosten. Wenn Sie kaufen möchten senden Sie mir Ihren vollständigen Namen und Lieferadresse und ich kontaktierst Amazon so schnell wie möglichum Ihre Bestellung zu verarbeiten. Versand ist durch normale UPS oder DHL Services führt 2-4 Tage je nachdem, wo in der Welt was Sie sind. Mein Rückgaberecht ist voller Geld zurück in 30 Tagen. Für weitere Informationen zögern Sie nicht mich zu kontaktieren. Vielen Dank." Verstehe nicht ganz, wieso es so günstig angeboten wird. Der Kauf scheint lt. Mail dann doch über Amazon stattfzufinden, so daß man als Käufer auch abgesichert sein sollte. Oder gibt es irgendeine Masche, wie er an mein Geld kommt ohne zu liefern?" --141.15.29.108 07:17, 18. Jan. 2016 (CET)

der Kauf läuft dann nicht über Amazon, sondern rein privat. Im besten Falle will der Verkäufer einfach die unverschämt hohen Verkäufer-Gebühren von Amazon vermeiden, im ungünstigsten Falle ist es schlicht Betrug. --gdo 07:53, 18. Jan. 2016 (CET)
Allgemein gesagt ist damit Verbunden: Keine Garantie, keine Bewertung und kein Käuferschutz des Portals, da Privatkauf. Habe ich noch etwas vergessen? Möglicherweise werden auf diese Weise B-Ware, Rückläufer und Geräte mit latenten oder nicht sofort ersichtlichen Fehlern/Schäden veräußert. Bei Computerzubehör kann der Schaden auch in der Software liegen. Die Sicherheitslücke: Der Straßenapfel (Roadapple) – ungenießbares ausgelegt als Köder. Wie sorgfältig ist das Gerät von Daten des Vorbesitzers oder schadhafter Software befreit? Nicht einmal Händler löschen teilweise die Zugangsdaten aus DSL-Router raus, die sie zurückgenommen haben. Es wurde kein Reset durchgeführt, die Firmware auf Manipulation nicht geprüft bzw. der Sicherheit halber neu aufgespielt. --Hans Haase (有问题吗) 09:27, 18. Jan. 2016 (CET)
Danke euch. Das Angebot ist inzwischen von Amazon entfernt worden. Und ich werde auch drauf verzichten, per Mail einen Kauf über die Summe zu regeln. --141.15.29.108 09:43, 18. Jan. 2016 (CET)
Die Frage ob das ein Betrüger ist oder nicht ist heutzutage irrelevant. Du zahlst einfach über Paypal und wenn der dich betrügt hast du mit einem Klick dein Geld zurück und er ist gegenüber Paypal in der Nachweispflicht das er wie angeboten geliefert hat. Da besteht keine Möglichkeit mehr über Tricks irgendwie an dein Geld zu kommen. --84.149.232.44 14:09, 18. Jan. 2016 (CET)
Paypal ist der letzte Dreck. Bin einmal bei ebay einem Betrüger auf den Leim gegangen (Händler mit Sitz in Vietnam, ebay-Konto war auf einmal gelöscht), aber Paypal hat den Fall sofort mit der Begründung "nicht zuständig" geschlossen (es ging um einen Gutschein), so dass sich an Paypal keine Nachrichten mehr zu dem Fall schicken ließen. Bin zum Glück dann doch noch an mein Geld gekommen, indem ich die Lastschrift von Paypal bei meiner Bank rückgängig machen ließ. Daraufhin erkannte Paypal den Betrug kommentarlos an und machte das Geschäft rückgängig, brummte mir aber eine "Verwaltungsgebühr" von 5 € auf, naja besser als ein Totalverlust. --195.36.120.126 15:12, 18. Jan. 2016 (CET)
Du hättest Paypal und ebay den Verkäufer melden müssen. Wenn Du Paypal zurückbuchen lässt, ist gleich dem das Paypalkonto zu überziehen. Zwar hast Du nun Dein Geld, aber möglicherweise Ärger oder Sperre bei Paypal. Die Unterschrift zum Zustellen hast Du nicht gleistet, daher wäre der Versender in Pflicht zu belegen. Damit kannst Du aber bei Paypal und ebay reklamieren. --Hans Haase (有问题吗) 16:37, 18. Jan. 2016 (CET)
Ich hab den Händler doch gemeldet. Ebay hat auf Paypal verwiesen, weil ich über Paypal bezahlt habe und Paypal hat dicht gemacht, weil Gutscheine nicht unter diesen Witz namens Käufer"schutz" fallen. Die Lastschriftrückbuchung war der einzige Weg, wieder an mein Geld zu kommen. Mit dem Paypalkonto ist dann auch alles in Ordnung gewesen (bis auf die Strafgebühr), ist schon ein paar Jahre her. Nur gut dass ich nicht per Überweisung gezahlt hatte. --195.36.120.126 18:03, 18. Jan. 2016 (CET)
Vermutlich hast du das zu spät gemeldet oder ähnliches. Ich kenne das aus der Sicht des Händlers. Wenn ein Päckchen mal einen Tag überfällig ist klicken Kunden gern sicherheitshalber auf den Button und das Geld ist sofort eingefroren. Der Händler ist dann in der Pflicht allumfassend nachzuweisen das er ordnungsgemäß geliefert hat. Tut er das nicht geht das Geld zurück an den Kunden. Und dafür werden Ihm keine Gebühren berechnet. Die Bearbeitungsgebühr hast du vermutlich gezahlt weil du den Bankeinzug rückgängig gemacht hast anstatt das über Paypal zu regeln. --84.149.232.44 19:46, 18. Jan. 2016 (CET)

Internationaler Zuchtverband für Kaninchen

Gibt es für Hauskaninchen auch einen Weltdachverband für die Zucht und Rassestandards wie die World Cat Federation für Katzen oder die Fédération Cynologique Internationale für Hunde? --Fiver, der Hellseher (Diskussion) 17:06, 18. Jan. 2016 (CET)
Ich vermute der internationale Verband wird sich mit mehr als nur Kanichen befassen. In der Schweiz sind die Kanichenzucht-Verien ja schon zusammen mit anderen, artfremden Vereinen im Verein Kleintieren Schweiz organisiert. --Bobo11 (Diskussion) 17:22, 18. Jan. 2016 (CET)
(BK)Entente Européenne d’Aviculture et de Cuniculture. --Rôtkæppchen₆₈ 17:22, 18. Jan. 2016 (CET)
Stelle ich die Frage mal noch genauer Einen Internationalen Dachverband, welcher sich mit der Zucht von Kaninchen, Meerschweinchen, Frettchen und anderer (Nagetiere) befasst gibt es nicht oder? Ich habe bloß über Kaninchen-, Meerschweinchen- Rattenrassen in Europa und den USA. Ich werde mal ne Mail an den Zentralverband Deutscher Rasse-Kaninchenzüchter schreiben. Mal sehen was die sagen. --Fiver, der Hellseher (Diskussion) 17:35, 18. Jan. 2016 (CET)
Es braut nicht mehr gesucht werden. Ich habe jetzt die Seite der WORLD RABBIT SCIENCE ASSOCIATION (WRSA) des Internationalen Dachverbandes für Kaninchenzucht und alles damit zusammen hängt gefunden. WEbseite: http://world-rabbit-science.com/ --Fiver, der Hellseher (Diskussion) 17:40, 18. Jan. 2016 (CET)*
Hmmmm - das ist aber kein Dachverband, oder?
Grösster Verein: ARBA (aber eben nur in den USA). Play It Again, SPAM (Diskussion) 17:43, 18. Jan. 2016 (CET)
Folgender Satz hat mich in positiver Weise vom Stuhl gehauen: "No meeting was held in 2012 because only one member of the AB-WRSA attended the Congress.". Tja... Play It Again, SPAM (Diskussion) 17:49, 18. Jan. 2016 (CET)

Rechnungsfrist Rechtsfrage

Nach VOB ist eine Rechnung innerhalb von 21 Tagen zu prüfen und sollte als Eingang beim Auftragnehmer verzeichnet sein. Mir hat heute jemand gesagt, ist die Rechnung in diesem Zeitraum nicht als Eingang verzeichnet, gilt die Rechnung in vollem Umfang als geprüft, muß also vollständig ohne Abzug überwiesen werden. Verhält es sich so?--Markoz (Diskussion) 18:42, 18. Jan. 2016 (CET)

Wenn man den Eingang nachweisen kann ... --Heletz (Diskussion) 19:12, 18. Jan. 2016 (CET)
.Wie meinst Du das, also die Rechnung ist mit Einschreiben und Mail beim AG eingegangen. Die Rechnung wurde erst nach 21 Tagen (krikekrakelig) geprüft uns aber nicht zugestellt. Die geprüfte Rechnung wurde uns erst nach 46 Tagen zugestellt und grade mal 10 % überwiesen, obwohl alle Arbeiten ausführlich + nachvollziehbar dargestellt worden sind.--Markoz (Diskussion) 19:22, 18. Jan. 2016 (CET)
Die Schlussrechnung ist im Normalfalle innerhalb von 30 Tagen zu prüfen, spätestens nach 60 Tagen. Fällig wird sie nach Prüfung und Feststellung. Ob der Auftraggeber mit Rügen der Rechnung nach Ablauf der Frist endgültig ausgeschlossen ist, ist nicht unumstritten; die meisten Gerichte gehen von einem Einwendungsausschluss oder mindestens einer Beweislastumkehr aus.
Nach 21 Tagen fällig wird eine Abschlagszahlung.
Zugang beweist man besser nicht mit Einschreiben (der Inhalt des Einschreibens ist ja nicht klar, nicht einmal beim so hoch geschätzten Einschreiben mit Rückschein) und auch nicht per Mail (da ist der Nachweis besonders schwierig), sondern per Boten, per persönlicher Übergabe mit Zeugen oder notfalls zur Fristwahrung vorab per Telefax. --Snevern 19:51, 18. Jan. 2016 (CET)
Danke war nur eine Abschlagzahlung, bin da Sub habe die Rechnung also nicht gestellt, sondern der hauptauftragnehmer mit dem ich befreundet bin.--Markoz (Diskussion) 20:19, 18. Jan. 2016 (CET)
Ich habe Fliesenarbeiten bei einem alteingesessenen Fliesenleger beauftragt. Das wurde von einem Gesellen ausgeführt und ich habe die Rechnung dann ziemlich zeitnah (14 Tage) bezahlt. Etwa 3 Monate später rief bei mir der Fliesenlegergeselle an und fragte, ob das stimmt, dass ich die Rechnung noch nicht bezahlt habe. Es kam folgendes heraus, der Fliesenleger hat seine 4 Gesellen schon länger entlassen und sie gebeten sich selbständig zu machen, er fungierte ab dato nur noch als Arbeitsvermittler und Rechnungschreiber für die Subs, bezahlte sie dann anteilig nach einem ausgemachten Schlüssel mit Provision für sich. So auch bei meinem Auftrag. Für den Kunden blieb das verborgen. Die Subs hatten nun den Verdacht, weil sie nur schleppend Geld sahen, dass die Ausflüchte, dass die Kunden so schlecht bezahlen nur vorgeschoben sind, daher der Anruf bei mir. Der Laden ging dann auch pleite, irgend wie hat sich auch der Staatsanwalt darum gekümmert. Also pass auf welches Spielchen dein Freund mit dir spielt, Geld versaut mitunter den Charakter.--2003:75:AF11:D400:CD9F:45A2:F3D2:F50B 22:22, 18. Jan. 2016 (CET)
.ja ich kenne die Falle. Aber der hat 25 Jahre lang immer pünktlich gezahlt. Das ist der Bauherr, bei der letzten Rechnung hat er auch x Positionen (fertig gestellt als nicht prüfbar gestrichen) und sich dann bei der überfälligen Überweisung um genau 10000,00 vertan und das macht der nicht nur bei uns so sondern bei allen Handwerkern...man sieht sich halt auf dem Bau....und unterhält sich.--Markoz (Diskussion) 22:35, 18. Jan. 2016 (CET)

Wiki-Phänomen

Seit heute erlebe ich mit sämtlichen Windows-Versionen das seltsame Phänomen, dass ich bei einem Seitenwechsel hin und wieder für zwei bis vier Sekunden auf der Seite "Du bist nicht angemeldet" lande, allerdings ist das Warnbapperl beige, statt magenta. Weiß jemand, was los ist? LG;--Dr.Lantis (Diskussion) 18:51, 18. Jan. 2016 (CET) PS: Falls es ne Technik-Support-Seite gibt, wo ich das ebenfalls melden kann, würde ich mich über eine sachliche Auskunft nebst Direktlink freuen.

WP:FZW, WP:TWS. Falls zwei bis vier Sekunden heißt, dass es dann ohne Wiederanmeldung weitergeht, ist mir das so unbekannt. Ständiges Rausfliegen kenn ich dagegen, beschränkt sich aber nicht auf die WP und in meinem Fall liegt's eher am Browser. -ZT (Diskussion) 20:21, 18. Jan. 2016 (CET)
Gehst Du über einen Mobilfunkanbieter online, der Network Address Translation (NAT) verwendet? Ja, kann problematisch werden, wenn nein, melde ab, lösche Cookies und Cache (STRG+SHIFT+DEL), starte den Browser neu, erlaube der Seite Wikipedia alle Zugriffe in Werbeblockern. --Hans Haase (有问题吗) 20:28, 18. Jan. 2016 (CET)

Änderung nicht rückgängig machen, sondern löschen

Hallo, ich habe öfters das Problem, dass ich Änderungen vornehme und dabei vergessen habe, mich anzumelden. In den meisten Fällen werde ich durch ein Captcha noch rechtzeitig vor dem Absenden gewarnt (um mich anmelden zu können), dummer Weise manchmal aber auch nicht! Frage: Ist es möglich, diejenige Änderung, die nun mit meiner IP versehen ist, angemeldet komplett zu löschen statt rückgängig zu machen? Besten Dank!-----hoep (Diskussion) 19:36, 18. Jan. 2016 (CET)

Nein, das konterkariert u.a. die lizenzwichtige Nachvollziehbarkeit von Änderungen. Eine sinnvolle Änderung rückgängig zu machen, ist auch suboptimal und könnte bei Wiederholungen auch zu einer Vandalismussanktion führen, denke ich. Grüße, Grand-Duc (Diskussion) 19:41, 18. Jan. 2016 (CET)
Melde Dich doch dauerhaft an...--Markoz (Diskussion) 19:48, 18. Jan. 2016 (CET)
Der Bearbeiter, Inhalt und Zusammenfassungszeile sind Versionslöschbar. Als Grund gibt es: „Entfernen von nichtöffentlichen, privaten Informationen.“. Wende Dich per Mail an den Admin Deines Vertrauens. Der Löschgrund ist in Deinem Fall etwas dubios. --Hans Haase (有问题吗) 20:13, 18. Jan. 2016 (CET)
Ja, dass man (s)eine IP verwendet hat, zählt nicht als nichtöffentlichen Informationen im Sinne dieser Regel. --Eike (Diskussion) 20:14, 18. Jan. 2016 (CET)
Nun, das "Dauerhaft"-Häkchen ist obligatorisch, nur fällt es unter den Tisch, wenn die Anmeldung - eben - vergessen wurde und man nicht den ganzen Tag ausschließlich mit der Wikipedia verbringt. Ich denke es ist verstanden worden: der Punkt ist eine Löschung einer Änderung mit einer IP, die nachweisbar versehentlich abgeschickt wurde bzw. man diese explizit um Löschung nachfragt. Das kann also kein Administrator für einen erledigen, der - klar - zwischen (möglichem) Vandalismus und eindeutigem Versehen unterscheiden kann, selbst wenn man darum höflich nachfragt? Warum sollte das "dubios" sein? So grausam geht das hier zu? ;-)
Andererseits möchte man das ja am Besten vermeiden, das unangemeldete Abschicken, nur, warum wird man manchmal durch ein Captcha gewarnt und manchmal nicht? Kein Ersatz möglich a la "Du möchtest speichern - willst Du Dich nicht vorher anmelden"? (einen solcher Hinweis kann man zwar beim Beginn einer Änderung ganz weit oben lesen, vergisst ihn aber nach einer gewissen Zeit doch, zumal er nicht mehr im Blickfeld neben dem Absendeknopf liegt...)-----hoep (Diskussion) 20:45, 18. Jan. 2016 (CET)
Ich versteh noch nicht, warum dir das so oft passiert. Ich bin ja auch den lieben langen Tag woanders unterwegs, aber die Wikipeedia-Anmeldung geht mir gefühlt nur einmal im Monat verloren. Hast du vielleicht ein Plugin, das Cookies löscht (oder den Browser so eingestellt)? --Eike (Diskussion) 20:48, 18. Jan. 2016 (CET)
Ja, richtig, die Kiste läuft nicht den ganzen Tag und der Browser löscht sämtliche Küchlein beim Beenden/Herunterfahren. Aber ... Admin Meines Vertrauens klingt nach Lösung - habe ich einen solchen? Und wenn ja, wo/wie kann ich ihn erreichen?-----hoep (Diskussion) 20:58, 18. Jan. 2016 (CET)
Ob du eine persönliche Vertrauensperson hast kann ich nicht beantworten. Die allgemeine Anlaufstelle wäre jedoch Wikipedia:Administratoren/Anfragen. ;) --StYxXx 04:14, 22. Jan. 2016 (CET)

China-Staatskarosse

Was ist das für ein Auto, in das Präsident Xi Jinping hier einsteigt? Etwas Chinesisches? Oder ein BMW? Leider nur Facebook-URL. --Aalfons (Diskussion) 03:16, 18. Jan. 2016 (CET)

BMW 7er --Heletz (Diskussion) 10:08, 18. Jan. 2016 (CET)
Es könnte aber auch ein BMW-Nachbau von BYD Auto sein. --Rôtkæppchen₆₈ 10:16, 18. Jan. 2016 (CET)
Das ist ein BMW der als Joint Venture in China gebaut wird, ist angeblich auch 17 cm länger als die hiesigen.--2003:75:AF11:D400:8F8:2C01:50A:8293 17:41, 18. Jan. 2016 (CET)
Wenn das stimmt, könnte es auch die Form der Scheinwerfer erklären, die etwas anders zu sein scheint als beim BMW_G11. --Aalfons (Diskussion) 18:39, 18. Jan. 2016 (CET)
In China werden viele europäische Modelle in einer speziellen Langversion gebaut. In der Luxusklasse lässt man fahren und der Sitz hinten rechts ist DER Sitz. Um dort mehr Platz zu schaffen, werden die Autos verlängert. Quelle: Werkzeugbauer für China. --FirestormMD ♫♪♫♪ 08:46, 19. Jan. 2016 (CET)

Wort gesucht

Wie nennt man folgenden Sachverhalt: Eine Gruppen von Menschen schließt eines ihrer Mitglieder aus und verpflichtet alle anderen jeglichen Kontakt mit dem/der Ausgeschlossenen einzustellen Gesucht ist ein soziologische Fachbegriff auf deutsch. Zusatzfrage: Entspricht das dem englischen Begriff "Shunnig". Gesucht ist eine Information von außerhalb der Wikipdia (deutsch und englisch). mit Gruß und Dank Catrin (Diskussion) 14:03, 18. Jan. 2016 (CET)

Bann (Bibel)? Verbannung? Exkommunikation? --Hans Haase (有问题吗) 14:17, 18. Jan. 2016 (CET)
Persona non grata? --Joschi71 (Diskussion) 14:36, 18. Jan. 2016 (CET)
Gebannt, Vogelfrei, Aussätzig--Markoz (Diskussion) 14:37, 18. Jan. 2016 (CET)
Im nicht-religiösen Kontext hört sich das nach "Ausgrenzung" an. --Eike (Diskussion) 14:38, 18. Jan. 2016 (CET)
Meidung ist gerade mit der Begründung der Begriffsetablierung in der Löschdiskussion. Der Artikel existiert seit 2003. Phonetisch lässt sich vom „shunning“ auf „schonen“ schießen, wobei dies positiv belegt ist, jemand „zu (ver)schonen“. Imo wäre es übereinstimmender von «[Aufruf/Weisung] zur Meidung einer Person» zu sprechen. --Hans Haase (有问题吗) 14:49, 18. Jan. 2016 (CET)
Im Artikel Tabu ist auch die Rede, dass es sich auf Personen bezeichnen kann.--Wikiseidank (Diskussion) 14:57, 18. Jan. 2016 (CET)
In der Literatur findet sich vereinzelt auch der Begriff „Kontaktsperre“ (nicht im strafrechtlichen Sinne gemeint), so etwa in Anja Amend-Traut, Albrecht Cordes, Wolfgang Sellert: Geld, Handel, Wirtschaft: Höchste Gerichte im Alten Reich als Spruchkörper und Institution, Walter de Gruyter, 2013, S. 51, Google books. --Altkatholik62 (Diskussion) 15:03, 18. Jan. 2016 (CET)

Soziale Zurückweisung --Hinnerk11 (Diskussion) 15:09, 18. Jan. 2016 (CET)

In Bezug auf die Folgen einer Exkommunikation gebraucht Christian Jaser ebenfalls den Begriff „Kontaktsperre“ (Ecclesia maledicens: Rituelle und zeremonielle Exkommunikationsformen im Mittelalter, Mohr Siebeck, 2013, S. 35, Google books. --Altkatholik62 (Diskussion) 15:10, 18. Jan. 2016 (CET)
Ich würde sagen "Ächtung" wäre der gesuchte Begriff (und so würde ich auch "shunning" übersetzen. --Maresa63 Talk 15:13, 18. Jan. 2016 (CET)

Ächtung gibts auch noch--Markoz (Diskussion) 15:14, 18. Jan. 2016 (CET)

Zitat aus dem Artikel "Meidung" (um dessen mögliche Löschung es hier ja offenbar geht): Als Gegenbegriff von Meidung als Soziale Ausgrenzung wurde in der deutschen Sprache Inklusion und Teilhabe gegen Ende des letzten Jahrhunderts entwickelt. Damit sollte, nach meinem Logik- und Sprachempfinden, "Meidung" ein Synonym für Exklusion oder "Ausschluss" sein. Geoz (Diskussion) 15:29, 18. Jan. 2016 (CET)
Boykott, sofern das ein deutsches Wort ist. --Logo 16:05, 18. Jan. 2016 (CET)
Boykott ist ein englischer Name und ein daraus entstandener irisch/englischer Begriff. --Dansker 16:24, 18. Jan. 2016 (CET)

Es ging mit exizit um eine Fachbegriff. Ich habe inzwischen zweifel ob shunnig mehtr als ein häufiger Alltagsbegriff ist. Von den genannten würde ich nur Ächtung (mit mittelalterliche Konnotation) und Exklusion für Möglich halten. Jedenfalls Danke für die Vorschläge Catrin (Diskussion) 18:09, 18. Jan. 2016 (CET)

Negative Integration wäre ein Fachbegriff, bezieht sich aber darauf, dass eine in-group durch die Ausgrenzung (out-group) geschaffen wird. --Commons2Contributor2 (Diskussion) 18:38, 18. Jan. 2016 (CET)

Im beruflichen Umfeld wäre das wohl Mobbing. Thogru (Diskussion) 11:55, 19. Jan. 2016 (CET)

Gerät gesucht

Hallo zusammen,

ich bin von der Auswahl am Markt etwas überfordert und wäre deswegen dankbar für Hinweise, auf was zu achten ist und natürlich auch für konkrete Erfahrungen und Empfehlungen. Momentanes Setup als Fernseh-Ersatz: Ein (recht neuer und großer) HP-Monitor, an einem HDMI-Port ein DVB-C-Receiver, an einem anderen ein Chromecast-Stick. Um DVDs und nicht-Chromecastfähige Streams (z.B. ARD Mediathek) abszuspielen ggfs. ein Laptop über Displayport. Das Ganze ist nicht besonders elegant, zusätzlich steht ein Umzug an im Zuge dessen der Kabelanschluss durch digitalen Satellit ersetzt wird. Ich würde also gerne ein Gerät anschließen, über dass ich sowohl fernsehen kann (inkl. Receiver, Hauptnutzer: Meine Frau, sollte also unkompliziert anwählbar sein und nicht über 3 Untermenüs), DVDs (Blu-rays) abspielen als auch Sendungen aufzeichnen kann (Festplattenrekorder). Beim letzten Punkt verhindern viele Anbieter wohl das Vorspulen, wenn dem nicht so wäre, wäre das Klasse. Import/Export an externe Platte oder NAS wäre super, ist aber keine Pflicht. Aufnahme von/auf DVD/Blu-ray brauche ich nicht, Onlinedienste ebenfalls nur optional. Kann mir jemand weiterhelfen? --Studmult (Diskussion) 16:45, 18. Jan. 2016 (CET)

Möchtest Du ein Gerät zum Anschluss an den bestehenden Bildschirm oder darf es ein Gerät sein, das seinen Bildschirm gleich mitbringt? Da würde ich zu einem Smart TV mit eingebautem DVD/BD-Player, HbbTV und DVB-C/S2/T-, ggf. auch DVB-T2-Tuner raten. Da kannst Du eine externe Festplatte als Festplattenrekorder anschließen. --Rôtkæppchen₆₈ 01:25, 19. Jan. 2016 (CET)
Würde den Monitor eigentlich gerne behalten. Es sei denn, ein komplettes neues smartes TV-Gerät kostet praktisch das gleiche wie ein externes für den Monitor... --Studmult (Diskussion) 07:32, 19. Jan. 2016 (CET)

Supermarktkasse

Hat man als Kunde eines Supermarkts das Recht, die gekauften Waren an der Kasse in angemessener Zeit derart einzupacken, daß man diese nicht aus der nachströmenden Flut der Waren des nachfolgenden Kunden herausfischen muß ?

In dem Markt gibt es keine Packzone. --89.13.101.246 19:59, 18. Jan. 2016 (CET)

--89.13.101.246 18:59, 18. Jan. 2016 (CET)

Eher Nicht. Man muß ja in der Warteschlange die Kleingeldsuche irgendwelcher schlechtsichtiger Mitbewohner die vor einem drann sind ertragen.--Markoz (Diskussion) 19:08, 18. Jan. 2016 (CET)
Also, ich handhabe das so: Die Waren werden nicht auf einen Haufen auf das Band gelegt, und erst, wenn ich alles eingepackt oder in den Wagen verstaut habe, kommt der Geldbeutel raus. --Heletz (Diskussion) 19:13, 18. Jan. 2016 (CET)
Ein typisches Beispiel dafür, dass nicht alles in juristische Kategorien gequetscht werden kann. Gesunder Menschenverstand, Höflichkeit und praktische Überlegungen reichen hier vollkommen aus - wenn nicht, tun's ein paar deutliche Worte.
Wenn man's aber unbedingt braucht: Selbstverständlich hat man das Recht, seine Ware in angemessener Zeit einzupacken, und zwar ohne dass man dabei ständig auf nachströmende Ware des nächsten Kunden achten müsste. Einklagbar ist dieses Recht aber wohl eher nicht.
Habt ein bisschen Verständnis mit der Kassenkraft, die die Schlange abarbeiten will und keine Lust hat, die Meckereien der nachfolgenden Kunden wegen der Trödelei abzukriegen, und beeilt euch beim Einpacken. Hat mit "Recht" und "Anspruch" aber nichts zu tun. --Snevern 19:26, 18. Jan. 2016 (CET)
Ich nutze regelmäßig das "Recht", die Waren hinter der Kasse in den Einkaufswagen zu legen und dann ein paar Meter weiter in aller Ruhe einzupacken. --Eike (Diskussion) 19:31, 18. Jan. 2016 (CET)
Du hast das Recht zu Gehen, Du hast das Recht Dir den Laden auszusuchen, Du hast sogar das Recht, einen Einkaufswagen Deiner Wahl zu benutzen. Wozu kein Recht besteht, ist die Belästigung anderer Kunden und die Behinderung des Geschäftsbetriebs. Zum "Einpacken" haben die meisten Läden Einpackzonen, wenn Du diese nicht nutzt, selbst Schuld, ist nun schließlich Deine Ware, für deren Verbleib Du zu sorgen hast. Hatte heute nen Geizhals vor mir, der sogar den Pappkarton vor der Kasse auf den Boden warf, um dann nach der Kasse zu merken, daß sein Rucksack zu klein ist. Mein Kommentar war entsprechend freundlich. Wer sich benimmt wie ein Schwein, wird behandelt wie ein Schwein. Ich sagte es hier ja schonmal bei den Pfandflaschen, es ist ein Irrtum, daß die Kassierer- und Verkäuferinnen die Sklaven und Opfer von Euch sind. Das hier häufig solche IP-Fragen auftauchen zeigt entweder die Verwahrlosung der Bevölkerung oder die Überstrapazierung des Rechtssystems zur Durchsetzung der eigenen Unartigkeiten.Oliver S.Y. (Diskussion) 19:32, 18. Jan. 2016 (CET)
Was man allerdings machen kann, wenn man drauf Lust hat, ist, sich beim Geschäftsführer zu beschweren, dass die Kassenauslaufzone zu klein dimensioniert ist - IIRC haben viele Supermärkte die Fläche nach den Kassen sehr stark verkleinert, im Bestreben, die umsatzbringende Fläche im Verhältnis zu "toten Verkehrsflächen" zu reduzieren. Einen sinnvollen Kundenservice einfordern, gegenüber den Verantwortlichen (und das ist nicht das stark belastete Kassenpersonal), finde ich akzeptabel. Grüße, Grand-Duc (Diskussion) 19:45, 18. Jan. 2016 (CET)
Und was machst Du bei dem verständlichen Hinweis auf die Einkaufswagen, in die nach der Kasse die bezahlte Ware gelegt werden kann, um anschließend in der Packzone eingepackt zu werden?Oliver S.Y. (Diskussion) 19:51, 18. Jan. 2016 (CET)
Na, da ist die Fläche ziemlich klein geworden. Ist bei uns um die Ecke (Edeka) auch so. --Eike (Diskussion) 20:06, 18. Jan. 2016 (CET)
@Oliver S.Y.: mit dem Wagen (wenn ich einen dabei habe) so verfahren, mich aber gelegentlich trotzdem über eingedelltes Frischobst, zerquetschte Tiefkühltorten, aufgeplatzte Pulvertüten (Zucker, Mehl) und ähnlich beschädigte Ware ärgern (wenn 1,5-Liter Getränkeflaschen umkippen oder das zweite Laufband nicht rechtzeitig stoppt), weil die Person an der Kasse die Sachen schneller über den Scanner zieht, als ich sie von der wohl nicht ganz DIN-A2-großen Fläche wegheben und ausreichend vorsichtig in den Einkaufswagen zurücklegen kann. Außerdem nutze ich oft tragbare Einkaufskörbe, die vor der Kasse zurückgegeben werden müssen. Grüße, Grand-Duc (Diskussion) 20:14, 18. Jan. 2016 (CET)

Man benutze den dafür vorgesehenen Einkaufswagen. Man lege an der Kasse alle Waren wie vorgeschrieben aufs Band. Wenn die Kassiererin ein Stück gescannt hat und es durchgegeben hat, nehme man dieses Stück und lege es zurück in den Einkaufswagen. Wenn bezahlt wird, ist bereits alles wieder im Einkaufswagen. Man zahlt und geht mit seinem Einkaufswagen weiter, damit der nächste Kunde zügig bedient werden. Aber leider ist so ein Einkaufswagen offenbar uncool oder unmännlich, so dass einige Leute lieber darauf verzichten und dann natürlich für Chaos und Verzögerung an der Kasse sorgen, weil sie ihre Waren erst nach der Zahlung wieder unter ihren Arm klemmen müssen anstatt schon vor der Zahlung alles wieder schön im Einkaufswagen zu haben. --84.119.204.210 20:14, 18. Jan. 2016 (CET)

Das "witzige" ist, dass zur Lösung des nachrückenden Verkehrs ohne stocken eine doppelte Ablagezone "erfunden" wurde (zumindest seit den 1980ern Jahren). Dadurch konnte einer in Ruhe einpacken, während der zweite Kunde gleichzeitig abkassiert werden konnte. Was genau hinter der Abschaffung vor ca. zwei Jahren steckte, ist mir bisher nicht erklärlich. Ich tippe auf die geniale Erfindung eines schlauen Betriebswirtschaftlers...--Wikiseidank (Diskussion) 21:21, 18. Jan. 2016 (CET)
Als Einkaufswagenbenutzer will ich nicht, dass die Kassiererin meine Waren in eine 2. Ablagezone nach hinten schiebt, bloß weil mein Vordermann zu blöd war, einen Einkaufswagen zu benutzen und er deswegen "in Ruhe" die 1. Ablagezone blockiert. Ich will dass sie die Waren zu mir schiebt, damit ich die Ware direkt in meinen Wagen legen und damit direkt weitergehen kann. Ich hab's nämlich eilig, wenn ich nach einem langen Arbeitstag abends auch noch einkaufen muss. --84.119.204.210 22:13, 18. Jan. 2016 (CET)
Sei froh, dass Du nur die zweite Ablagezone hast. Ich hab immer die Oma, die meint "Moment, ich hab´s passend...", dann eine Unmenge von Kleingeld auf der Ablage ausbreitet und wenn sie merkt, dass es doch nicht reicht, ganz verbittert zur Kassiererin sagt "Sagen Sie mal, die Tomaten...die waren doch im Angebot!" ... --Optimum (Diskussion) 23:22, 18. Jan. 2016 (CET)
  1. Die Bedingungen sind in Supermärkten unterschiedlich. Dazu kommt eine im Tagesablauf unterschiedliche Frequentierung der Einkaufsstätten. Eine Antwort, die z.B. mit der doppelten Ablagezone argumentiert, ist für bestimmte Supermärkte realitätsfremd und nicht hilfreich. Ebenso gibt es Märkte, bei denen man die Körbe auch nach der Kasse abstellen kann.
  2. Wenn Waren durch die Kassiererin beschädigt werden („ (...) eingedelltes Frischobst, zerquetschte Tiefkühltorten, aufgeplatzte Pulvertüten (Zucker, Mehl) und ähnlich beschädigte Ware“), kann und sollte man sich beschweren und auf einem Umtausch bestehen. Empfindliche Waren wie Obst oder Eier kann man aber auch als Letztes auf das Band legen und Flaschen legen und nicht stellen.
  3. Insgesamt reden wir über Servicewüste. Es werden nicht nur die Kunden gegeneinander ausgespielt sondern auch die Angestellten gegen die Kunden (z.B. durch zu wenig geöffnete Kassen, übermäßigen Zeitdruck beim Kassieren etc.). Ob die in den USA bisweilen eingesetzten „Einpackhelfer“ eine sinnvolle Option sind, ist eine andere und sicher (hinsichtlich der Arbeitsbedingungen und Bezahlung) kontroverse Diskussion.
  4. Wenn keine Packzone existiert oder sehr viel eingekauft wurde ist das zwischenzeitliche Zurücklegen in den Einkaufswagen der angemessene Umgang. Eine Rechtsfrage daraus zu machen ist absurd. --2003:45:4635:C200:31B6:7148:4510:14C6 23:35, 18. Jan. 2016 (CET)
Es wird vom Supermarktbetreiber erwartet, dass der Kunde Einkaufswagen oder -korb nutzt und die Ware unmittelbar, nachdem die Kassenkraft sie über den Scanner gezogen hat, in Einkaufswagen oder korb verstaut. Packzonen an der Kasse, teilweise sogar doppelt und mit „Weiche“ für zwei Kunden, gibt es nur noch in kleinen oder teuren Supermärkten. aber leider gibt auch immer weniger und immer schlechtere Packtische hinter der Kasse. Ich erlebe sogar manchmal, dass die Gemüsewaage hinter der Kasse, die eigentlich dafür da ist, dass Kunden, die das Wiegen von Obst oder Gemüse vergessen haben, dies nachholen können, als Packtisch missnutzt wird. Bei zweien meiner Stammsupermärkte (Kaufland und Netto) gibt es nur einen Packtisch, der gleichzeitig Entsorgungsstation ist. Ich nutze dann einfach eine geschlossene Kasse als Packtisch. --Rôtkæppchen₆₈ 01:56, 19. Jan. 2016 (CET)
Nur geht das nicht, wenn ich bei einem größeren Einkauf das letzte Viertel noch aufs Band packe, während das erste schon über den Scanner gezogen wird und einen kleinen Berg hinter der Kasse bildet (manche Kassierer scheinen einen Sport darin zu sehen, diesen Berg so hoch wie möglich anwachsen zu lassen, bevor der Kunde dazu kommt, ihn wieder zu demontieren). In dem Fall packe ich zwar zügig, aber nicht hastig alles in meinen Einkaufswagen, und erst danach kommt der Geldbeutel ins Spiel. --Kreuzschnabel 08:48, 19. Jan. 2016 (CET)
@unmittelbar nach dem Scannen erwartet: ich kenne mittlerweile schon zwei neurenovierte Supermaerkte, wo die gescannte Ware erstmal 2 Meter weiter faehrt und in eine Sammelbucht gekippt wird, ohne dass man die Chance hat, das Zeugs wieder in den Wagen zu laden: Auf der Hoehe des Kassierers ist eine kleine Plastewand, dann kommt das RFID-Gate, dann so'n Payback-Scann-Dings, und dann ist endlich der Arm lang genug, um sein Zeugs vom Band zu fischen und wieder in den Korb zu stapeln. Derweile ist schon alles ueber den Haufen gefallen. Zum Kassieren nach dem Einpacken muss man dann die Kassenstrasse wieder rauf laufen, aber der nachfolgende Kunde hat seinen Einkaufswagen schon vor den Kassierer geschoben... Auf Anmerkung zum Kassierer, dass das hier aber nicht sehr ergonomisch ist, meinter nur, dass er das nicht so gebaut hat. Hatter ja Recht, und fuer den Geschaeftsfuehrer hab ich auch nicht wirklich die Zeit. Bleibt mir nur, meinen Krempel woanders zu kaufen, und zu hoffen, dass sich das System nicht nochmehr ausbreitet. --Nurmalschnell (Diskussion) 11:41, 19. Jan. 2016 (CET)

Zur Eingangsfrage: Man muss dabei wissen, dass das Kassenpersonal Zeitlimits zum Abfertigen des einzelnen Kunden hat. Ist dir schonmal bei ALDI aufgefallen, dass du es kaum schaffst die Waren nach dem Einscannen vernünftig in den Einkauswagen zu räumen? Oft werden die Waren von der Kassiererin/dem Kassierer in den Wagen geschoben. Bei ALDI weiß ich aus zuverlässiger Quelle einer stellv. Marktleiterin im guten Bekanntenkreis, dass dort genaue Vorgaben bestehen, wie lange ein Kassiervorgang dauern darf. Aber das ist nicht zuletzt eine Folge der Preispolitik "Der Kunde will es so billig wie möglich haben". Ich empfinde es an der Kasse als wohltuend wenn wie zB in Frankreich sehr oft erlebt, Zeit für ein Schwätzchen des Kunden mit dem Kassenpersonal bleibt. Ich werde auch nicht ungeduldig, wenn der "Kopf" der Schlange höflich und hinreichend bedient wird. Aber da ist jeder anders gepolt. Jedenfalls sollte man nicht pauschal das Kassenpersonal für die Vorgaben der Konzernleitung beschuldigen. Nur meine Meinung. VG Thogru (Diskussion) 11:50, 19. Jan. 2016 (CET)

Die Unterschiede bei Kassen von Supermärkten und Discounter werden auch immer mal wieder bei den mittlerweile unsäglichen Verbrauchertestsendungen bei ARD und ZDF thematisiert, Fakt ist aber auch, dass offenbar auch die Supermärkte (etwa REWE) umgerüstet haben, vgl etwa hier, Grund sei Platzmangel insbesondere in Innenstadtfilialen, die Verbraucherschützer raten auch zur Beschwerde direkt bei den Marktketten--in dubio Zweifel? 12:37, 19. Jan. 2016 (CET)

Abführmittel

dem Artikel Cascara sagrada verdanke ich folgende Erkenntnis: „Regelmäßiger Gebrauch von Cascara sagrada wie auch anderer Abführmittel kann zu psychischer Abhängigkeit führen.“ Wie hab ich mir denn eine solche "psychische" Abhängigkeit von Abführmitteln vorzustellen? Oder ist da vielleicht doch eher "physisch" gemeint? Rätselt --Edith Wahr (Diskussion) 19:16, 18. Jan. 2016 (CET)

Eher letzteres.....--Markoz (Diskussion) 19:24, 18. Jan. 2016 (CET)

Du kannst nur noch auf den Topf, wenn Du ein Abführmittel genommen hast, ansonsten denkst Du, daß Du an Verstopfung leidest.Oliver S.Y. (Diskussion) 19:28, 18. Jan. 2016 (CET)
so weit so nachvollziehbar, aber ist das dann wirklich psychisch oder nicht doch eher physisch/physiologisch bedingt? --Edith Wahr (Diskussion) 19:30, 18. Jan. 2016 (CET)
Das ist vermutlich unglücklich formuliert. Gemeint dürfte eine Abhängigkeit sein das Mittel nehmen zu müssen, obwohl man es nicht braucht.--2003:75:AF11:D400:8F8:2C01:50A:8293 19:46, 18. Jan. 2016 (CET)
hm, also wenn ich dem psychisch Abhängigen ein Placebo gebe, auf dem Cascara sagrada draufsteht, und ihm vielleicht noch „Läuft bei dir“ zurufe, dann sollte es also laufen. Ok, damit hätte ichs mir dann halbwegs zusammengereimt. Merci, --Edith Wahr (Diskussion) 19:50, 18. Jan. 2016 (CET)
Was in dem Artikel tatsächlich gemeint ist, weiß ich nicht, psychische Abhängigkeit kann es jedoch gut sein. Meines Wissens kann man gegenüber jedem Stoff und jedem Verhalten psychisch abhängig werden. --BlackEyedLion (Diskussion) 20:00, 18. Jan. 2016 (CET)

Der Hersteller von Dulcolax schreibt zum Thema: „Abführmittel können nicht süchtig bzw. körperlich abhängig machen, weil sie weder ins Gehirn gelangen, noch bestimmte Botenstoffe beeinflussen, die eine Sucht bzw. Abhängigkeit erzeugen. Wichtig: Die Begriffe Abhängigkeit und Gewöhnung sind nicht gleich zu setzen. Bei einer Abhängigkeit (körperlich) sind Entzugserscheinungen bei Absetzten des Medikaments typisch, bei einer Gewöhnung beobachtet man eine Toleranzentwicklung, die mit einem Wirkverlust einhergeht und zu einer Dosissteigerung führen kann. Gewöhnungseffekte sind bei bestimmungsgemäßem Gebrauch mit dem Ziel eines weich, geformten Stuhls für Abführmittel nicht zu beobachten.“ Andererseits: „Für alle Dickdarm-wirksamen Laxanzien gilt, dass sie bei chronischem Gebrauch zur Abhängigkeit mit der Gefahr von Hypokaliämie und Colon-Atonie führen können.“ --Vsop (Diskussion) 22:03, 18. Jan. 2016 (CET)

Da geht es um körperliche Abhängigkeit (das wird in der ersten Erwähnung sogar ausdrücklich betont). Körperliche Abhängigkeit ist nur bei manchen Stoffen möglich.
Die Drogenberatung Wolfsburg schreibt: „Jeder Stoff, der angenehme Gefühle verschafft, birgt das Risiko, davon [psychisch] abhängig zu werden.“ [5] --BlackEyedLion (Diskussion) 00:25, 19. Jan. 2016 (CET)
Danke für die Belehrung. „Regelmäßiger Gebrauch von Cascara sagrada wie auch anderer Abführmittel“ verschafft also angenehme Gefühle? --Vsop (Diskussion) 00:33, 19. Jan. 2016 (CET)
Hast Du nie Deine anale Phase gehabt und mit angenehmem Gefühl nach Herzenslust defäkiert? Mit Obstipation geht das weniger gut. --Rôtkæppchen₆₈ 01:07, 19. Jan. 2016 (CET)
Mit Rotkäppchens Beitrag hat die Diskussion endgültig unterirdisches Niveau erreicht. Chapeau! --Vsop (Diskussion) 03:06, 19. Jan. 2016 (CET)
??
Was er zusammenfasst ist über GoogleBooks nachprüfbares Basiswissen. Darf man hier "Basiswissen" sagen? Play It Again, SPAM (Diskussion) 10:42, 19. Jan. 2016 (CET)
Hm. Es gibt ja verschiedene Varianten des Defäkierens. Die durch Abführmittel meist herbeigeführte war mir aber glaub ich noch nie angenehm. --Eike (Diskussion) 10:46, 19. Jan. 2016 (CET)
Vom Hörensagen weiss ich, dass der systemische Zustand von Obstipation sehr unangenehm ist. Die Leute sind reizbar, fühlen sich "bloated", generell sehr unwohl.
Die Tatsache, dass ihnen die Einahme eines Laxativums "Erleichterung" verschafft - wortwörtlich: kurzzeitige Euphorie - legt das oben Gesagte nahe. Eine weitere Variante ist Laxantienabusus. Vielleicht ist diese Abhängigkeit gemeint? Play It Again, SPAM (Diskussion) 11:02, 19. Jan. 2016 (CET)
Ich kenn den Zustand zum Glück (noch?) nicht so, aber ja, "Erleichterung" kann ich mir sehr viel besser vorstellen als "angenehmes" Defäkieren. --Eike (Diskussion) 11:04, 19. Jan. 2016 (CET)
Wahrnehmungsparameter des Menschen haben ein weiiiites Spektrum.
Einer verdreht beim Schlürfen einer Auster orgiastisch die Augen (und macht entsprechende Geräusche: „Oh-uuungh!“), der andere spuckt sie - begleitet von völlig anderen Geräuschen („Uargh ÄÄh!“) - wieder auf den Teller.
Warum sollte es am anderen Ende ... anders sein ??? Play It Again, SPAM (Diskussion) 11:16, 19. Jan. 2016 (CET)
Ja danke, Geezer. Vielleicht eignen wir uns auch einmal „Basiswissen“ über Taktgefühl und wikt:Distanzverlust an. Zur Sache noch: Abhängigkeit (Medizin) >> Schädlicher Gebrauch von nichtabhängigkeitserzeugenden Substanzen, dort Entstehung und Beispiele für verwendete Substanzen, Abführmittel (Laxantien). --Vsop (Diskussion) 12:06, 19. Jan. 2016 (CET)

Schriftart gesucht (Comiczeichnung einer Netscape-Browser-GUI)

Hallo!

Mich würde mal interessieren, welche Schriftart in diesem Beispielbild (Bilderhoster) für den Text verwendet wurde. Es ist ein Ausschnitt aus einer Bande Dessinée und soll wohl einen Chat in einem Netscape-Fenster auf einem PC, der schwer nach Win98-Ära aussieht, darstellen. Außerdem, das wäre mir auch sehr wichtig, woher kann man diese oder eine ausreichend ähnliche Schrift kostenlos bekommen (am besten wäre ja, falls es was passendes in der Out-of-the-Box-Schriftenausstattung von Ubuntu 14.04 gäbe...)? Danke und Grüße, Grand-Duc (Diskussion) 19:39, 18. Jan. 2016 (CET)

Ein Sans-serif, das "v" ist sehr speziell! Du kannst eine Website wie diese verwenden und eine Probe hochladen, muss aber kleiner sein als die jetzige, etwa 1/3 davon mit dem "v". Play It Again, SPAM (Diskussion) 19:56, 18. Jan. 2016 (CET)
Das hatte ich schon, ohne richtigen Erfolg, ausprobiert... Aber Danke. :-) Grüße, Grand-Duc (Diskussion) 20:16, 18. Jan. 2016 (CET)
jo danke :-)
Ja, eine fette Grotesk, die durch die „schmutzigen“ Konturen eine Grunge/Industrial-Anmutung hat. Allerdings ist die Auflösung derart mies, dass nicht einmal klar ist, ob dieser Effekt Teil des Schriftentwurfs oder nur durch die Reproduktion entstanden ist. Hinzu kommt, dass die Schriftprobe extrem wenige unterschiedliche Buchstaben zeigt (wenn ich richtig gezählt habe, sind es nur 18 verschiedene) und gerade diejenigen Buchstaben fehlen, die meistens charakteristische Unterschiede aufweisen (wie z. B. Großbuchstaben oder das kleine g, k oder y). Das macht eine Identifizierung extrem schwierig. Die einzige auffallende Besonderheit ist in der Tat das asymmetrische kleine v mit der gerundeten rechten Flanke. Das ist sehr selten, ich habe aber auf Anhieb auch keine passende Schrift finden können. --Jossi (Diskussion) 20:26, 18. Jan. 2016 (CET)
@Jossi: Hier sind nochmal weitere Proben: Nummer 1 und Nummer 2, leider ebenfalls nicht besonders diversifiziert, zumindest ein y ist dabei. Danke für die Mühen und Grüße, Grand-Duc (Diskussion) 20:38, 18. Jan. 2016 (CET)
So, das war 'ne lange Nacht.
Bin überzeugt, dass es ein Centennial-Derivat ist; hochgezogene Buchstaben, 1970er Jahren.
Aber: Es gibt grosse Fragen, die kann man ohne vorherige Beantwortung der kleinen Fragen nicht lösen.
Also: Wer kennt jemanden, der weiss, wie der Buchstabenstil des "v" genannt wird? Play It Again, SPAM (Diskussion) 09:11, 19. Jan. 2016 (CET)
Sieht aus wie Chicago (Schriftart). --Komischn (Diskussion) 09:19, 19. Jan. 2016 (CET)
 
Charakterisches 'w' (analog 'v')
Wollte auch gerade schreiben, die Schriftart erinnert an Chicago. War bis Ende der 1990er-Jahren die Hauptschrift des Mac OS. Ich würde bei deinem Sample aber eher auf ein Chicago-Derivat tippen (oder es wurde wirklich schlecht gescannt/abfotografiert/gedruckt). --Gestrandete 55-cm-Geschirrspülmaschine (Diskussion) 09:32, 19. Jan. 2016 (CET)
Die Proportionen - Freiraum beim "a" oben und unten, Länge/Bauch beim "d", das "v" etc. - stimmen. Es ist dann wohl eine Chicago, aber TrueType (sowas findet sich im Web zum download). Play It Again, SPAM (Diskussion) 10:24, 19. Jan. 2016 (CET)
Bingo! Auf die Auskunft ist doch noch Verlass. --Jossi (Diskussion) 12:13, 19. Jan. 2016 (CET)

Wikipedia Android-App

Warum steht in der Android-App der Wikipedia in der Rubrik "Was geschah am" im Gegensatz zur Webversion kein tagesaktuelles Datum? --2003:56:CF65:7EB4:A0E1:2BBC:9155:597D 03:25, 19. Jan. 2016 (CET)

WP:FZW ---91.3.29.200 05:49, 19. Jan. 2016 (CET)
Platzmangel?! also ich schätze bei kleineren Bildschirmen ist es störend, wenn die Überschrift zweizeilig erscheint--in dubio Zweifel? 13:42, 19. Jan. 2016 (CET)

Wachstumsbranche

Immer wieder hört und ließt man von der Krise. Fast alle Geschäftsbereiche scheinen betroffen zu sein, dennoch wachsen die großen Volkswirtschaften. Deshalb meine Frage (mit Recherche komme ich nicht weiter), in welchen Bereichen wächst die Wirtschaft in Deutschland eigentlich noch? Was sind die Boom-Betriebe? --178.6.52.112 11:22, 19. Jan. 2016 (CET)

Krise? Welche Krise? In Deutschland jedenfalls nicht. --Jossi (Diskussion) 12:20, 19. Jan. 2016 (CET)
Und wie dieser Ifo-Index funktioniert, kannst Du Dir hier ansehen. --2003:76:E4C:F1AD:44A9:1C06:8201:5545 12:34, 19. Jan. 2016 (CET)

Alten Neuwagen anmelden

Situation: Anfang der 1990er Jahre wurde ein hochwertiger Neuwagen gekauft, unangemeldet in die Garage gestellt und dort stehen gelassen. Heute verweigert die Zulassungsstelle die Zulassung, weil der Wagen nicht den für Neuanmeldungen nötigen Abgaswerte usw. erfüllt. Wie kann man so ein Auto auf die Strasse bekommen? Kurz im Ausland zulassen, damit er schonmal zugelassen war? Wenn ja, wo? --2003:76:E4C:F1AD:44A9:1C06:8201:5545 11:44, 19. Jan. 2016 (CET)

Warten bis der Wagen 30 Jahre alt ist und dann mit H-Kennzeichen anmelden. Ich weiß aber nicht ob die 30 Jahre erst ab Erstzulassung gerechnet werden. --Mauerquadrant (Diskussion) 11:59, 19. Jan. 2016 (CET)
Anhand von Typschlüsselnummer aus der Zulassungsbescheinigung und genauer Bezeichnung von Wagen und Motor recherchieren, ob es eine typgeprüfte Nachrüstlösung gibt. Diese von einer Fachwerkstatt einbauen lassen und den Papierkram der Zulassungsstelle vorlegen. --Rôtkæppchen₆₈ 13:07, 19. Jan. 2016 (CET)
Die Mitarbeiter hinter dem Schalter der Zulassungsstellen sind mit solchen Sonderfällen regelmäßig überfordert. Da redet man mit dem Dienstellenleiter. Es ist Unsinn, die Zulassung zu verweigern, wenn das Fahrzeug seinerzeit zugelassen worden wäre, dürfte es noch heute so rumfahren. Ggf. würde ich mir vom Kraftfahrt Bundesamt KBA, Technischer Dienst bestätigen lassen, dass die damalige Typzulassung noch unverändert gilt. Natürlich bekommst du keinen Aufpapper als umweltfreundliches Fahrzeug.--2003:75:AF16:8F00:89E0:8EF8:3939:B02C 13:18, 19. Jan. 2016 (CET)
Was immer geht, ist eine Zulassung als Eigenbau mit Sondergutachten vom TÜV. --Pölkkyposkisolisti 13:22, 19. Jan. 2016 (CET)
(BK)Jepp. Mein Bekannter, der obiges gemacht hat, müsste für jede einzelne Umweltzone, die er befahren will, eine separate Ausnahmegenehmigung beantragen. Das ist ihm zuviel Aufwand; deswegen fährt er nur in seiner Heimatumweltzone mit Ausnahmegenehmigung rum. --Rôtkæppchen₆₈ 13:23, 19. Jan. 2016 (CET)
"Hochwertig" - ich habe auch keine Ahnung, was man da machen kann. Ich weiß aber, wen ich fragen würde, nämlich die Firma (BMW? Mercedes?), die den Wagen damals hergestellt hat. NfdA (Diskussion) 13:44, 19. Jan. 2016 (CET)
Mein ich hat seinem 34 Jahre alten Mercedes, weit weg von heutigen Umweltstandards, letztes Jahr ein H-Kennzeichen beantragt. War nicht ganz billig, denn der Sondergutachter musste den erhaltenswerten Zustand und Originalität bestätigen. Selbst das neue Radio wurde beanstandet und musste durch das alte ersetzt werden.) Natürlich kann mein ich/ sein Mercedes nicht ohne Ausnahmegenehmigung in ausgewiesene und aktive Umweltzonen einfahren, aber das will mein ich auch nicht. Die Kröte muss es Schlucken. Der alte KFZ-Brief wurde übrigens entwertet und ein neuer nach derzeitigem Muster ausgestellt. Passt nicht zum Altertum, aber der alte blieb erhalten.--2003:75:AF16:8F00:89E0:8EF8:3939:B02C
Das ist alles sehr bedauerlich. Historische Umweltverschmutzer gehören in Museen. --BlackEyedLion (Diskussion) 13:58, 19. Jan. 2016 (CET)
@„die Firma (BMW? Mercedes?), die den Wagen damals hergestellt hat“: Die oder deren Werksniederlassung sind oft unglaublich hilfsbereit, bieten aber selbst keine Nachrüstungen zur Besserung der Schadstoffklasse an. Es müssen also die vielen kleinen Tüftler angefragt werden, die typgeprüfte Nachrüstsets, also Oxidationskatalysatoren, Kaltlaufregler und Ähnliches, anbieten. Ich habe da Fälle erlebt, wo sich die Suche über Monate hingezogen hat, wohlgemerkt bei Fahrzeugen süddeutscher Hersteller. --Rôtkæppchen₆₈ 15:41, 19. Jan. 2016 (CET)
Was du sagst, ist sicherlich alles richtig. Ich meinte aber etwas anderes. Keine Niederlassung, sondern direkt der Hersteller. Und nicht zur Lösung des technischen Problems, sondern des rechtlichen.
Diese Hersteller sind sozusagen auch offiziell stolz auf ihre hochwertigen und langlebigen Autos; das drückt sich aus in Ersatzteilvorhaltung auch für sehr alte Wagen der Marke, in musealen Aktivitäten, etc.
Deshalb wissen die vielleicht besser als jeder andere, wie man das rechtlich in Ordnung bringen kann. Daher direkt an den Konzern, dort schauen, wer für Oldtimer zuständig ist, dort so weit nach oben wie möglich. Dorthin schreiben, kurz und knackig (es ist ein Ingenieur und wohl kein junger...), dabei auch um Weitergabe der Frage an eventuell kompetentere Kollegen bitten. NfdA (Diskussion) 01:23, 20. Jan. 2016 (CET)

(nach links) Das Problem, ist nicht unbekannt. Es gilt normalerweise das Datum der Erstzulassung. Wenn es ein solches nicht gibt, wird es schwierig. Manchmal gelingt es, die Zulassungsbehörde über einen Nachweis des Produktionsdatums (z.B. Abgleich der Fahrgestellnummer mit Produktionsdaten - manchmal in Fachbüchern abgedruckt) davon zu überzeugen, dass es sich wirklich um ein altes Auto handelt. Auch ein H-Kennzeichen erhält solch ein Auto nur mit großem Aufwand (unabhängig vom Zustand), weil das Datum der Erstzulassung noch keine 30 Jahre her ist.

Ich rate dringend davon ab, den Brief und die Fahrgestellnummer eines Schrottfahrzeugs zu übernehmen. Das wäre streng genommen Steuerhinterziehung (wg. "Erschleichung" eines H-Kennzeichens, bei Bj. 1990 noch nicht relevant) oder möglicherweise ein Verstoß gegen die Zulassungsvorschriften. Es soll aber vorgekommen sein, dass dieser Weg problemlos funktioniert hat. --Vertigo Man-iac (Diskussion) 14:13, 19. Jan. 2016 (CET)

rüste den doch auf Gas um kostet etwa 2000 Euro--Markoz (Diskussion) 15:45, 19. Jan. 2016 (CET)
Ohne jede Information darüber, um was für einen Motor es sich handelt, halte ich das für eine sehr mutige, geradezu verwegene Aussage :-) --Kreuzschnabel 15:51, 19. Jan. 2016 (CET)
Ist aber eine gute Idee ne Bekannte hatte ein Golfcabrio, so 98 Baujahr der durfte nirgendwo mehr rein , die hat den auf Gas gesetzt,dat ging...und freie Fahrt für freie Bürger.--Markoz (Diskussion) 15:56, 19. Jan. 2016 (CET)

Wie kommt Artikel ins Netz???

Ich habe den Artikel "Jutta Sauer" verfasst:letzte Version 3.12.2016: wie bekomme ich den jetzt ins Netz? Ich bin neue Autorin, habe derzeit großen Zeitmangel und nichts gefunden, wie ich da einfach vorgehen muss.. Also: kurz knapp Anweisungen, die ich schnell umsetzen kann, damit ich das abhaken kann. --FannyBeyer (Diskussion) 12:34, 19. Jan. 2016 (CET)

Der Artikel ist schon "im Netz", hier (klicken). Er wurde auch schon von freundlichen Helfern verbessert. Bei Google, falls du das meinst, taucht er dann von allein bald auf. --Eike (Diskussion) 12:38, 19. Jan. 2016 (CET)
Bei Google erscheint er auch schon auf der zweiten Seite, zweiter Eintrag. Der wird bestimmt noch steigen;)--in dubio Zweifel? 13:19, 19. Jan. 2016 (CET)
PS: Der Artikel wurde erst kürzlich in den WP:Artikelnamensraum verschoben, erst dort wird er von Google indexiert.--in dubio Zweifel? 13:45, 19. Jan. 2016 (CET)
Jutta Sauer wurde bereits gesichtet. Danke für den Artikel! Erledigt? --Hans Haase (有问题吗) 13:30, 19. Jan. 2016 (CET)
Die wesentlichen Aussagen sollten noch belegt werden. Hab mal den Baustein draufgesetzt. --Kreuzschnabel 15:30, 19. Jan. 2016 (CET)
Aber wenn da steht "Näheres siehe Diskussionsseite" und Diskussionsseite ist dann ein Rotlink, sieht das immer so aus, als wären dem Bausteinsetzer am Ende doch keine Mängel eingefallen.</Mäkelmodus> --Optimum (Diskussion) 16:59, 19. Jan. 2016 (CET)

Briefmarke nicht gestempelt, sondern ausgekreuzt

Liebe Auskunft, ich habe heute eine Maxi-Briefsendung der Deutschen Post, korrekt frankiert mit 1,65 Euro, erhalten. Die zwei Briefmarken sind nicht gestempelt, sondern von Hand ausgekreuzt. Ist das üblich? Wie nennt man diese Art der Entwertung? Vielen Dank für Antworten! --BlackEyedLion (Diskussion) 20:04, 18. Jan. 2016 (CET)

Entwertung_(Philatelie)#Nachentwertung Gruss --Nightflyer (Diskussion) 20:06, 18. Jan. 2016 (CET)

Seit ca. 1840 zum Leid der Briefmarkensammler im Gebrauch :-) -jkb- 20:12, 18. Jan. 2016 (CET)

Wenn kein Stempel drauf ist, halten es die meisten Briefmarkensammler auch für unbrauchbaren Müll, der meist entsorgt wird. Oder sammeln verkaufen das über ebay. Da bekommt man gut die Hälfte des Wertes, obwohl es natürlich an der Grenze zur Beihilfe zum Betrug ist. --Hachinger62 (Diskussion) 16:54, 19. Jan. 2016 (CET)

Trident-Abkommen

Kann mir jemand erklähren was das hier erwähnte Trident-Abkommen ist? Wenn ich google geht es in Richtung START aber dort wird kein Trident genannt und bei genaur Suche gibt google keine Ergebnisse.--Sanandros (Diskussion) 06:28, 19. Jan. 2016 (CET)

 
Wappen der Ukraine
In Bezug auf die Ukraine ist Trident immer eine Anspielung auf das Trysub oder Trident genannte Wappen der Ukraine. --Rôtkæppchen₆₈ 08:05, 19. Jan. 2016 (CET)
Das liegt nah. Im England des konservativen Telegraph, auf den sich der ZEIT-Artikel bezieht, versteht man ganz aktuell unter Trident aber wohl eher das en:UK Trident programme, die britischen U-Boote mit en:UGM-133 Trident II-Raketen, über deren Zukunft derzeit heftig gestritten wird, en:Successor to the UK Trident system. Gerade erst ist Labour/Jeremy Corbyn den diesbezüglichen Plänen der Regierung Cameron entgegengetreten, http://www.nytimes.com/2016/01/18/world/europe/british-labour-leader-offers-compromise-on-trident-program.html?_r=0. Da passt es doch gut, unter der Überschrift „Russia accused of clandestine funding of European parties“ einen Experten zu präsentieren, der von einer „Russian campaign [...] in a grey area, operating covertly“ spricht „with the clear aim of weakening Western will to fight, maturing doubts over Nato, the EU, Trident and economic sanctions“. Aus „Trident“ „das Trident-Abkommen“ zu machen, ist ein Übertragungsfehler der ZEIT. --Vsop (Diskussion) 09:28, 19. Jan. 2016 (CET)
PS: „The term "Trident" is popularly used to refer to Britain's entire current system of nuclear weapons“, en:Successor to the UK Trident system. --Vsop (Diskussion) 14:03, 19. Jan. 2016 (CET)
Mal bei Trident ergänzt. Thx.--Sanandros (Diskussion) 21:40, 19. Jan. 2016 (CET)

DNA Analyse aber mit was denn?

Gerade wurde nach einem Überfall auf einen Geldtransporter belastendes DNA Material gefunden das vermuten lässt das daran ehemalige RAF Mitglieder beteiligt waren. Von zwei wurden DNA Spuren gefunden. Wie denn? :) Die haben sich ja wohl kaum beide die Nase geputzt und das Taschentuch liegen lassen oder ein Wettspucken an die Frontscheibe veranstaltet. Aus was wird denn in so einem Fall das DNA Material extrahiert? Suchen die echt nach einzelnen Haaren oder Hautschüppchen? --84.149.232.44 13:33, 19. Jan. 2016 (CET)

Ja.[6] --BlackEyedLion (Diskussion) 13:36, 19. Jan. 2016 (CET)
Gesichtsmasken - Haare, Hautschuppen, Speichel mit Zellen ... Da ist "Abby" im DNA-Wunderland! Grossartig, gell? Play It Again, SPAM (Diskussion) 13:46, 19. Jan. 2016 (CET)
Ein Witz (aber nur für Akademiker!): Sie benutzen Masken, damit sie nicht von Zeugen erkannt werden (Zeugenaussagen haben eine Validität von etwa 50 %) und lassen daran DNA-Spuren (die mit 99,999... prozentiger Wahrscheinlichkeit zugeordnet werden können!). Das sagt und auch etwas über die intellektuellen Statistik-Fähigkeiten der Räuber aus.... Bei WP hätten die keine Chance! ;-) Play It Again, SPAM (Diskussion) 13:51, 19. Jan. 2016 (CET)
Ja, das ist schon lange bekannt. Bei der Entführung der Schlecker-Kinder saugten die Erpresser die Wohnung und nahmen den Staubsaugerfilter mit. Das reichte damals, heute nicht mehr. --Hans Haase (有问题吗) 13:48, 19. Jan. 2016 (CET)
Hoffentlich waren die Wattestäbchen sauber, vgl Heilbronner Phantom--in dubio Zweifel? 13:50, 19. Jan. 2016 (CET)
Nur welcher Anfänger hat das Zeug in den inzwischen vergangenen 17 Jahren nicht verbrannt? --Hans Haase (有问题吗) 13:52, 19. Jan. 2016 (CET)
Wie alt ist denn der Transporter? Kann ja auch im Vorbesitz der RAF gewesen sein. Das da DNA von Terroristen gefunden wurde, sagt nicht, dass sie an dem Überfall beteiligtwaren. Nur dass die mal im Wagen gesessen haben.--Markoz (Diskussion) 15:48, 19. Jan. 2016 (CET)
Da ist die Staatsanwaltschaft schon weiter, siehe Überfall auf Geldtransporter in Stuhr – drei ehemalige Mitglieder der RAF verdächtig. Keine Hinweise auf terroristischen Hintergrund. Pressemitteilung. In: Staatsanwaltschaften.Niedersachen.de, 19. Januar 2016. --Andropov (Diskussion) 15:54, 19. Jan. 2016 (CET)
Wunderbar: dieser Link gibt Antworten! Man muss ihn aber lesen, daher hier ganz kurz: Es gab drei Autos: Das Opferauto, das Täterauto und das Fluchtauto, und die DNA-Spuren fanden sich in den beiden letzteren. Da bereits Artikel zu allen drei Verdächtigen erstellt sind und da der Fall auch morgen in Aktenzeichen XY ungelöst kommen wird, ist aus enzyklopädischer Sicht alles klar. --Pp.paul.4 (Diskussion) 17:16, 19. Jan. 2016 (CET)
Zum Niveau dieser Helden: "Auf der Bekleidung zumindest eines der Täter stand auf dem Rücken „POIZEI" (Polizei ohne L)" aus dem bericht der Staatsanwaltschaft (oben verlinkt). --Hachinger62 (Diskussion) 17:19, 19. Jan. 2016 (CET)
Da könnte man vermuten, das „L“ sei bei einer alten Weste abgegangen, aber Poizei ist auch in der Literatur etabliert. --Pp.paul.4 (Diskussion) 01:00, 21. Jan. 2016 (CET)

Da wollten die drei wohl ihre Rentenkasse auffüllen. Die Million DM, die sie vor ein paar Jahren erbeutet hatten, scheint wohl ausgegangen zu sein. BTW, schön war heute auch ein Interview mit Christian Pfeiffer dazu. Laut ihm fänden die ja keinen Job zum Lebensunterhalt, weil sie ja kein Führungs- oder Arbeitszeugnis vorweisen könnten. Nee, ist klar: Mit Sturmgewehr Geldtransporter überfallen, aber vor einer Urkundenfälschung zurückschrecken... Kopf -> Tisch. 18:07, 19. Jan. 2016 (CET) (ohne Name signierter Beitrag von Morty (Diskussion | Beiträge))

Wie kann man denn den Pfeiffer auch nur erwähnen ...? ^^ --Heletz (Diskussion) 19:00, 19. Jan. 2016 (CET)

Der genannte Volker Staub der hier ein Artikel hat ist nicht RAF Mitglied sondern Musiker--Markoz (Diskussion) 19:43, 19. Jan. 2016 (CET)

schau mal hier, er hat noch einen Ernst vor seinem Volker, und einen Wilhelm hinterher (bitte mal prüfen, vgl Guttenberg ;) Nachtrag: stimmt wohl--in dubio Zweifel? 21:07, 19. Jan. 2016 (CET)

Problem mit neuem Handy

Hat jemand schon mal folgendes Problem mit Android gehabt? Ich hab mit dem alten Handy mein Adressbuch mit meinem Google-Konto verknüpft gehabt. Nun hab ich mir ein neues Handy gekauft und die Daten synchrinisiert. In Whats App werden die Nummern angezeigt (ohne Namen), im Adressbuch überhaupt nicht.--93.228.103.189 21:40, 19. Jan. 2016 (CET) --93.228.103.189 21:40, 19. Jan. 2016 (CET)

Nein. Aber ein Hinweis: Whatsapp speichert m.W. nur die Nummern, und holt die Namen aus dem Adressbuch. Soll heißen, dass einfach das Adressbuch nicht synchronisiert ist. Hat nichts mit Whatsapp zu tun. --46.189.28.219 22:06, 19. Jan. 2016 (CET)
Hattest Du das Googlekonto mit Deinem Benutzerpasswort eingerichtet oder nur die Googlemail-Adresse eingegeben? --Hans Haase (有问题吗) 23:42, 19. Jan. 2016 (CET)

Autogas im Langstreckenbetrieb

Ich bitte schon mal vorsorglich darum, hieraus keinen Glaubenskrieg zu machen :-) Einklich fahre ich gern Diesel, mein derzeitiger Wagen läuft seit 150 Mm störungsfrei und macht noch keine Anstalten, das zu ändern. Könnte so weitergehen, wenn da nicht die Feinstaub- und NOx-Belastung wäre und auch nicht schon einige Städte über Dieselfahrverbote nachdenken, um ihre NOx-Meßwerte im Rahmen zu halten – darunter solche, in denen ich geschäftlich zu tun habe. Als schadstoffarmer und budgetschonender Kraftstoff bietet sich Autogas an. Nun fahre ich aber mit 30-35 Mm jährlich nicht gerade wenig, das meiste davon auf Strecken in der Größenordnung 300 km. In etlichen Forenbeiträgen ist zu lesen, daß Gasmotoren recht wartungsintensiv sind und der höhere Verschleiß von Ventilen etc. gelegentlich auch zu kostspieligen Motorüberholungen führt, die den Kostenvorteil des Kraftstoffes an sich wieder zunichte machen. Hat jemand in der Gemeinde da Erfahrungen? Angenommen, ich kaufe dieses Jahr ein acht Jahre altes LPG-Fahrzeug mit 140 Mm Laufleistung, wie realistisch wäre dann zu erwarten, nochmal 100 Mm draufzufahren, ohne jährlich einige Tausender in der Werkstatt zu lassen? --Kreuzschnabel 23:40, 18. Jan. 2016 (CET)

Dieselfahrverbote werden sich m.E. nicht durchsetzen lassen. Bei den Gas-Fahrzeugen gibt es viele Unterschiede seitens der Technik sowie seitens der Hersteller der Anlagen. Ich kenne einige, die mit ihren Anlagen zufrieden sind und einige, die permanent Probleme haben. Da sollte man schon wissen, welche Anlage von welchem Hersteller in welchem Auto sitzt, um eine Prognose wagen zu können. Vielleicht ergänzt Du das noch, damit Tippgeber genauer drauf eingehen können. Ich selbst kann leider nicht konkret helfen, da ich selbst so eine Anlage nicht habe und Benzin und Diesel treu bleibe. --2003:76:E4C:F1AD:587F:822D:3036:6D9B 01:10, 19. Jan. 2016 (CET)
Ich habe da noch nichts Konkretes im Blick, mir geht es mehr darum, mir erst mal ein allgemeines Bild zu machen. Wenn es konkret wird, würde ich schon einen ab Werk mit Gasanlage ausgestatteten (und damit hoffentlich auch „gasfesten“) Wagen bevorzugen, damit im Fall eines Schadens nicht Umrüster und Motorhersteller einander den schwarzen Peter zuschieben.--Kreuzschnabel 08:40, 19. Jan. 2016 (CET)
Diese Horrorstories stammen aus der guten alten Zeit des verbleiten Sprits, bei dem die Ventilsitze noch ungehärtet waren. Gibt es schon seit über 25 Jahren nicht mehr. Evtl. muss das Ventilspiel häufiger nachgesehen und nachgestellt werden, aber das sollte man auch bei Benzinbetrieb machen. Allerdings braucht die LPG-Anlage nach einigen Jahren Betrieb eine Generalüberholung, deren Einspritzventile haben auch nur eine begrenzte Lebensdauer. Die normale jährliche Wartung der LPG-Anlage kannst du mit 100 Euro ansetzen. Nachschau, Filter ersetzen. Ventilspiel nachschauen/einstellen irgendwo im Bereich 100€, vielleicht alle 50.000..100.000km nötig. Du merkst es zuerst am Mehrverbrauch, dann an nachlassender Leistung. -- Janka (Diskussion) 01:26, 19. Jan. 2016 (CET)
Autogas verbrennt bei höherer Temperatur als Benzin, somit muss der Motor dafür geeignet sein, diese Belastung auch auszuhalten. Selbst aktuelle Motoren haben damit Probleme. Sonst brennen die Ventile einfach irgendwann durch, was dann spontan hohe Kosten nach sich zieht da ein Ventilwechsel arbeitsintensiv ist und damit die Kostensenkung ad absurdum führt. Zu beachten ist meiner Meinung nach: Wer steht für den Betrieb gerade? Wenn die Umrüstung durch einen Drittanbieter erfolgt, schließt dieser normalerweise Motorschäden aus. Der Hersteller aber auch. Das Risiko trägt also der Besitzer. Risiko: Ausfall des Motors mit 1-2 Wochen Reparaturzeit durch einen Fachbetrieb mit niedrigen vierstelligen Kosten. Nebenschauplatz: Die Hauptuntersuchung sowie oft auch die reguläre Wartung des Autos werden teurer (meiner Erfahrung nach 20-30€ je Vorgang). Pannenhilfe wird auch problematisch. Ich würde auf jeden Fall vor dem Kauf ein paar lokale Schrauber die selbst Umrüsten befragen, ob da Erfahrungswerte zur Umrüstung dieses Modells mit dieser Anlage bestehen, dazu ist es oft hilfreich, den genauen Typ der eingebauten Anlage zu kennen. Mit der Betankung ist das eigentlich unproblematisch, im Internet findet man immer was in der Nähe. Ich fand es allerdings lästig das nach Umrüstung die Reichweite so gering war. Meines Wissens rechnet man ca. regulärer Verbrauch in Benzin + 20%, was dann schnell bei 10 Litern landet. Da der Tank aber nur zu 80% gefüllt werden darf, ergibt dies bei Reserveradausführung oft nur 350km Reichweite.Quark (Diskussion) 02:36, 19. Jan. 2016 (CET)
Ein externer Fuhrparkunternehmer der Firma hat die Gasautos wieder abgeschafft, weil die Wagen durchschnittlich alle 14 Tage, bei 1000 km Laufleistung pro Tag in Wartung mussten. Die Diesel machten mindestens das doppelte. Gas ist etwas für Idealisten die sich (um der Umwelt willen) die Zeit nehmen. Die öfteren Werkstattbesuche werden den Kosten und Zeit nach in den Vergleichsrechnungen meist unterschlagen. Deshalb bleiben bei den Taxiunternehmen die Gasautos auch Exoten.--2003:75:AF16:8F00:982D:8FA2:F567:58CB 07:56, 19. Jan. 2016 (CET)

Also unser Wagen hat jetzt knapp 80.000 km mit umgebautem Autogas und keinerlei Probleme. Volltanken für 25 Euro ist eine einzige Freude. Wobei es tatsächlich darauf anzukommen scheint, entweder eine vertrauenswürdige Werkstatt oder Autogas gleich ab Werk zu haben. Wer hätte es gedacht, aber jeder Tankstopp zaubert ein Lächeln auf meine Lippen. -- southpark 08:45, 19. Jan. 2016 (CET)

Die Wolga-Taxis in der DDR waren in Berlin fast ausschließlich mit Gas betrieben und mußten nicht öfter in die Werkstatt als andere Wagen. Die Motoren haben etliche 100.000 km gehalten, allerdings waren die sowieso extrem robust gebaut. im UAZ hatte ich einen mit knapp 1 Mio. km, ein Verschleiß war kaum festzustellen. --Pölkkyposkisolisti 13:27, 19. Jan. 2016 (CET)
Auf Gas umgerüstete Benziner starten mit Benzin in der Kaltlaufphase. Gas hat ein separates Motorsteuergerät, dass auf das Einspritzkennfeld des Motors eingestellt wird. Das vorhandene Motorsteuergerät im Auto unterstützt Gas idR nicht. Wird das Gas zu mager eingestellt, kommt es zu Motorschäden. Das Motoröl sollte eine Nummer besser sein, da das Gas heißer verbrennt. Es sind aber weniger Rußpartikel drin. Einige Umrüster sind mit dem Einbau überfordert. Teils werden die Leitungen nicht richtig verlegt, vereinzelt nicht sicher durch die Karosse geführt, das ist der in der Yellow-Press skandalierte Pfusch, wenn es heißt „Auto abgefackelt“. Wird das alles richtig gemacht und geprüft, ist Gas eine feine Sache. Mit der Elektrik haben es einige Umrüster auch nicht so, da die Kabel nicht Rüttelfest und Entlastet verlegt werden. Gasumrüstsätze sind meist Universal oder für eine Reihe von Fahrzeugen. Der Musterumbau ist meistens nicht beschrieben, daher häufig improvisiert werden. Hätte der Gesetzgeber die Autoindustrie verpflichtet, auch elektrisch standardisierte Schnittstellen und Vorbereitungen zur Nachrüstung mit vorzusehen, wäre Gas viel verbreiteter und akzeptierter. In Bulgarien ist Gas verbreiteter als Benzin. --Hans Haase (有问题吗) 14:16, 19. Jan. 2016 (CET)
Mein Bruder hat sich gezielt ein bestimmtes älteres Mercedes Modell gebraucht gekauft und selbst mit Hilfe eines befreundeten Bastlers auf Gas umgerüstet. Läuft seit Jahren zu seiner vollsten Zufriedenheit. Dabei fährt er eher Langstrecken bis mehrere 100 KM als Kurzstrecken. Entscheidend sind also wohl Erfahrungswerte mit bestimmten Modellen und etwas eigenes Know How schadet nicht. --92.202.94.65 15:28, 19. Jan. 2016 (CET)
Richtig, ungeachtet des Modells ist Gas eine feine Sache, wenn sorgfältig und richtig gearbeitet wurde. Nur ist unser Begriff von Ausbildung und Qualität nicht zuletzt durch Medien, Vertriebsinteressen und Wirtschaftspolitik schwer verzerrt. --Hans Haase (有问题吗) 16:24, 19. Jan. 2016 (CET)
Archivierung dieses Abschnittes wurde gewünscht von: --Kreuzschnabel 09:32, 21. Jan. 2016 (CET)

Kann mich nicht abmelden

Woran liegt dass, dass ich mich nicht abmelden kann sondern weiterhin angemeldet bin obwohl ich mehrfach auf "abmelden" klicke? --PL DK NL D (Diskussion) 21:11, 21. Jan. 2016 (CET)

Die Zeile bleibt seit einigen Tagen. Der Benutzername wird aber durch „Nicht angemeldet“ ersetzt. War es das? --Hans Haase (有问题吗) 21:28, 21. Jan. 2016 (CET)
Jo frage beantwortet. --PL DK NL D (Diskussion) 21:31, 21. Jan. 2016 (CET)
Archivierung dieses Abschnittes wurde gewünscht von: Rôtkæppchen₆₈ 01:20, 22. Jan. 2016 (CET)

Genannter Baurat des dem k.k. Eisenbahnministerium in Wien unterstellten Lokalbahnamtes (auch von Chabert-Ostland) hat in Südtirol entlang der Vinschgaubahn mehr als ein Dutzend Bahnhöfe projektiert. Seltsamerweise kann man online nicht viel mehr über den Mann ausfindig machen. Wenn man auf Google Books nach Baurat Chabert sucht, erfährt man immerhin, dass 1910 der von ihm erbetenen Versetzung in den dauernden Ruhestand unter Anerkennung seiner langjährigen, sehr zufriedenstellenden Dienstleistung entsprochen wurde. Aber schon das Geburtsjahr bleibt ein Rätsel... Entdeckt irgendjemand irgendwo Angaben, aus denen man eine kleine Biographie zusammenzimmern könnte? --Mai-Sachme (Diskussion) 13:40, 18. Jan. 2016 (CET)

Infoschnipsel: Schaust Anno. --Aalfons (Diskussion) 13:57, 18. Jan. 2016 (CET)
Danke für den Hinweis, leider zu wenig... Immerhin weiß ich jetzt, dass Ihrer Majestät beliebte, Herrn Baurat Ritter von Chabert anno 1900 das Ritterkreuz des Franz-Joseph-Ordens zu verleihen, ihn 1906 zum Oberbaurat zu erheben und 1910 anlässlich seines Ruhestands mit dem Titel eines Hofrates zu beglücken. Urlaub machte der verdiente Diener Ihrer Majestät a.D. in Bad Ischl, wo er im Juni 1912 im Hotel Kaiserkrone verweilte, im August desselben Jahres hingegen in der Villa Schwalbe. --Mai-Sachme (Diskussion) 17:56, 18. Jan. 2016 (CET)
1873 wurde er zum Lieutenant ernannt, 1876 war er schon Ingenieur, und 1915 machte er immer noch Urlaub in Bad Ischl. Aber für eine Kurzbiographie reicht auch das nicht. --Niki.L (Diskussion) 20:04, 18. Jan. 2016 (CET)
Dutzende Treffer gibt's auch in Tiroler Zeitungen. --Niki.L (Diskussion) 06:45, 19. Jan. 2016 (CET)
@Niki.L: Danke dir! Auch offline nichts auszumachen... Wird ja immer spannender. --Mai-Sachme (Diskussion) 11:45, 19. Jan. 2016 (CET)
Die n.ö. Oberrealschule St. Pölten meldet ihn im Jahresbericht 1871 als Abiturienten. Diese Schule hatte nur drei Oberrealschulklassen, für die er jeweils auch genannt ist; der Abschluss der Oberrealschule berechtigte zum Besuch einer Technischen Hochschule. KvC war ein guter Schüler und ist nie sitzengeblieben. Damit dürfte er um 1853 geboren sein. Er hat sich dann offenbar als Einjährig-Freiwilliger beim Tross verpflichtet, da lugt schon der Verkehrsbauingenieur hervor, nur wieso geht aus der Quelle von 1873 hervor, dass er da Leutnant war, @Niki.L:? Hab's nicht gefunden. Jedenfalls müsste er bei der Pensionierung 1910 erst etwa 57 Jahre alt gewesen sein. --Aalfons (Diskussion) 15:05, 19. Jan. 2016 (CET)
Die oben schon verlinkte Seite beginnt mit dem Satz: "Seine Majestät der Kaiser und König hat ernannt zu Lieutenanten in". Ich verstehe das so, dass dieser Satz vor jeden der Absätze auf dieser Seite zu stellen ist, also zunächst "zu Lieutenanten in der Reserve", dann "in der Jägertruppe", "in der Cavallerie" usw., und schließlich der Absatz, in dem Chabert erwähnt wird: (zu Lieutenanten) "im Militär-Fahrwesencorps". --Niki.L (Diskussion) 16:39, 19. Jan. 2016 (CET) Über eine Genealogie-Datenbank hab ich gefunden, dass im Jahr 1853 in Wien, Sankt Stephan, ein Konstantin Alfred Leo Armand von Chabert getauft wurde. Ich hab aber nicht die Berechtigung, dort mehr Daten zu sehen. --Niki.L (Diskussion) 17:41, 19. Jan. 2016 (CET)
Ah ja, Leutnant ist jetzt einleuchtend, mich hatten die dort genannten Unteroffiziere verwirrt, aber wer weiß, wie die Beförderungswege waren. Abiturienten+Militärdienst=Leutnant ist ja nicht ungewöhnlich. – Also haben sich die Anzeichen für 1853 verdichtet, das ist doch ein Fortschritt. --Aalfons (Diskussion) 20:07, 19. Jan. 2016 (CET)
@Niki.L:, @Aalfons: Hast du einen Link auf die Genealogie-Datenbank? Jedenfalls ist das hunderprozentig unser von Chabert. Den (ursprünglich französischen?) Namen gab's wohl mehrmals im Kaiserreich (hab ihn auch in Böhmen gefunden), aber bei dem Vornamen im gegebenen Zeitfenster ist das sicher kein blöder Zufall. In einem kakanisches Speziallexikon, das sämtliche Adelserhebungen mit genauem Datum verzeichnet, hab ich nur einen einzigen Namensträger gefunden, der in den 1840ern oder 1850ern (genaues Datum hab ich mir jetzt natürlich nicht gemerkt...) geadelt wurde. Dürfte dann wohl der Vater unseres Ritters gewesen sein.
Jedenfalls eine wirklich erstaunliche Forschungslücke bis jetzt. Der gute Mann hat ja immerhin eine ausgewachsene Bahnlinie geplant und mehr als ein Dutzend Bauwerke entworfen, die da immer noch in der Gegend rumstehen, darunter einen ausgewachsenen Jugendstil-Bahnhof in einer noblen Kurstadt. Hab auch eine Dissertation zum Meraner Jugendstil einschließlich ausgewählter Architektenbiographien eingesehen, aber von Chabert wird da (wie überall) mit bloßer Namensnennung abgespeist. --Mai-Sachme (Diskussion) 11:45, 20. Jan. 2016 (CET)
Vorsicht bei den Verwandschaftsverhältnissen. Beim Rumstöbern zeigte sich erstens, dass es mehrere Zweige der Chaberts gibt und Chabert-Ostland (in zahlreichen Schreibweisen) nur eine ist. Aber diese Chabert-Ostlands sind selbst im 19. Jahrhundert mit vielen Vertretern belegt, darunter am Hof des Sultans, dein Konstantin treibt sich aber nicht bei den bekanntesten herum, habe ich auf MyHeritage.de oberflächlich geprüft. Die Adelung im Jahr 1840 bezog sich übrigens nicht auf "das Ritter von Ostland", sondern das "von" vor Chabert, also aufpassen. Konstantins Personalakte, sofern erhalten, wäre im ö. Staatsarchiv zu finden, online ist der Name aber nicht zu finden. Vielleicht sind die entsprechenden Findmittel noch nicht digitalisiert (Personalakten dieser Zeit sind bereits Massenakten, die vllt nur über Karteien oder Abgabelisten erschlossen sind), aber eine Mail mit der Frage, *ob* eine Personalakte vorhanden ist, lohnt sich bestimmt. --Aalfons (Diskussion) 12:53, 20. Jan. 2016 (CET)
@Aalfons: Doch doch, bei der Adelung ging's ganz explizit um den Ritterstand, siehe eins drunter. --Mai-Sachme (Diskussion) 13:04, 20. Jan. 2016 (CET)
Kleine österreichische Chabert-Sammlung:
  • August Chabert, kaiserlich-königlicher Professor der Rechte, 1818 in Linz geboren und 1849 in Wien gestorben, ist wohl kein naher Verwandter. Auf Google Books kann man das Bruchstück einer Staats- und Rechtsgeschichte der deutsch-österreichischen Länder einsehen: Das Vorwort enthält eine ausführliche Biographie des Verstorbenen und berichtet von kleinbürgerlicher Herkunft und von einer kurzen Ehe mit zwei Töchtern.
  • Thomas von Chabert: Orientalist, 1766–1841, (siehe knappe Biographie hier mit falschem Sterbejahr 1856). Die Zeitschrift für österreichische Rechtsgelehrsamkeit und politische Gesetzkunde berichtet 1840, der k. f. Rath und Niederösterr. Landrechts-Dolmetsch, wurde in den Ritterstand des österr. Kaiserstaates mit dem Prädicate »von Oftland" erhoben. Der Orientalist ist also eindeutig der Ursprung sämtlicher Ritter von Chabert mit dem Zusatz Ostland. Da bereits 1841 gestorben, doch eher Großvater als Vater von Konstantin.
--Mai-Sachme (Diskussion) 12:56, 20. Jan. 2016 (CET)
ich hab dir per Mail geantwortet zu genealogischen Daten. Was du davon hier veröffentlichst, überlasse ich dir, weil es zum Teil nur Mutmaßungen sind. lg, --Niki.L (Diskussion) 13:25, 20. Jan. 2016 (CET)
(BK) Es gibt da z.B. den Hof- und Ministerial-Kopisten Karl Ritter Chabert-Ostland in einem Staatskalender von 1854, Wilhem Ritter von Chabert-Ostland, ö. Konsul in Smyrna (+1846), vielleicht der Sohn des Orientalisten Thomas, Antoine de Chabert-Ostland, um 1905 Jurist an der Universität Lille. Okay, alles nach 1840. Aber die Genealogie: Thomas hatte zwei Söhne Karl und Wilhelm und der Sohn von einem von den beiden ist Konstantin, und Antoine gehört zur dritten Generation, geht mir irgendwie zu glatt. Schön wär's natürlich. --Aalfons (Diskussion) 13:34, 20. Jan. 2016 (CET)
Konzipist, nicht Kopist ;-) --Niki.L (Diskussion) 14:03, 20. Jan. 2016 (CET)
Ich kenn mich ja nicht so wirklich mit Adelsprädikaten und so weiter aus, aber: Angenommen, dass spezifische vererbliche Adelstitel nur ein einziges Mal vergeben wurden (scheint mir plausibel, dass Ihre Majestät nicht noch weitere Ritter von Ostland kreiert hat und selbige Kreierung wäre auch nirgendwo verzeichnet...), dann kann Konstantin gezwungenermaßen nur ein direkter Nachkomme (zweiter oder dritter Generation) des Orientalisten gewesen sein. Aber im Grunde ist's ja auch alles Spielerei. Sekundärliteratur scheint nicht zu existieren, damit ist die Anlage eines Artikels leider nicht möglich. Danke nochmals für euer Bemühen! --Mai-Sachme (Diskussion) 14:08, 20. Jan. 2016 (CET)

Wozu sitzt ein Blech hinter einer 3,5"/5,25" Slot-Abdeckung beim PC?

Habe neulich alte P4 Rechner bzw. Cases auseinandergenommen. Die haben ja vorne idR 2 Slots für Floppy und 2-3 für CD-LW etc. Zum Teil waren die mit einfachem Plastik verschlossen, zum Teil aber auch mit sowas. Wozu ist das gut? --92.202.75.74 21:03, 18. Jan. 2016 (CET)

Einfach gesagt: Es ist eine Abschirmung. Hier ein offenes PC-Gehäuse. In den beiden Mittelstegen und rechts sind federnde Blechelemente zu sehen, die einen guten Kontakt zum Blechgehäuse herstellen. Gruss --Nightflyer (Diskussion) 21:08, 18. Jan. 2016 (CET)
Der Grund dafür sind die zwischenzeitlich gestiegenenen EMC-Anforderungen, bedingt durch die Richtlinien 89/336/EWG, 2004/108/EG und Richtlinie 2014/30/EU über die elektromagnetische Verträglichkeit. --Rôtkæppchen₆₈ 21:28, 18. Jan. 2016 (CET)
Ich halte das für ziemlich alt und heute nicht mehr gebräuchlich. P4 kommt hin, ich hatte mal ein doppeltes Blechgehäuse für einen damaligen Server, das wog über 10 Kg. Leider hab ich die Slotbleche nicht mehr, die waren praktisch: Durch die Federbleche klemmten die selbstständig fest und man konnte die Schraube sparen :-) Gruss --Nightflyer (Diskussion) 21:42, 18. Jan. 2016 (CET)
Mein allererster PC hatte sowas noch nicht, da waren es Plastikabdeckungen. Später gab es dann bei den einfacheren Gehäusen vorgestanzte Abdeckungen zum Rausbrechen. Teurere Gehäuse hatten oben abgebildetes oder auch hinter der Plastikblende eine separate Blechblende. Mittlerweile gehen aber einige Hersteller wieder dazu über, auf Blecheinsätze in den Laufwerksschächten zu verzichten. Bei den Steckplätzen für Erweiterungskarten gibt es auch rausbrechbare Abdeckungen, aber auch welche zum Einschrauben und solche aus Federstahl, die einfach nur in das Loch geklemmt werden. Früher gab es auf der Rückseite des PC-Gehäuses Löcher für zusätzliche Schnittstellen, also seriell, parallel, Gameport, SCSI und ähnliches. Manchmal waren das nur vorgeprägte Löcher mit ausbrechbarer Abdeckung, manchmal waren da auch schraubbare Abdeckungen. --Rôtkæppchen₆₈ 22:32, 18. Jan. 2016 (CET)
Ging das dabei nicht um die Luftführung? Jedenfalls bei sinnvoll designten Gehäusen saugte der Netzteilventilator die warme Luft aus dem Innenraum. Dabei strömte Frischluft über die CPU nach. Wenn man ein Slot-Blech einfach wegließ, wurde im ungünstigsten Fall die CPU zu heiß und von hinten kam auch mehr Staub rein.--Optimum (Diskussion) 23:07, 18. Jan. 2016 (CET)
Das war bei den ersten PCs vor Richtlinie 89/336/EWG der Grund für die hinteren Slotabdeckungen und neben der Optik auch der Grund für die Plastikdeckel vorn. Bei neueren PCs können die hinteren Slotabdeckungen auch aus Lochblech sein, erstens zur Gewichtsersparnis, zweitens, um die Luftzirkulation rund um die Steckkarten zu erhöhen. Heutige PCs haben meist einen zusätzlichen Gehäuselüfter, den es früher nicht gab. In der 8088- bis 80386SX-Ära gab es sogar passiv gekühlte CPUs. --Rôtkæppchen₆₈ 23:50, 18. Jan. 2016 (CET)
Richtig, der Netzteillüfter reicht heute nur mehr bei Low-End-PC alleine für die Gehäusebelüftung aus. High-End-PCs haben sogar mehrere große Gehäuselüfter. --MrBurns (Diskussion) 12:05, 19. Jan. 2016 (CET) PS: jedoch haben viele Fertig-PCs zur Abschirmung metallische Gitter vor den Lüftern (eventuell zusätzlich zu Staubfiltern, wobei letztere nicht immer vorhanden sind und wenn dann meist nur bei Lüftern, die die Luft von außen reinblasen), bei Gehäusen zum Selberbauen können schon solche Gitter bei den Lüftermontageplätzen sein, wenn dort nur ein Loch ist, muss man das Gitter zusätzlich kaufen wenn man will, dass der PC den EMC-Anforderungen entspricht. --MrBurns (Diskussion) 12:14, 19. Jan. 2016 (CET)
Das ist der Unterschied zwischen zugekauften und Gehäusen und den Fertig-PCs der großen Hersteller. Die Fertig-PCs halten die Vorschriften ein. Den Händler und Bastler juckt es nicht, was der PC von sich gibt. Nur wenn hinterher in der Presse ist, die NSA hat Rechner, die nicht am Netz hängen durch ihre Abstrahlung ausspioniert, macht alles große Augen. Übrigens hat sich damit die deutsche mittelständische Computerindustrie weitgehend von Behörden gestützt selbst zerlegt und vom Markt bereinigt. Es ist nicht das einige Merkmal an Gehäuse, Netzteilen und sonstigen Komponenten. --Hans Haase (有问题吗) 14:24, 19. Jan. 2016 (CET)
Die überwachbare Abstrahlung entsteht aber so viel ich weiß hauptsächlich am Monitor und am Kabel dort hin. Siehe Van-Eck-Phreaking (funktioniert auch bei LCDs, nur dann halt nicht mehr mit 100$-Equipement, aber 2000$ reichen da auch). Da nutzt dann auch ein besten abgeschirmte Gehäuse nicht. Tastatur und Maus kann man dank des USB-Kabels auch dann überwachen, wenn sie kabelgebunden sind. Soviel ich weiß gibt es auch keine Systeme, wo diese Verbindungen alle abgeschirmt sind, die richtig "paranoiden" bzw. auch Regierungsbehörden verwenden wohl abgeschirmte Räume. --MrBurns (Diskussion) 14:30, 19. Jan. 2016 (CET)
Das ist Sache der Schirmung von USB-Kabeln. Bei einer Maus darf das Kabel nicht zu schwer oder starr sein. Bei Cordless Desktops ist die Sicherheitslücke schon mit drin, außer, es gibt mehrere Gigabyte Schlüsselvorrat. Das Problem ist aber oft der Taktgenerator und der Takt des USB wird auch von diesem gemacht. --Hans Haase (有问题吗) 16:29, 20. Jan. 2016 (CET)
Und dazu hat man dann noch das Monitorbild, daher die NSA kann auch ganz genau wissen, was auf einem Offline-Computer passiert, wenn sie wollen und nah genug ran kommen (ich glaub ca. 100m reichen), außer der Computer steht in einem abgeschirmten Raum... --MrBurns (Diskussion) 19:17, 20. Jan. 2016 (CET)

Onlinebanking und Wertstellung

Ich nehme am Onlinebanking teil und wollte mal fragen: Die Kontobewegungen werden ja immer nur einmal pro Bankarbeitstag aktualisiert. Das heißt, ich sehe alles bis zu 23:59 Stunden später (bei Nicht-Bankarbeitstagen sogar noch größerer Verzug). Angenommen heute um 12:00 Uhr werden 1.200 EUR auf mein Konto gutgebucht und ich gehe zum Geldautomat und will mir von diesem Guthaben (im Online-Kontoauszug nicht ersichtlich) etwas abheben: Geht das? Ich weiß ja gar nicht, ob die Buchungen bankintern oder via Online-Kontoauszug übereinstimmen. Vermutlich stimmen diese Daten immer nur kurz nach dem Update des Online-Kontoauszuges überein. Bin bei der Netbank. --93.134.184.198 13:12, 20. Jan. 2016 (CET)

Keine direkte Antwort, aber vielleicht wird das Online-Banking bei verschiedenen Geldinstituten unterschiedlich gehandhabt. Bei mir (Sparkasse) werden die Kontobewegungen durchaus mehrmals täglich aktualisiert. Wenn mir beispielsweise um 6.00 Uhr ein Betrag gutgeschrieben wird und ich um 12.00 Uhr einen Betrag abhebe, kann ich beides um 13.00 Uhr sehen. --Euroklaus (Diskussion) 13:40, 20. Jan. 2016 (CET)
Meine Bank aktualisiert auch zeitnah; ich weiß nicht, ob ein Zeitraster vorliegt, aber eintreffende Überweisungen sind noch am selben Tag sichtbar. Wirf doch mal einen Blick auf die Valuta-Zeile: ab diesem Datum kannst du über das Geld verfügen. Wenn eine heute eintreffende Buchung erst morgen in der Übersicht erscheint, muß als Valuta-Datum der 20.1. drinstehen, d.h. du kannst heute schon darüber verfügen. --Kreuzschnabel 14:30, 20. Jan. 2016 (CET)
Gegenfrage: Woher willst du wissen, dass um 12 Uhr 1200 Euro auf dein Konto gutgebucht wurden, wenn du sie in Onlinebanking nicht siehst? Wenn im Onlinebanking noch keine Buchung angezeigt wird, gehe ich davon aus, dass auch noch nichts gebucht wurde. Ich habe übrigens ebenfalls ein Netbank-Girokonto, und nach meiner Erfahrung werden die im Onlinebanking angezeigten Buchungen mehrmals täglich aktualisiert. --91.221.58.20 15:12, 20. Jan. 2016 (CET)

Helligkeit halb sichtbarer Mond

Da ich die Antwort in den Artikeln Mond, Vollmond und Mondphase nicht gefunden habe, versuche ich es einmal hier, vielleicht kann jemand die Antwort gleich in einem der Artikel einbauen:
Bei Vollmond ist die Helligkeit gegen Mitternacht (bei höchstem Mondstand) laut Mondphase#Lichtwirkung des Mondes 0,25 Lux; meine Frage ist nun: Wenn man eine Viertel-Mondphase (rund eine Woche) vorher oder nachher die Hälfte der Mondfläche sieht, der Mond um 18 bzw. 6 Uhr maximal hoch steht, und es Winter ist, so dass man zu dieser Zeit kein Sonnenlicht sieht: Wie hell ist es dann? Logischerweise nicht einfach halb so hell, sondern deutlich dunkler, weil uns ja nur das Licht erreicht, das um die Ecke reflektiert wird, aber wie viel ist das?
Vielen Dank, Gruß, Aspiriniks (Diskussion) 20:15, 21. Jan. 2016 (CET)

Kann man nicht davon ausgehen, dass die Oberfläche in alle Richtungen etwa gleich stark streut (in 1. Näherung)? Dann müsste tatsächlich bei Halbmond auch die halbe Helligkeit gegeben sein. --Blutgretchen (Diskussion) 20:22, 21. Jan. 2016 (CET)
Nein, siehe den 2. der 4 aufgezählten Punkte bei Mondphase#Lichtwirkung des Mondes. Gruß, Aspiriniks (Diskussion) 20:39, 21. Jan. 2016 (CET)
OK...überzeugt. Aber googeln nach "Helligkeit Halbmond" hilft weiter.... z. B. liefert es das hier. Der Halbmond hat etwa 1/9 der Helligkeit des Vollmonds. Gruß. --Blutgretchen (Diskussion) 20:56, 21. Jan. 2016 (CET)
Prima, das ist doch eine interessante Frage und gute Quelle, ich hab's eingebaut. --Neitram  11:39, 22. Jan. 2016 (CET)
Vielen Dank! Gruß, Aspiriniks (Diskussion) 17:07, 22. Jan. 2016 (CET)
Dank zurück für die Frage. Hab auch wieder dazugelernt. --Blutgretchen (Diskussion) 17:57, 22. Jan. 2016 (CET)
Archivierung dieses Abschnittes wurde gewünscht von: Aspiriniks (Diskussion) 17:07, 22. Jan. 2016 (CET)

Vorgehen bei Auffinden von Schundprodukten

Den Wutanfall habe ich schon ein paar Tage hinter mir: Bei einer großen Discounterkette gab/gibt es aktuell so eine Art abgespeckte Wetterstation, bestehend aus einem Indoor-Gerät, das als Funkuhrwecker mit Temperaturanzeige fungiert, und einem Außentemperaturfühler, der seine Meßwerte per Funk an das Innengerät überträgt. Lt. Betriebsanleitung liegt der Meßbereich des Außensensors zwischen -20° C und +60° C, also brauchbar. Bei den vorherrschenden Temperaturen um den Gefrierpunkt herum stellte ich allerdings fest, daß kein Sensorsignal am Empfänger ankam. Auch das erklärte die Betriebsanleitung damit, daß unter 0° C mit Störungen der Übertragung zu rechnen sei, maW, das Gerät funktioniert bei Kälte nicht. Für mich ist sowas Betrug, ich habe mich bei der Hotline beschwert.

Das ist nun nicht so dramatisch, weil man Produkte "wegen Nichtgefallen" oder aus einem beliebigen Grund zurückgeben kann und der Kaufpreis erstattet wird. Ich finde aber, daß solche Fehlleistungen öffentlich gemacht werden sollten, auch mit dem Ziel, auf solche Anbieter einen Druck auszuüben, derartigen Schrott vom Markt zu nehmen, zurückzurufen und künftig nicht mehr anzubieten.

Daher die Frage: Wem sollte man solche Sumpfblütenfunde mitteilen? (nicht signierter Beitrag von 92.224.153.86 (Diskussion) 17:58, 18. Jan. 2016 (CET))

reclabox? --Heimschützenzentrum (?) 18:01, 18. Jan. 2016 (CET)
zuückgeben und gut isses. --Heletz (Diskussion) 18:39, 18. Jan. 2016 (CET)
Isses nicht.
Stiftung Warentest schreibt ab und zu über sowas, ansonsten Verbraucherschutz-Verbände. --Eike (Diskussion) 19:29, 18. Jan. 2016 (CET)

Als Privatkunde hast du bei jedem Gerät das Recht dieses zurückzugeben.

Wie kommst Du denn auf die Schnapsidee?

Bevor du irgend einen Verkäufer öffentlich denunzierst solltest du aber erstmal mit Ihm sprechen und sehen wie er sich verhält. Händler verlassen sich auf Ihre Zulieferer und prüfen nicht jedes einzelne Produkt das Sie verkaufen.

Wer, "Sie"? Also, ich nicht.

Können Sie gar nicht, das wäre viel zu Zeit und kostenintensiv. Der Händler wurde eventuell von seinem Zulieferer übers Ohr gehauen und mit der Kritik am Händler fügst du Ihm zusätzlich Schaden zu.

Also, ER ist allmächtig und wird durch mich Sterblichen sicher nicht geschädigt. Ach ja: Den Täter zu schädigen ist das Prinzip jeder Bestrafung, also gewollt. Der Sinn der Übung ist, daß er das nicht wieder tut und sich z. B. dagegen absichert, von Zulieferern übers Ohr gehauen zu werden.

Sich beim Händler zu beschweren reicht normalerweise, wenn genug Beschwerden eingehen nimmt der das Produkt aus dem Programm oder trennt der sich gar von seinem Zulieferer. Und dann hast du mit deiner Kritik auch den richtigen erwischt und nicht dem Händler der einfach nur Ware eingekauft hat. --2003:76:4E0D:F637:C173:719:5E84:79B9 19:37, 18. Jan. 2016 (CET)

Und wovon träumst Du nachts?

Auch hier wieder die Frage, was Du Dir erwartest. Irgendwie wirkt es hier manchmal so, als mancher sich einen überbordenden Kontroll- und Ordnungsstaat wünscht, der alles bis ins Detail regelt.

Erstens: Ja, ich wünsche mir in der Tat einen Kontroll- und Ordnungsstaat, der solchen Schurken auf die Finger haut, und finde daran nichts "überbordend". Wir brauchten ganz dringend einen obligatorischen "Produkt-TÜV", also eine offizielle Zulassungsstelle, bei der jeder Inverkehrbringer jegliches Serienprodukt untersuchen zu lassen hat, ob es sich nicht um Schund handelt, und zwar auch auf Funktion und Haltbarkeit und nicht nur auf Sicherheitsfragen bezogen. Ich möchte für mein sauer verdientes Geld keinen Müll angedreht kriegen. Und zweitens: Von "Staat" war gar nicht die Rede, die Frage war, wo man solche Funde melden kann. Ich dachte dabei an Organisationen oder Medien, die sowas publizieren.

Frage ist ganz einfach, was wurde versprochen. Dabei kann man sich auf die Verpackung und Werbung beziehen. Was Du jedoch machst ist eine Schlussfolgerung. Der Meßbereich hat nicht automatisch etwas mit der gesicherten Übertragung zu tun. Dieses Problem ist so weit verbreitet, das mancher fast von Allgemeinwissen ausgehen kann.

Was schreibst du nur für einen Stuß. Auf der Packung stand "Temperaturstation", sonst nix, die Betriebsanleitung ist inliegend, man kriegt sie erst zu sehen, wenn man das Produkt gekauft und ausgepackt hat. Und es ist nicht Allgemeinwissen, daß man bei Produkten übers Ohr gehauen wird: Die vernünftige Annahme ist vielmehr, daß ein Lufttemperaturmeßgerät für die Region, in der es als Consumer product verkauft wird, auch vollumfänglich geeignet ist und nicht bei üblichen Einsatzbedingungen ausfällt - naja, also auf Deinem Planeten vielleicht nicht, aber in unserer Welt schon.

Denn das liegt teilweise an der fehlenden Leistung der Batterien, aber auch Beeinträchtigung der Kontakte durch Wasser und Eis. Der Preis ist da meist schon ein Indikator für den Aufwand bei der Verarbeitung, und wer beim Discounter zum Discountpreis kauft, bekommt ein Discountprodukt. Das hat weder was mit Betrug, Schrott oder Schund zu tun.

Ich bin wirklich froh, nicht in Deiner Welt leben zu müssen, in der es offensichtlich weder Moral noch Anstand gibt.

Egal was Du vom Beruf bist, auch in Deinem Bereich wird sowas bestimmt erstellt, was nicht den vollen Erwartungen der Kunden entspricht. Oliver S.Y. (Diskussion) 19:44, 18. Jan. 2016 (CET)

Der Messbereich des Temperatursensors und die Betriebstemperatur der verbauten Batterie – denn daran wird es hängen – sind zwei paar Stiefel. Es ist leicht, Halbleitertemperaturfühler mit großem Erfassungstemperaturbereich zu bauen, die bei entsprechender Stromversorgung auch prima funktionieren. Es ist aber sehr schwer, Batterien zu bauen, die auch bei tiefen Temperaturen noch ausreichend Leistung erbringen. Wenn der Hersteller einen Messbereich spezifiziert, dann spezifiziert er einen Messbereich und keinen Betriebstemperaturbereich. Versorge das Gerät mit einer externen Spannungsquelle oder trenne Temperatursensor und Auswerteelektronik räumlich. Früher ging das auch wunderbar: Man hatte eine Wetterstation, an der an einem Kabel der Außenfühler hing. Den hat man zum Nordfenster rausgehängt und das Grundgerät innen auf den Tisch gestellt oder an die Wand gehängt. Mein Außenfühler hängt zum Ostfenster raus und zeigt deswegen im Sommer gelegentlich „Sonne, Mond und Sterne“ an, wie mein verstorbener Kollege, ein Mess-Steuer-und-Regelungstechnik-Entwicklungsingenieur, zu sagen pflegte. --Rôtkæppchen₆₈ 01:45, 19. Jan. 2016 (CET)
Es geht aber nicht um den Meßbereich des Sensors, sondern um den des Außenfühlers, also des kompletten Teilgeräts. Und es ist dabei das Problem des Herstellers, wie er die geeignete Stromversorgung hinkriegt. Was aber auch mit Batterien durchaus geht: Erstmal ist der Verbrauch dieser Dinger vernachlässigbar, so daß der bei Kälte zunehmende Innenwiderstand der Zellen kaum eine Rolle spielt - bei solchen Geräten "sterben" die Batterien nicht durch Entladung, sondern an Alterung - und zweitens sackt die Spannung nicht bei irgendeiner Temperatur (> 0 K) schlagartig auf null ab, sondern geht mit sinkender Temperatur ein bißchen zurück, aber eben nicht so sehr viel. Es wäre also durchaus möglich, die Schaltung auf dieses Verhalten der Batterie abzustimmen - z. B. dadurch, daß ein Spannungswandler sie entsprechend hochsetzt oder man die Schaltung gleich auf kleinere Betriebsspannungen auslegt, also z. B. nicht auf 2 x 1,2 V, sondern auf 2 x 1,0 V, dann klappt's auch in der Kälte. (Aber manchmal habe ich den Eindruck, daß in der Entwicklung wieder die Deppen oberhand nehmen, die in der Uni nichts gelernt haben, von den "Alkalines" nur die Nennspannung 1,5 V kennen und die Abschaltspannung dann auf 1,3 V oder so festlegen mit dem Erfolg, daß die Digi-Knipse mit NiMH-Zellen nicht besonders gut läuft (Beispiel: Pentax Optio E10 - das Mistding behauptet nach relativ kurzer Betriebszeit immer, daß die Batterien leer sind, was nicht wahr ist).
Und außerdem: Wen interessieren denn technische Probleme im Detail? Der Kunde erwartet, daß das Produkt im üblichen Anwendungsfall funktioniert, und das sind eben "gängige" Temperaturen zwischen z. B. -20° C und +60° C. Wenn der Entwickler das nicht kann, warum auch immer, dann gehört das Produkt nicht in den Verkauf. Innen in die Betriebsanleitung "Ätschi-bätschi, funktioniert aber genau dann nicht, wenn du's brauchst" reinzuschreiben ist dafür keine akzeptable Lösung. (nicht signierter Beitrag von 92.224.243.219 (Diskussion) 07:17, 19. Jan. 2016 (CET))
reclabox? hier hilft das alles gar nix... natürlich ist es irreführend, wenn der Messbereich nicht zu den Betriebsbedingungen passt... --Heimschützenzentrum (?) 08:25, 19. Jan. 2016 (CET)
Kennt Reclabox denn außer Dir noch jemand? Vielleicht habe ich nicht richtig geguckt, aber ich hatte eher nicht den Eindruck, daß die Seite bei der Kaufentscheidung sonderlich hilfreich wäre. (nicht signierter Beitrag von 92.224.73.30 (Diskussion) 17:45, 19. Jan. 2016 (CET))
vor allem nervt die reclabox direkt bei dem Störer rum, ob er sich nich doch mal kümmern möchte... manchmal wird es denen dann zu blöd und die werden vernünftig... --Heimschützenzentrum (?) 10:06, 21. Jan. 2016 (CET)
Ein anderer Punkt ist: Für die richtigen Batterien ist immer der Kunde verantwortlich, selbst, wenn die Batterien mit dem Gerät mitgeliefert werden. Oft steht das auch in Bedienungsanleitung und Gewährleistungsbedingungen drin. Hast Du das Gerät mit den mitgelieferten Batterien in Betrieb genommen oder hast Du frisch gekaufte Batterien eingesetzt? Das kann schon einen Unterschied machen. --Rôtkæppchen₆₈ 08:32, 19. Jan. 2016 (CET)
Steht in der Betriebsanleitung "Funktioniert mit den mitgelieferten Batterien nicht, mit den 'richtigen' aber schon" oder nicht vielmehr "funktioniert unter 0° C nicht"? Die Batterien sind in Ordnung (überprüft!), und wenn nicht, dann wäre das ebenfalls ein Sachmangel. (nicht signierter Beitrag von 92.224.73.30 (Diskussion) 17:45, 19. Jan. 2016 (CET))
Das kannst Du gerne versuchen. Meistens liegen die Geräte vor dem Verkauf so lange, dass die mitgelieferten Batterien nicht mehr frisch sind. Es wird auch in der Produktdokumentation darauf hingewiesen, dass die mitgelieferten Batterien nur für Demonstrationszwecke sind und im übrigen als Verschleißteile von der Gewährleistung ausgeschlossen sind. --Rôtkæppchen₆₈ 22:16, 19. Jan. 2016 (CET)

Beratungsresistenter Schnäppchenkäufer der sauer ist das er beim Discounter ein Discounterprodukt bekommen hat. Als Onlinehändler hat man Gott sei Dank die Möglichkeit solche Kundenaccounts einfach zu sperren. --87.140.192.0 11:24, 19. Jan. 2016 (CET)

Sorry, aber ich kann den Ärger des Posters im Prinzip verstehen. Grundbedingung für ein verkehrsfähiges Handelsgut (auch ein Discounter-Produkt!) ist, dass es für den bestimmungsgemäßen Gebrauch geeignet ist. Bei einem Temperaturmessgerät mit Außenfühler heißt das konkret, dass es innerhalb des hierzulande üblichen Temperaturspektrums funktionieren muss. Sonst ist es tatsächlich Schrott. Im vorliegenden Fall wäre allerdings noch zu prüfen, ob das Ding bei Kälte wirklich nicht läuft oder ob es sich bei dem Passus in der Bedienungsanleitung nur um eine der üblichen salvatorischen Klauseln handelt, die heutzutage zuhauf in die Bedienungsanleitungen geschrieben werden, damit der Hersteller bei einem allfälligen Fehler nur ja nicht haftbar gemacht werden kann. --Jossi (Diskussion) 12:01, 19. Jan. 2016 (CET)
Und er kann es ja zurückgeben und das Geld bekommen. Oder umtauschen, möglicherweise funktioniert ja nur das eine Gerät nicht. Kommt schon mal bei Discounter-Ware vor, da wird nicht jedes genauestens kontrolliert. Solange man nicht ausschließen kann, dass das Gerät prinzipiell nichts taugt, sollte man auch mit verallgemeinernder Kritik vorsichtig sein, das ist ggf. abmahnfähig. --Hachinger62 (Diskussion) 16:46, 19. Jan. 2016 (CET)
Ich kann nicht ausschließen, daß das Gerät prinzipiell nichts taugt. Im Gegenteil, das legt schon die Betriebsanleitung nahe. Die Erfahrung bestätigt es lediglich. (So blöd, sich mit juristischem Gehabe zum Affen zu machen, sind übrigens höchstens halbseidene eBay-Betrüger und ähnliche Möchtegern-Onlinehändler; größere Discounterketten wissen vielmehr, daß der Schuß regelmäßig nach hinten losgeht.) (nicht signierter Beitrag von 92.224.73.30 (Diskussion) 17:45, 19. Jan. 2016 (CET))

Reise zum Mittelpunkt der Erde

Angenommen, es wäre technisch möglich, einen stabilen Tunnel durch die komplette Erde zu bauen und jemanden in einen aerodynamisch günstigen Transporter darunter zu schicken. Man betrachte die folgenden Fragestellungen unter zwei Szenarien:
1.) In der Röhre herrscht normaler Luftdruck (wie verändert sich der auf dem Weg? Wie modelliert man den Einfluss?)
2.) In der Röhre wird ein Vakuum erzeugt.
Wenn man die Superrotation des Erdkerns außer acht lässt, auf welche Art und Weise berechnet man (abhängig von den Koordinaten) den benötigten Verlauf der Röhre, damit der Passagier nicht sofort aufgrund der Erdrotation gegen die Wand knallt?
Wie berechnet man die Gravitationskraft? Innerhalb der Erde ist der Fall mit 2 Punkten mit bestimmter Masse ja nicht mehr gegeben - da ist die Masse schön um einen rum verteilt (mehrdimensionale Integration???).
Wie verwertet man die so gewonnene Formel bei der Berechnung der "Durchfluggeschwindigkeit"? Eine gleichförmig beschleunigte Bewegung ist das dann ja nicht mehr.
Könnte ein Mensch eine derartige Beschleunigung überleben? (Ich vermute ja, bei Raketenstarts wirkt ja auch eine mehrfache Erdbeschleunigung)
Angenommen, das Transportgerät hat Hitzeschilder wie heutige Raumflugkörper. Wie heiß würden diese (ungefähr)?
Wie erlebt der Astronaut (wenn er nicht gerade mit der Beschleunigung zu kämpfen hätte) Schallereignisse (an der Wand, im Transporter, der ja auch beschleunigt,...)

Ich weiß, dass diese Fragen alle höchst irrelevant sind und sowas nie möglich sein wird. Trotzdem fand ich das Thema spannend, auch wenn meine Physik-Lektionen schon ein paar Jahre zurückliegen...

Besten Dank, --217.237.164.210 17:57, 19. Jan. 2016 (CET)

PS: Weitere spannende Fragestellungen sind herzlich willkommen
PPS: Ähnlich, aber einfacher: Gegeben sei eine Form (im Weiteren ein Kreis) in der euklidischen Ebene, die eine große Stadt darstellt. Wie kann man mathematish zeigen, dass der Punkt, der von allen Punkten im Kreis den geringsten Durchschnittsabstand zu allen anderen Punkten hat, der Mittelpunkt ist (um z. B. dort einen Brunnen zu bauen - ich weiß, dass Menschen im echten Leben diskret sind :-)? Wie verallgemeinert man das auf mehrere Dimensionen und andere geometrische Objekte?

Weitere Fragestellung: Wie heißt das Zeugs, das du genommen hast?--2003:75:AF16:8F00:DD6E:3B1C:E965:6D0F 18:16, 19. Jan. 2016 (CET)
In einer Hohlkugel besteht kein Schwerefeld. Du kannst nach teilweisem Eintauchen dir die Erde aufgeteilt denken in eine Hohlkugel außerhalb deines Punktes und eine Rest-Erde innerhalb deines Punktes. Die erfahrene Schwerkraft wird dann nur noch von der Resterde ausgeübt, da die Hohlkugel, wie gesagt, keine Gravitation im Inneren ausübt. Unter der vereinfachenden Annahme einer homogenen Massenverteilung innerhalb der Erde ist die gravitativ wirksame Restmasse = ((Erdradius − Eintauchtiefe) / Erdradius )^3 * Erdmasse. --Kreuzschnabel 18:49, 19. Jan. 2016 (CET)
Mir ist nicht klar, was ein Mensch unter dieser Röhre sollte! --Heletz (Diskussion) 19:01, 19. Jan. 2016 (CET)
Gib mir mal bitte eine Erklärung dafür, warum an einem beliebigen Punkt innerhalb einer Hohlkugel diese nun plötzlich gravitativ weggdacht werden könnte. -- Janka (Diskussion) 22:28, 19. Jan. 2016 (CET)
Newtonsches Theorem. --Wrongfilter ... 22:38, 19. Jan. 2016 (CET)
Ah fein, danke. Ich hab mir die nötige Geometrie im Kopf umhergeworfen und überlegt, wie man das beweisen könnte. -- Janka (Diskussion) 02:57, 20. Jan. 2016 (CET)
Wir haben auch Schwerefeld#Erdschwerefeld im Erdinneren. -- HilberTraum (d, m) 08:06, 20. Jan. 2016 (CET)
Wir haben vor allem den Artikel Gravitationstunnel. --132.230.195.184 10:41, 20. Jan. 2016 (CET)
…, der genau null Mal im Artikelnamensraum verlinkt ist. Wie soll man den finden können? -- HilberTraum (d, m) 10:58, 20. Jan. 2016 (CET)
Jetzt 1x verlinkt. --Neitram  13:13, 20. Jan. 2016 (CET)

Erinnert ihr euch an den Benutzer, der hier vor Jahren seine physikalischen Theorien zur Debatte stellte, weil er sich beim Verfassen von Science-Fiction-Romanen nicht blamieren wollte? Dies zur Gegenfrage von Benutzer:2003... --Aalfons (Diskussion) 13:02, 20. Jan. 2016 (CET)

Zum Szenario 1) solltest du wissen, dass der Luftdruck in so einem Loch derart mit der Tiefe zunimmt, dass die Lufttemperatur etwa pro 1 Kilometer Tiefe um 10°C ansteigt. Nach ein paar hundert km ist sie ein en:supercritical fluid. Ansonsten noch der Tipp, bohr den Tunnel vom Nordpol zum Südpol, dann kannst du die Rotation der Erde vernachlässigen. Und weiterhin empfehle ich eines der zig Foren zu benutzen, in denen genau deine Fragen seit langem immer wieder mit Begeisterung diskutiert werden (sie sind ja auch spannend) -- du findest sie mit Google. --Neitram  13:13, 20. Jan. 2016 (CET)
Zum PPS: ich mag mich irren, aber ich glaube, der Punkt ist der geometrische Schwerpunkt. --Neitram  13:16, 20. Jan. 2016 (CET)
Ich irre mich wohl tatsächlich, denn für Dreiecke ist es der Fermat-Punkt, und der ist nicht der geometrische Schwerpunkt. Im Artikel en:Geometric median steht im Abschnitt "Computation" mehr dazu. --Neitram  13:33, 20. Jan. 2016 (CET)
Ich glaube, dass es dem Fragesteller um den Mittelwert der Abstände über alle Punkte der Kreisfläche geht. Der Fermatpunkt beim Dreieck bezieht sich auf die Ecken. Der Schwerpunkt minimiert den Mittelwert über das Quadrat der Abstände. Der Mittelwert über die Abstände selbst ist mathematisch auf jeden Fall komplizierter. --Digamma (Diskussion) 15:59, 20. Jan. 2016 (CET)
Ah ja, danke. Zum Punkt "Könnte ein Mensch eine derartige Beschleunigung überleben" noch die Anmerkung, dass die Beschleunigung gar kein Problem ist. Im Szenario 1) hat das Gefährt nach ein paar Sekunden Beschleunigung mit 1 G seine Maximalgeschwindigkeit erreicht (wie beim Fallschirmsprung im freien Fall), dann fällt es mit etwa gleichbleibender Geschwindigkeit auf seinem Luftkissen, während es langsam von der zunehmenden Hitze gebraten und vom zunehmenden Luftdruck zerquetscht wird. Im Szenario 2) nimmt die Beschleunigung von anfangs 1 G bis auf 0 G im Erdmittelpunkt (ca. 21 Minuten später) ab (bzw. erreicht nach File:EarthGravityPREM.svg in der Hälfte der Strecke noch ein kleines Maximum leicht über 1 G, kein Problem), ist also für den "Astronauten" unkritisch. Auch die negative Beschleuigung für die zweite Hälfte der Fahrt ist gespiegelt gleich, also gleichermaßen unkritisch. --Neitram  16:46, 20. Jan. 2016 (CET)
Die gravitative Beschleunigung ist gespiegelt, das Luftkissen wirkt aber immer noch in dieselbe Richtung, daher wird das Fahrzeug wohl nicht weit über den Mittelpunkt hinausgehen, bis es zurückpendelt. --Kreuzschnabel 16:55, 20. Jan. 2016 (CET)
Im Szenario 1) gibt es keine zweite Hälfte der Fahrt. Das "Luftkissen" verwandelt sich in einen Stöpsel aus flüssiger-fester Luft, bevor das in dieser Hölle gebratene Gefährt auch nur annähernd den Erdmittelpunkt erreicht. Noch eine Korrektur: Im Szenario 2) ist der "Astronaut" die ganze Zeit über in Schwerelosigkeit, weil gleich schnell fällt wie sein Gefährt. Er spürt also die auf ihn wirkende Beschleunigung überhaupt nicht (die wie oben gezeigt auch nicht sonderlich groß ist). --Neitram  17:04, 20. Jan. 2016 (CET)
Woher soll eigentlich diese flüssige Luft kommen? OK, durch die Erdmasse herrscht im Inneren ein enormer Druck und die Kerntemperatur ist 6000°C, aber wenn die Superröhre diesem Druck und dieser Temperatur standhält, herrscht im Erdkern Schwerelosigkeit, und das gilt auch für die Luft.--Optimum (Diskussion) 17:36, 20. Jan. 2016 (CET)
Der reine Druck der Luftsäule auf sich selbst. Du weißt, wieviel dünner und kälter die Luft in 10 km Höhe als auf Meereshöhe ist? Genauso nimmt der Druck und die Temperatur in dieser Luftsäule mit jedem weiteren Kilometer in die Tiefe zu (unabhängig von Druck oder Temperatur der Erde, die du bei der Betrachtung der reinen Luftsäule auch ignorieren kannst). Grob (adiabatisch) überschlagen, "This yields a temperature of about 48000 kelvin at the center of the Earth and a pressure of about 56 million atmospheres". Also erst Gas, dann nach ein paar hundert km superüberkritisches Fluid, dann vielleicht sogar kristallin. --Neitram  20:08, 20. Jan. 2016 (CET)
Vielleicht übersetzt ja mal jemand den Artikel en:Shell theorem. Nach dem Sachverhalt wird schon seit Jahren immer wieder gefragt. --Pp.paul.4 (Diskussion) 00:15, 21. Jan. 2016 (CET)
Ja, es wäre sicher gut, das Thema etwas ausführlicher als unter Newtonsches Gravitationsgesetz#Ausgedehnte Körper, unter einem eigenen Artikel Newtonsches Schalentheorem zu behandeln. Zur Lektüre zum Thema verlinke ich noch The Internal Pressures of the Planets. --Neitram  11:16, 21. Jan. 2016 (CET)
Etwas verspätet: Dazu gibt es auch ein ganz unterhaltsames Videos: MinutePhysics: What if the Earth were Hollow? --StYxXx 04:26, 22. Jan. 2016 (CET)

... bei Minderjährigen in Deutschland.

Ich suche nach einem realen Beispiel mit "Minimalalter", in dem einem/einer Minderjährigen gestattet wurde, "selbstbestimment" (nach Kontrolle durch das Jugendamt) alleine (finanziert z.B. durch die Eltern) zu leben. Gibt es so etwas?
Zusatzfrage: Hat jemand ein Beispiel aus einem anderen europäischen Land ? (ausgeschlossen natürlich Situation wie in Brasilien oder Ländern Afrikas - also: Das ganze muss schon legal sein.)
Was sind "Bedingungen" dafür, oder wird das jeweils von Fall zu Fall entschieden? Play It Again, SPAM (Diskussion) 14:22, 20. Jan. 2016 (CET)
Hm in der Schweiz ist es möglich vor erreichen der Volljähigkeit alleine in einer Wohnung zu leben, der hat aber immer einen Beistand. Der Jugentliche kann dann zwar über „Angelegenheiten des täglichen Lebens“ selbständig bestimmen, nicht aber über die anderen die bei euch im Aufenthaltsbestimmungsrecht geregelt sind. Zum Teil schlicht aus dem Grund dass er er noch nicht voll geschäftsfähig ist. Dieser Enstscheid des selbständigen Lebens ist aber immer eine Enzelfallensscheideung, es gibt eben auch verschiedene Formen des betreutes Wohnen's. Von der Gastfamilie, über eien WG-Zimmer bis zur "Hartcore"-Version Einer eigene Wohnung. Aber eben auch bei der eigene Wohnung, kann es Unterschiede geben. Es macht acuh da ein Unterschied aus, ob ein Betreuer merhmals wöchentlich vorbeikommt, oder ob der Beistand nur noch vereinzelnt vorbei kommt (bzw. der Schützling bei ihm auf der Schelle steht, wenn er das Haushalsgeld abholt geht). Das wird durchaus Jugentlichen in der Ausbildung zugetraut, wo eben eine zusätliche indirekte Kontrolle via Schule/Lehrbetrieb stattfinden kann. Aber eben es ist immer Eine Einzelfallendscheidung und das Aufenthaltsbestimmungsrecht ist noch immer in Händen Dritter. Wie stark sie es noch ausüben, das steht eben auf einem anderen Blatt.--Bobo11 (Diskussion) 14:49, 20. Jan. 2016 (CET)
In Deutschland dürfte das vor allem an der Haftungsfrage scheitern. Wenn der Jugendliche irgendeinen Mist baut wird immer gleich geschaut ob man die Sorgeberechtigten wegen Aufsichtspflichtverletzung in Haftung nehmen kann. Das sind dann entweder die Eltern oder wenn denen das Sorgerecht entzogen wurde das Jugendamt als Amtsvormund. Das will natürlich niemand riskieren. -- Liliana 14:54, 20. Jan. 2016 (CET)
Richtig dieses Stimmchen ist natürlich immer im Hintergund. Aber eben das ist ganz klar nicht nur vom Alter sondern auch vom einzelnen Jugendlichen abhängig, wie weit entwickelt der ist. Wie "erwachesn" sprich Eigenverantwortlich dieser ist. Ich kenne etliche die mit Beginn der Lehre oder Besuch des Gymnasiums zumindest als Wochenaufenhalter von Zuhause ausgezogen sind. Schlicht und einfach aus dem Grund, dass sonst der tägliche Arbeitweg zu lange gewesen wäre. Das ging eben von Wohnen bei Verwanten, über Lehrlingsheim/Zimmer beim Lehrmeister, über WG bis zur eigenen Wohnung. Da war aber eben in der Regel auch ein Interesse da, keinen Ärger zu machen, weil nur so der eigene Traumberuf erlehrnt werden konnte. Und eben es gab immer einen Person (Meist die Elteren), die dafür Verantwortlich war was der Jugentliche während des allein seins anstellt. Und meist noch andere Personen (Lehrmeister, Verwandte) die sich zusätzlich dafür verantwortlich fühlten. Bei meinem Lehrbetreib gab klare Regeln für die Auswärts wohnenden (ganz alleine wohnen war dabei nicht drin), die der Lehrmeister dann auch überwachte. Die meisten wohnten dabei eh in der Lehrlings WG, und da wurde halt mal jemand um 22:00 vom Lehrmeister ans Telefon gerufen (an den Festnetz-Anschuss der WG versteht sich). --Bobo11 (Diskussion) 15:28, 20. Jan. 2016 (CET)
Ich bin mit 17 in eine eigne Wohnung gezogen. Meine Eltern hatten da auch nichts gegen, eher im Gegenteil--ich hatte ein eigenes Einkommen als ZeitungsboteMarkoz (Diskussion) 15:38, 20. Jan. 2016 (CET)
Geht es - praktisch - auch mit 16 Jahren? Play It Again, SPAM (Diskussion) 16:19, 20. Jan. 2016 (CET)
Ja, es geht auch schon mit 16. Es geht sogar schon mit 15. Genau genommen gibt es keine fest definierte Grenze. Das Aufenthaltbestimmungsrecht bleibt bis zur Volljährigkeit bei den Sorgeberechtigten, und auch die unbeschränkte Geschäftsfähigkeit tritt erst mit 18 ein. Wenn das Alleinewohnen das Kindeswohl gefährdet, könnte das Jugendamt einschreiten - aber das muss keineswegs so sein. Eine frühreife 15jährige kommt womöglich besser zurecht als manche 19jährigen.
Es trifft auch nicht zu, dass immer die Eltern Partei des Mietvertrags sein müssen - nur ohne ihre Zustimmung geht's eben nicht. --Snevern 17:09, 20. Jan. 2016 (CET)
Also eher Fall zu Fall?
Das wollte ich wissen. Play It Again, SPAM (Diskussion) 17:20, 20. Jan. 2016 (CET)
Richtig es ist immer eine Einzefallendscheidung, die natürlich auch vom Jugendamt mitgetragen werden können muss (Bei uns in de Schweiz wäre es die Kinder- und Erwachsenenschutzbehörde). --Bobo11 (Diskussion) 17:30, 20. Jan. 2016 (CET)
Rein praktisch muss man in Deutschland nicht das Jugendamt im Vorfeld um Erlaubnis bitten oder das Jugendamt auch nur informieren - man tut es einfach, so wie man vieles andere im Rahmen der elterlichen Sorge tun kann, ohne irgendjemanden davon zu unterrichten oder um Zustimmung zu bitten. Die Verantwortung und die Entscheidungsbefugnis liegt bei den Eltern (Erziehungsberechtigten). Wenn alles gut geht und es zu keinen Auffälligkeiten kommt, kann so etwas jahrelang gut gehen. Das Kind wird dann irgendwann volljährig, ohne dass das Jugendamt überhaupt davon erfahren hat. --Snevern 17:37, 20. Jan. 2016 (CET)
Bin Anfang der 90er Jahre, wenige Monate vor meinem 16. Geburtstag, also noch mit 15, in eine kleine Wohnung umgezogen, die meine Eltern für mich angemietet haben. Der Vermieter war zufrieden über die pünktliche Miete von meinen Eltern, die Nachbarn waren zufrieden, einen ruhigen Nachbarn zu haben... Also hatten alle ein Interesse daran, es so zu belassen und die paar Jährchen bis zur Volljährigkeit ablaufen zu lassen. Unabhängig von der Rechtslage gilt ja sowieso immer der Grundsatz: Wo kein Kläger, da kein Richter und kein Jugendamt ;-) --84.119.204.210 20:37, 20. Jan. 2016 (CET)
Wenn das in Deutschland war, dann war das völlig im Einklang mit der Rechtslage. --Snevern 21:18, 20. Jan. 2016 (CET)
Snevern, was verstehst du denn konkret unter 15-jähriger Frühreifer? --Bundesparteienforscher (Diskussion) 08:31, 21. Jan. 2016 (CET)
Einen individuellen Einzelfall, der beim Umzug aus der elterlichen in eine eigene Wohnung weder Probleme bekommt noch welche macht. Und du? --Snevern 08:40, 21. Jan. 2016 (CET)

E-Book Format

Wäre es nicht ein leichtes, die in Papierformat kürzlich erschienene kritische Mein Kampf Edition auch als epub Dokument bereitzustellen ? --89.14.6.178 15:03, 20. Jan. 2016 (CET)

Gewiss. --195.36.120.126 16:25, 20. Jan. 2016 (CET)
Wenn wir mal annehmen, daß es nicht aus historischen Gründen per Bleisatz produziert wurde, dann ist das technisch wohl kein Problem. Das ist übrigens keine kritische, sondern eine kommentierte Ausgabe, soweit ich weiß, oder wurde da am Text gegenüber anderen Ausgaben auch was geändert? --Kreuzschnabel 16:47, 20. Jan. 2016 (CET)
Dann wäre es auch leichter die Kommentare heraus zu löschen und ein unverfälschtes braune Soßepamphlet digital zu haben. Wer den Blödsinn mag.--2003:75:AF11:2E00:D5A1:17BE:5C09:BB1A 16:53, 20. Jan. 2016 (CET)
Hier. Play It Again, SPAM (Diskussion) 17:10, 20. Jan. 2016 (CET)
Wer das schafft, kriegt es auch hin, "mein kampf epub" zu googeln. --FGodard||± 19:56, 20. Jan. 2016 (CET)
Wir sollten sehr vorsichtig sein und nicht alle Geschäftsgeheimnisse der Auskunft ausplaudern! Wenn das jeder Hansel da draussen weiss - und es dann AUCH tut! - wären wir nämlich hier ganz flott arbeitslos - Willst du das wirklich ?!? Play It Again, SPAM (Diskussion) 10:30, 21. Jan. 2016 (CET)
technisch wäre das ohne weiteres machbar. Die Herausgeber legen aber ausgesprochenen Wert darauf, dass die Anmerkungen permanent in Sichtweite des Textes sind. Das geht in Epub nicht oder nur, indem man die Anmerkungen irgendwie ausgezeichnet in den Text integriert. Im ersten Fall ist der Zugang zu den Anmerkungen fummelig (jedenfalls auf dezidierten Readern), im zweiten Fall wird der Originaltext zerrissen. Beides nicht so toll. Das Konzept mit Marginalien, die den Text »umzingeln« ließe sich allenfalls für größere Tablets umsetzen, in Epub 3 oder schlicht als PDF. Rainer Z ... 21:22, 20. Jan. 2016 (CET)
die kürzlich erschienene kritische kommentierte Mein Kampf Edition erschien jetzt, weil "zufällig" in diesem Januar die Urheberechte für die Orignalversion verfallen sind. Die neue "kommentierte Ausgabe" hat auf Grund der Kommentare (die ja nicht von einem bereits 70 Jahre verstorbenen Verfasser stammen) ein neues Urheberechtsdatum erzeugt. Die neue Ausgabe geht also nicht nach unseren Bestimmungen, nur eben die Orginalversion, weil "adolf" schon 70 Jahre tod, und alle Rechtsnachfolger (erben,rechtsnachfolger und so, in diesem Falle warum auch immer, wohl der Staat Bayern, obwohl "adolf" ja wohl eher östereicher war) nun keine weiteren Rechte an dem Machwerk haben. Und wenn wir das dann in Wp aufnehmen ist es doch ein recht großes Vorhaben für "wikisource" (ich denke die haben dafür zur Zeit zu wenig Personal, um das im bewährten Umpfang aufzunehmen) um es dann hier in wikipedia als quelle zu nutzen.--Jmv (Diskussion) 23:04, 20. Jan. 2016 (CET)
Zur Frage, warum das Land Bayern Inhaber des Urheberrechts war, kenne ich zwei Versionen: Nach der ersten wurde das Recht dem Land durch einen Beschluss der US-Militärregierung übertragen. Nach der anderen fiel das (auch immaterielle) Vermögen im Zuge der Entnazifizierung an das Land, in dem der Betreffende seinen Wohnsitz hatte. Und Hitler war nicht in Berlin, sondern in München gemeldet. --Snevern 23:24, 20. Jan. 2016 (CET)
Zumindest hat er in München Steuern gezahlt bzw. nicht gezahlt. --2003:45:4655:A600:F077:BE72:ABD7:73AC 01:50, 21. Jan. 2016 (CET)
Welche Frage wird eigentlich diskutiert bzw. beantwortet? Die des TO, ob die kommentierte Ausgabe als eBook publiziert werden könnte? Antwort ist: Ja, könnte, ist technisch kein Problem - warum das die Autoren/Herausgeber, die das Urheberrecht wegen der Kommentierung haben, nicht tun, war nicht gefragt. Oder war die Frage, ob es MK (ohne Kommentierung und urheberrechtsfrei) als eBook gibt? Gibt es auch, für Peanuts. Und die Frage, ob der Text im Internet aufzufinden ist, ist auch leicht positiv zu beantworten. Schwierig zu finden ist er nicht: Einfach aus einem Original ein paar ungewöhnlich Ausdrücke/Formulierungen raussuchen und danach googeln, schon findet man passende Treffer. Ich persönlich finde die Neuherausgabe etwas unglücklich: Vermutlich wird sie benutzt, um Schüler damit zu quälen. Der (originale) Text ist nämlich erstens stilistisch ziemlich ungenießbar (weswegen es plausibel ist, daß das Buch kaum gelesen wurde) und zudem von heute aus ohne relativ gründliche historische Kenntnisse auch kaum verständlich: Während Hitler ganz selbstverständlich auf Ereignisse im 1. WK Bezug nimmt, die damals, wenige Jahre nach Kriegsende, natürlich jedermann bekannt waren, wissen die Durchschnittsbürger von heute davon gewöhnlich nichts und können den Zusammenhang deswegen nicht verstehen. (Den Wert der Kommentare in der kommentierten Ausgabe kann ich nicht beurteilen, weil ich sie nicht kenne und auch nicht die Absicht habe, mich damit zu befassen. Und MK zu lesen halte ich für Zeitverschwendung, wenn man sich damit nicht gerade als Fachwissenschaftler befassen will - das nicht aus Unkenntnis, sondern gerade weil ich MK früher mal teilweise gelesen hatte und in etwa weiß, was drin steht.) (nicht signierter Beitrag von 92.224.155.95 (Diskussion) 03:33, 21. Jan. 2016 (CET))
Die Gefahr, dass Schüler damit gequält werden, halte ich für eher gering. Die Stundenzahl für den Geschichtsunterricht ist insgesamt so niedrig, dass im normalen Unterricht schlicht keine Zeit ist, um auch noch in nennenswertem Umfang MK zu lesen (egal, was kenntnisfreie Politiker so daherschwadronieren). Eher dürfte es die Studenten treffen. ;-) --Jossi (Diskussion) 13:49, 21. Jan. 2016 (CET)

Schermaschinenkämme

Wie schleift man Schermascinenkämme?


--2003:49:CD20:AC44:15BA:DDCD:B494:5E57 20:18, 20. Jan. 2016 (CET)

Google Wie schleift man Schermaschinenkämme liefert bei mir als ersten Treffer das hier. --Rôtkæppchen₆₈ 22:33, 20. Jan. 2016 (CET)
Schermaschinenkämme gibts noch nicht. Sollten wir da was haben? --Jmv (Diskussion) 22:41, 20. Jan. 2016 (CET)
Nach Wikipedia:Namenskonventionen#Singularregel wäre das Lemma Schermaschinenkamm. --Rôtkæppchen₆₈ 22:47, 20. Jan. 2016 (CET)
gibts aber auch noch nicht ;-) --Jmv (Diskussion) 00:02, 21. Jan. 2016 (CET)
Wir haben aber die Lemmata Schermaschine, Haarschneidemaschine und Schermaschine (Textil), die leider nicht auf konstruktive Eigenheiten eingehen, aber gemäß wp:WWNI auch nicht auf Instandhaltung oder Instandsetzung. --Rôtkæppchen₆₈ 00:20, 21. Jan. 2016 (CET)
Wenn es und den Kopfrasenmäher geht: Normal garnicht, bestimmt gibt es Dienstliester oder Ersatzteile dafür. Sie halten länger, wenn die gereinigt, richtig eingestellt und geölt werden. Sonst dürften sie von oben zum gegenüberliegenden Kamm hin geschliffen werden. Das geht nur ausgebaut. --Hans Haase (有问题吗) 12:40, 21. Jan. 2016 (CET)
Hier wird detailliert dargestellt, wie das funktioniert. --Rôtkæppchen₆₈ 13:06, 21. Jan. 2016 (CET)

Thermische Auslese bei Vitaminen?

Hallo! Kann mir einer der Chemiker, Physiker der Biologen hier erklären, warum nur ein Teil des Vitamins C in Lebensmitteln durch Erhitzen (Kochen, Backen) verlorengeht? Bei Fleisch sind mir die unterschiedlichen Wärmezonen im Gargut schon klar, aber wenn ich Zitronensaft mit Tee aufbrühe, wirkt die Temperatur ja auf jedes einzelne Atom/Molekül. Sind damit Abbauprodukte der Ascorbinsäure gemeint, welche genauso wie diese wirken?Oliver S.Y. (Diskussion) 22:31, 19. Jan. 2016 (CET)

Das Vitamin geht nicht sofort kaputt, sondern der Gehalt nimmt in Abhängigkeit von Temperatur und Druck exponentiell mit der Zeit ab. Das heißt, je länger Du erhitzt oder je höher Du erhitzt, desto weniger Vitamin C bleibt übrig. --Rôtkæppchen₆₈ 22:45, 19. Jan. 2016 (CET)
(BK) Hier wird es an Broccoli erklärt. Geringere Temperatur "verändert" Vitamin C (zu dehydroascorbic acid [DHAA]), bei "schnell höherer Temperatur" wird es erhalten. Die Hypothese ist, dass bei niedriger (warmer) Temp. erst noch Enzyme aktiv sind, die Vit. C. umsetzen. Werden die Enzyme aber sofort "kaputtgeheizt", ist Vit. C länger stabil.
Sie waren selber von dem Resultat überrascht. DAS ist Wissenschaft! Play It Again, SPAM (Diskussion) 22:48, 19. Jan. 2016 (CET)
L-Ascorbinsäure-Oxidase ist das verantwortliche Enzym, aber wir haben noch keinen dt. Artikel. Play It Again, SPAM (Diskussion) 23:03, 19. Jan. 2016 (CET)
(BK)Hatte die Diskussion vor einigen Wochen mit meinem Bruder (beide ziemlich erkältet), er meinte heiße Zitrone u.ä. habe keine Wirkung, ich dagegen, Ascorbinsäure (vulgo Vitamin C) gehe wenn erst durch Kochen (für mich Temperatur = Siedepunkt = 100 Grad Celsius). Wenn man Zitronensaft in einem bereits gebrühten Tee kippt, ist die Temperatur deutlich unter 100 Grad C, gleiches gilt auch für die Heiße Zitrone (Wasser aufbrühen, erst dann Zitronensaft + evtl Süßungsmittel)--in dubio Zweifel? 23:07, 19. Jan. 2016 (CET)
Ascorbinsäure zersetzt sich aber erst bei 190 Grad. 100 oder 95 Grad ist da völlig unerheblich. Mir gings darum, das es ja keine "Masse" ist, die sich abbaut, sondern ein gelöster Stoff, auf den die Temperatur direkt wirkt. Das mit dem DHAA ist aber wirklich hilfreich, genau wie die Forschung, danke. Fühle mich bei meinem Tee mit Zitrone eigentlich sehr gut, und von Oma die Anweisung, die Zitrone aufzubrühen, nicht als Geschmack dazugeben. So unterschiedlich sind die Legenden, und gleiches Ergebnis, es hilft :) Oliver S.Y. (Diskussion) 23:22, 19. Jan. 2016 (CET)
Plazeboeffekt Yotwen (Diskussion) 08:32, 20. Jan. 2016 (CET)
(BK)Dehydroascorbinsäure hat aber eben auch Vitaminwirkung, siehe Abschnitte Ascorbinsäure#Dehydroascorbinsäure und Ascorbinsäure#Recycling der Oxidationsprodukte. Die im Abschnitt Ascorbinsäure#Vorkommen postulierten „erheblichen Vitamin-C-Verluste[]“ sind also nur temporärer Natur, bis der Organismus die Dehydroascorbinsäure wieder zu Ascorbinsäure reduziert hat. Dennoch werden Dehydroascorbinsäure und Ascorbinsäure mit der Zeit abgebaut und ausgeschieden. --Rôtkæppchen₆₈ 08:46, 20. Jan. 2016 (CET)

Das wäre mal eine schöne Literatur-Fingerübung:

  • Stabilität von Ascorbinsäure in wässriger Lösung
    • bei unterschiedlichem pH und konstanter Temperatur (einfach zu finden)
    • bei unterschiedlicher Temperatur und konstantem pH
    • und die Hyperfläche Temperatur/pH
  • Analyse von ascorbinabbauenden Enzymen in Tee (und deren Thermostabilität)
    • Andere Verbindungen in Tee, die mit Ascorbinsäure reagieren (... und Zusatz von Rum, welcher %-Satz ist optimal?)
  • Stabilität von Ascorbinsäure in frischem Zitronensaft
  • etc. etc.
Der Heilige Gral der Studie wäre dann ein Tee, der supergesund macht, Krebs bekämpft und - vielleicht - Unsterblichkeit verleiht. Wer weiss? Play It Again, SPAM (Diskussion) 11:07, 20. Jan. 2016 (CET)
Davon ab, ob es Erkältungen überhaupt gibt (also solche, die durch Kälte hervorgerufen werden), wirkt da Vitamin C nicht sowieso nur prophylaktisch, wenn überhaupt? Sobald man erst "erkältet" ist, ist doch schon alles zu spät, oder? Geoz (Diskussion) 16:21, 20. Jan. 2016 (CET)
Vitamin C ist eines der ersten Produkte, das ohne Marketing völlig unbekannt geblieben wäre, wie man z.B. hier sehr schön lesen kann. Man konnte Vitamin C industriell herstellen, aber für die paar Fälle, die von Skorbut bedroht waren, lohnte sich die Produktion nicht. Also pries man es erstmal als leistungssteigerndes Mittel an. - Der individuelle Bedarf ist von Person zu Person unterschiedlich, deshalb sind Mengenempfehlungen und die in Ascorbinsäure beschriebene "Unterversorgung" auch größtenteils Käse. --Optimum (Diskussion) 18:57, 20. Jan. 2016 (CET)
@Geoz: Ich habe den Verdacht, dass du weisst, dass die Frage "Was ist der kürzeste und was der weiteste Abstand des Mars vob der Erde" leichter zu beantworten ist, als deine Frage.
WP windet sich genial und schreibt (das ist kein Witz):
"Der Stellenwert von Vitamin-C-Gaben zur Bekämpfung und Vorbeugung von Krankheiten wie dem grippalen Infekt ist wissenschaftlich allerdings umstritten, wobei größere Reviews einen generellen Trend sehen, dass während Vitamin C zwar keinen messbaren prophylaktischen Effekt bei saisonalem grippalen Infekt hat, allerdings ein moderater positiver Effekt auf den Krankheitsverlauf beobachtet wurde. Dieser konnte in therapeutischen Studien allerdings nicht reproduziert werden. Es gibt auch Meta-Analysen, die zeigen, dass Nahrungsergänzungsmittel mit Vitamin C bei grippalen Infekten weder prophylaktisch helfen noch die Genesung beschleunigen können." D'OOOH !
Wir sind also hier im sehr individuellen Bereich, wo sich Personen nach einem warmen Tee mit Zitronensaft (und Rum) besser fühlen als ... D'OOOH! Das P-Wort steht ja auch schon oben. Play It Again, SPAM (Diskussion) 19:29, 20. Jan. 2016 (CET)
Ich vermute mal, genau wie bei anderen Dingen kommt es auf die Umstände an. Wer auf seinen Vitamin C Haushalt achtet, achtet auch auf andere Bereiche seines Lebens intensiver, und verringert damit seine Anfälligkeit gegenüber bestimmten Krankheiten. Ansonsten sehe ich eher den Beweis erbracht, daß ein Vitamin-C Mangel zu Krankheiten führt. Ob die ausreichende Versorgung allein Krankheiten verhindert ist wohl wirklich PR, wobei es relativ unsinnig ist, da Sauergemüse und Äpfel immer vorhanden waren, und eher die Mangel von fettlöslichen/tierischen Vitaminen wie B oder D ein Problem waren. Aber hier gings ja darum, ob nochwas im Tee drin ist, das wirkt.Oliver S.Y. (Diskussion) 23:09, 20. Jan. 2016 (CET)
Bonusrunde mit Selbstversuch: Zitrone ist nicht die Frucht mit dem höchsten Vit. C Gehalt.
Wäre jemand von den Teetrinkern hier gewillt, mal Tee mit frisch gepresstem Papayasaft oder Kiwisaft zu probieren? (Rote Paprika wäre ideal, aber für den Anfang...)
Ich vermute, dass das Angenehme/Vertraute von "Tee mit Zitrone" eher durch die Zitronensäure hervorgerufen wird. Play It Again, SPAM (Diskussion) 09:28, 21. Jan. 2016 (CET)
Vielleicht wirkt die Fruchtsäure schleimlösend/adstringierend/desinfizierend? Ich kann mir Tee mit Petersiliensaft nicht so richtig vorstellen, obwohl Petersilie anscheinend fast dreimal soviel Vitamin C hat wie Zitrone. --Optimum (Diskussion) 20:39, 21. Jan. 2016 (CET)
Ich vermute eine kombinierte Wirkung der Protonen der Citronensäure und der Reduktionswirkung der Ascorbinsäure auf die Polyphenole und Tannine des Tees. --Rôtkæppchen₆₈ 01:19, 22. Jan. 2016 (CET)

Fachfrage zum Scannen

Ich habe eine Broschüre vorliegen deren Rückenhöhe ca. 20 cm und deren Breite knapp 21 cm ist, also fast quadratisch. Welche Formateinstellung müßte ich auf dem Scanner wählen, der bis A3 scannt, damit ich das Heft doppelseitig mit möglich wenig Verlust scannen kann? Bitte sachbezogene Antworten. Danke.scif (Diskussion) 09:57, 21. Jan. 2016 (CET)

Was meinst du mit "Verlust"? A3 ist doch größer als die Broschüre doppelseitig; es sollte also keine Verluste geben? Überhang kannst du vermeiden, indem du die passende Größe händisch im Scanprogrammn einstellst, das sollte eigentlich jedes Scanprogramm können, denke ich. Man ist da nicht auf vorgegebene Größen beschränkt. --Eike (Diskussion) 10:06, 21. Jan. 2016 (CET)
Ja meine damit den Überhang, klappt jetzt auch, habe die Maße händisch eingegeben. Problem ist jetzt nur, das der Drucker das Papier für zu kurz hält und den Scan nicht komplett ausdruckt, grrrrr.scif (Diskussion) 10:08, 21. Jan. 2016 (CET)
Also, der A3-Drucker druckt dann auf A4? Da solltest du A3 einstellen können. Man oft kann auch einstellen, dass ein Bild beim Ausdruck vergrößert wird, wenn du magst. --Eike (Diskussion) 10:12, 21. Jan. 2016 (CET)

Nee, der Scanner schickts mir auf den Rechner, vom Rechner geht's an nen A4-Drucker. Selbst mit Anpassen usw geht da nix.scif (Diskussion) 10:55, 21. Jan. 2016 (CET)

In welchem Datenformat scannst Du? Probier mal PDF, da kannst Du durch geschicktes Drehen und Zurechtschneiden auch A3 auf zweimal A4 ausdrucken. Das müsste auch mit Deinem ca. 420×200-Scan gehen, was ja etwas kleiner als A3 quer (420×297 mm) ist. --Rôtkæppchen₆₈ 11:00, 21. Jan. 2016 (CET)
Wenn dir A4 als Gesamtgröße reicht, kannst du's auch automatisch verkleinern lassen. Ansonsten würd ich glaub ich eher die Seiten einzeln einscannen. --Eike (Diskussion) 11:05, 21. Jan. 2016 (CET)
Es scannt in PDF, bis zum Rechner ist alles OK ich hab mittlerweile durch benutzerdefinierte Größe nen sauberen Scan ohne Überhang. Problem ist der Drucker, der spinnt. Ich werde es zuhause probieren und berichten.scif (Diskussion) 11:08, 21. Jan. 2016 (CET)
Üblicherweise kann man in Druckertreiber oder PDF-Software Dokument- und Druckformat separat einstellen. Ich habe bei derartigen Problemen aber auch oft mehrere Versuche gebraucht, bis das Ergebnis zufriedenstellend war. --Rôtkæppchen₆₈ 11:26, 21. Jan. 2016 (CET)
Drucker haben technisch bedingt einen Rand auf dem Ausdruck. Bei Laserdruckern und Kopieren liegt dieser in Papierlaufrichtung. Der Grund ist das Abstreifen des Papiers von der Fixierwalze, da bei beim Fixieren der Toner schmilzt. Beim Tintenstrahler ist ein Rand unten, da das Papier vor dem Druckkopf geführt und transportiert wird. Danach muss die Tinte trocken und kann nicht durch eine Papiertransportwalze geschoben werden, da die sonst das Stempelkissen macht. Das Endlospapier mit seinem Traktortransport hatte das nicht. A4 ist 21 cm breit. Wenn du nachträglich zuschneidest kannst Du die Doppelseite auf A3 scannen und drucken oder machst von Anfang an A4, zentrisch ohne Großenanpassung. Einige Drucker können ohne oder minimalen Seitenrändern drucken. Wenn Du auf 97 mm breite Notizzettel Wert legen solltest, schaue Dir den Drucker an, ob der oben oder unten Rand benötigt. --Hans Haase (有问题吗) 12:34, 21. Jan. 2016 (CET)
Gell da glotscht, so sieht ein Fachbeitrag zum Scanner aus, trotzdem, der 3D-Drucker und Druck eines 80 mm Karnickels fehlt noch.--2003:75:AF10:700:A1BA:E2A1:F9D3:11BB 13:01, 21. Jan. 2016 (CET)

Bei den meisten Druckertreibern kann man die Option "Auf Papiergröße anpassen" anschalten. Dann wird die Vorlage automatisch auf das gewählte Papier skaliert, egal wie groß die Vorlage ist. Benutzerkennung: 43067 13:07, 21. Jan. 2016 (CET)

PDF-Betrachter haben das. Mir fällt keiner ein, der es nicht kann. --Hans Haase (有问题吗) 13:27, 21. Jan. 2016 (CET)
Was das Scannen angeht, so kann in der Datei Platz gespart werden, indem Gammakorrektur zu Filterung von Schatten und Papierfasern. Dazu wird der Eingangslevel Weiß/Hintergrund angehoben. Sind keine Fotos drin, kann die Farbzahl begrenz werden. Alles andere muss so versucht werden, das ist so nicht zu beantworten. --Hans Haase (有问题吗) 16:41, 21. Jan. 2016 (CET)
Darf ich ohne wissenschaftliche Abhandlung antworten? Zuhause gehts, Punkt. Wir reden nur von Laserdruckern und nur von Schwarz-Weiß. Der räusper Drucker hängt in einem Netzwerk und hat was weiß ich für Einstellungen, aus nachvollziehbaren Gründen werde ich da kaum die IT-Abteilung ransetzen. Das leistungsstarke schuhschrankgroße Scan-/Kopiergerät steht leider nicht zuhause, zwinker. Mehr muß ich sicher nicht erzählen. Sascha Grammels Prof. Peter Hacke würde sagen: Alles im Dienste der Wissenschaft (Ja es geht um Quellensicherung für WP-Artikel) Grüße--scif (Diskussion) 19:42, 21. Jan. 2016 (CET)
Digital macht es möglich. Nimm das PDF dazwischen. Bei den heutigen Windmühlen ist nur der Rotor am Mast, das Mahlwerk steht wo anders und das Stromnetz ist dazwischen. Dafür musst Du nicht auf Wind warten, es geht auch mit Sonne oder Atomdingsda. --Hans Haase (有问题吗) 21:42, 21. Jan. 2016 (CET)

Zufrieren von Seen aus ganz speziellen Flüssigkeiten

Aufgrund der Dichteanomalie von Wasser (max. Dichte bei ca. 4°C) gefriert bei Wetter, wie es momentan herrscht, ein See zuerst an der Oberfläche. Soweit klar. Aufgrund der Tatsache, dass das Wassereis ein größeres Volumen hat als das flüssige Wasser, wird eine geschlossene Eisdecke auf dem See im Prinzip immer dicker, je länger es draußen kalt bleibt. Auch klar. Jetzt die Frage: Wie würde ein trichterförmiger See aus einem Stoff gefrieren, der im flüssigen Zustand eine Dichteanomalie aufweist (ähnlich wie Wasser), aber beim Übergang in den festen Aggregatzustand sein Volumen verkleinert (anders als Wasser)? Das Gefrieren müsste ja eigentlich auch an der Oberfläche beginnen, die Eisschicht würde aber aufgrund ihres geringeren Volumens, das sie im Vergleich zum Wasser, aus der sie hervorgegangen ist, einnimmt, nicht kontinuierlich weiterwachsen können (da der See trichterförmig ist, würde die Eisschicht am Ufer aufliegen und kann nicht "nachrutschen"). Wie friert der See jetzt weiter? --Blutgretchen (Diskussion) 17:27, 21. Jan. 2016 (CET)

An welcher Stelle kühlst Du den See ab? Luft oder Boden? --Hans Haase (有问题吗) 17:31, 21. Jan. 2016 (CET)
Ja, wo übeall die Wärme abgeführt wird ist hier sehr wichtig. Der andere Punkt ist das Wie. Gibt es eien Luftstömmung oder ist es Windstill? usw. usw. : Generelles Problem ist wie entstehen in der Flüssigkeit die Kristallisationskeime, also wie unsauber ist die Flüssigkeit. Ohne zu Wissen wie die Kristallisation/das Erstarren angestossen wird, wird die Frage nicht zu beantworten sein. Das Verhalten danach ist dann eben von Dichteunterschide abhängig. Denn die Kristalle werden bis zu einer gewissen Grösse in der Flüssigkeit schweben, also werden weder sinken noch steigen. --Bobo11 (Diskussion) 17:44, 21. Jan. 2016 (CET)
(BK) Luft. --Blutgretchen (Diskussion) 17:45, 21. Jan. 2016 (CET)
Erstmal ein Zwischen-Danke @Bobo11. Ich hatte übersehen, dass mit der Abnahme des Volumens beim Gefrieren die Dichte der Eisschicht steigt. Die an der Oberfläche entstehenden Kristallisationskeime würden daher gar nicht auf der Oberfläche verbleiben, sondern absinken, so dass es zu Beginn des Gefrierens erst gar nicht zur Bildung einer geschlossenen Eisschicht an der Oberfläche kommen würde (wovon ich zunächst noch ausgegangen bin). Insgesamt bleibt es aber wohl ziemlich kompliziert. --Blutgretchen (Diskussion) 18:40, 21. Jan. 2016 (CET)
Jep, das „ziemlich kompliziert“ trifft den Nagel auf den Kopf. Es kommt dann eben auch auf die Stömmungen ( Konvektion ) in der Flüssigkeit an. Also wie der Wärmeausstausch in der Flüssigkeit geschieht. Das kann durcchaus zu "unlogischen" Resultaten führen. --Bobo11 (Diskussion) 18:46, 21. Jan. 2016 (CET)

Weil die Frage oben kam: Ich würde mal zur weiteren Konkretisierung davon ausgehen, dass kein Wind und keine von anderen äußeren Einflüssen herrührende Strömungen im Wasser herrschen (also alle Strömungen nur Ergebnis von Konvektion durch Dichteunterschiede und Gravitation sind). Außerdem sollen genügend Verunreinigungen (winzigste Schwebteilchen) in der Flüssigkeit enthalten sein, dass kein Gefrierverzug auftritt.--Blutgretchen (Diskussion) 18:44, 21. Jan. 2016 (CET)

In der Frage steckt schon ein (Denk-?)Fehler: Ein Stoff, der beim Festwerden dichter wird, weist keine Dichteanomalie auf, sondern das ist der Normalzustand. Du könntest bspw. Wachs beim Festwerden beobachten, da bildet sich so eine Delle aus. --Benutzer:Duckundwech 19:10, 21. Jan. 2016 (CET)
Bist Du da ganz sicher? Könnte nicht aus welchem Grund auch immer ein Stoff im flüssigen Zustand (irgendwo zwischen Schmelz und Siedepunkt) eine Anomalie dieser Art aufweisen:
Beispiel: Schmelzpunkt: 50°C; Siedepunkt 150°C; zwischen -273.13 °C und 60°C stetige Abnahme der Dichte; bei 60° ein Tiefpunkt (1. Ableitung =0); zwischen 60°C und 100°C stetige Zunahme der Dichte; bei 100°C ein Hochpunkt (1. Ableitung =0); dann wieder stetige Abnahme der Dichte oberhalb 100°C bis unendlich.
Aber auch wenn es keinen solche Stoffe geben sollte: Als Gedankenexperiment kann man das ja doch mal durchspielen...Gruß. --Blutgretchen (Diskussion) 20:03, 21. Jan. 2016 (CET)
Wenn 50° der Schmelzpunkt ist und die Dichte bis 60° abnimmt, würde gefrierende Flüssigkeit sofort nach unten sinken.--Optimum (Diskussion) 20:25, 21. Jan. 2016 (CET)
Soweit waren wir ja schon oben. Unten ist es aber wärmer, weil sich da die dichteren Flüssigkeitsschichten befinden (z. B. 100° am Grund). Wie aber friert der See schließlich zu? --Blutgretchen (Diskussion) 20:39, 21. Jan. 2016 (CET)
Ah, tricky. Angenommen, die gesamte Flüssigkeit ist anfangs 100° warm und kühlt von oben ab. Die kältere Flüssigkeit kann aber nicht sinken, weil es ja leichter als das darunterliegende ist, es gibt also keine Konvektion, höchstens ein wenig Wärmeleitung. Bis die obere dünne Schicht kälter als 60° wird und die 59°-Schicht endlich unter die etwas wärmere 60°-Schicht sinkt. Dann gibt es oberhalb der 100°-Flüssigkeit eine Konvektionswalze, die sicherlich immer etwas von der oberen 100°-Schicht mitreißt, solange bis die 100°-Schicht aufgezehrt ist. --Optimum (Diskussion) 21:25, 21. Jan. 2016 (CET)

Football-Kenner vor!

Folgende Situation: Der Quarterback wirft den Ball in die gegnerische Endzone. Dort wird er aber nicht von einem eigenen Mitspieler gefangen, sondern von einem Gegenspieler, der dann mit dem Ball aus dem Spielfeld läuft. Was ist das?

  • Ein Safety ist es nicht, weil dafür ja die Offense mit dem Ball die Endzone verlassen müsste.
  • Am nächsten kommt es noch an einen Touchback: Die gegnerische Mannschaft hat den Ball in ihrer eigenen Endzone erhalten, ihn aber nicht mehr ins Spielfeld hinausgetragen. Aber es ging kein Kickoff und kein Punt voraus und somit ist es auch kein Touchback.

Was ist es? --87.123.29.224 17:33, 21. Jan. 2016 (CET)

Ich denke schon, es ist ein Touchback. Warum der deutsche Artikel im Gegensatz zum englischen nur Kickoff oder Punt, nicht aber einen Fumble oder einen Pass gelten lässt, weiß ich nicht. Grüße Dumbox (Diskussion) 17:43, 21. Jan. 2016 (CET)
Ein Fumble ist das nicht, denn bei einem Fumble müsste ein Spieler den Ball verlieren. Das ist hier aber nicht der Fall; der Quarterback verliert ihn nicht, sondern gibt ihn willlentlich ab und der Passempfänger verliert ihn auch nicht, er bekommt ihn gar nicht erst.
Letztlich ist das eine Interception in der Endzone. Und danach endet der Spielzug durch Verlassen des Feldes. Der englische Artikel sagt in der Tat, das sei ein Touchback... --87.123.29.224 18:30, 21. Jan. 2016 (CET)
Mit "Pass" meinte ich natürlich einen intercepted pass, geht ja nicht anders. Grüße Dumbox (Diskussion) 18:36, 21. Jan. 2016 (CET)
Ich hab den deutschen Artikel mal angepasst. --87.123.29.224 18:40, 21. Jan. 2016 (CET)

Treiber gesucht

Moin, ich suche Treiber für Arcsoft Show biz 2. Google bietet mir einen toten Link an und sonst nix. Müssen für XP sein. Alternativen sind nicht möglich. Wer kann helfen? Danke und LG --Gwexter (Diskussion) 17:48, 21. Jan. 2016 (CET)

Bei mir benutzt Arcsoft ShowBiz die Treiber meiner TV-Karten, Webcams und Videoaufnahmeadapter. Bei mir läuft Arcsoft ShowBiz 3 unter Windows 10 x64. Es nutzt die BDA-Treiber der möglichen Quellen. Ob Arcsoft ShowBiz 2 ebenfalls BDA- oder aber VfW- oder WDM-Treiber benötigt, musst Du ausprobieren. Suche am besten auf der Website Deiner Videoquelle (Webcam, TV-Karte, Grabber) nach passenden Treibern für Dein Betriebssystem. --Rôtkæppchen₆₈ 18:00, 21. Jan. 2016 (CET)
Hi und thx, ich gehe deinen Anregungen mal nach. --Gwexter (Diskussion) 18:20, 21. Jan. 2016 (CET)

Google vs. Presse

Was ist der aktuelle Stand, weiss man schon, wie die Rechtslage eigentlich ausschaut? --85.4.233.141 19:08, 21. Jan. 2016 (CET)

Bedeutung ./.

Hallo, wofür steht ./. und wie kann man es verwenden und einsetzen, das heißt, wann und wo steht es!? Denke, dass es negativ bedeutet, aber ist es auch wie das Vorzeichen - zu verwenden und wenn ja, warum nicht gleich -12 usw. --Commons2Contributor2 (Diskussion) 21:43, 21. Jan. 2016 (CET)

Es heißt „abzüglich“, etwa in Rechnungen oder Kostenaufstellungen. --Kreuzschnabel 21:45, 21. Jan. 2016 (CET)
+1 Es wird auch als kaufmännisches Minuszeichen bezeichnet.--Blutgretchen (Diskussion) 21:58, 21. Jan. 2016 (CET)
Ich kenne es von Akten beim Anwalt: Mandant ./. Gegner. --22:30, 21. Jan. 2016 (CET)
+1. Gemäß DIN 5001:2011 7.4 bedeutet das Zeichen "gegen". --Snevern 23:21, 21. Jan. 2016 (CET)
<ironie>„Abzüglich“ wäre aber auch nicht ganz falsch, wenn man bedenkt, wie oft der Gegner den Mandanten nicht nur Geld sondern auch Lebenszeit und vor allem Lebenslust kostet... </ironie> --2003:45:4656:CD00:381A:DD8D:942E:F1CB 23:25, 21. Jan. 2016 (CET)

Technik vs. Technologie

Was ist der Unterschied zwischen Technik und Technologie?--Skyscraper1996 (Diskussion) 19:20, 22. Jan. 2016 (CET)

Technik ist wohl recht klar definiert, bei Technologie gibts unterschiedliche Definitionen, ich denke in Wikipedia ist das schon recht ausführlich erklärt... --MrBurns (Diskussion) 19:23, 22. Jan. 2016 (CET)
Okay, Technologie beinhaltet also eine Nuance im Gegensatz zu Technik, ist aber dasselbe. Oder?--Skyscraper1996 (Diskussion) 19:24, 22. Jan. 2016 (CET)
Also ich würde es so sehen: urspünglich war Technologie die Lehre der Technik, heute wird Technologie oft für ein Konzept in der Technik gesehen, also z.B. ein Computer = eindeutig Technik, die Funktionsweise eines Computers = Technologie, aber diese Verwendung würde ich als Anglizismus betrachten und auch nicht als die übliche deutsche Form... --MrBurns (Diskussion) 19:31, 22. Jan. 2016 (CET)
Jemand meinte, Technologie habe eine Analogie wovon Logik stammt, doch ich denke da ist eher analog als gegenteil von Digital, was entgegen der Technologie ist. Wie muss ich das genau sehen? Und danke für deine Meinung.--Skyscraper1996 (Diskussion) 19:36, 22. Jan. 2016 (CET)
Das Wort Technologie ist eigentlich ein Übersetzungsunfall des englischen Wortes technology, denn das englische Wort technique beinhaltet nicht das Bedeutungsspektrum des deutschen Wortes Technik, das technology mit einschließt. --Rôtkæppchen₆₈ 22:45, 22. Jan. 2016 (CET)
Kommt aber aus dem Französischen.
Wenn man sich Grimm und Duden ansieht, ist es die Technik, die komplizierter geworden ist:
  • Grimm (technik):
  • f. im 18. jh. aufgenommen aus franz. technique vom griech.-lat. technica (nämlich ars), die kunst- oder gewerbsthätigkeit und der inbegriff der erfahrungen, regeln, grundsätze und handgriffe, nach denen bei ausübung einer kunst oder eines gewerbes verfahren wird: er sei nie dahin gelangt, die kunst mit vollkommner technik auszuüben. Göthe 25, 151
  • Duden (Technik):
    • Gesamtheit der Maßnahmen, Einrichtungen und Verfahren, die dazu dienen, die Erkenntnisse der Naturwissenschaften für den Menschen praktisch nutzbar zu machen
    • besondere, in bestimmter Weise festgelegte Art, Methode des Vorgehens, der Ausführung von etwas
    • technische Ausrüstung, Einrichtung für die Produktion
    • technische Beschaffenheit eines Geräts, einer Maschine o. Ä.
    • Stab von Technikern
    • (österreichisch) technische Hochschule
  • Grimm (technologie):
    • f., im 18. jahrh. aus franz. technologie vom griech.-lat. technologia kunst-, gewerblehre, gewerbkunde; davon das adj. technologisch.
  • Duden (Technologie):
    • Wissenschaft von der Umwandlung von Roh- und Werkstoffen in fertige Produkte und Gebrauchsartikel, indem naturwissenschaftliche und technische Erkenntnisse angewendet werden
    • Gesamtheit der zur Gewinnung oder Bearbeitung von Stoffen nötigen Prozesse und Arbeitsgänge; Produktionstechnik
Play It Again, SPAM (Diskussion) 23:20, 22. Jan. 2016 (CET)
Super, vielen Dank.--Skyscraper1996 (Diskussion) 16:40, 23. Jan. 2016 (CET)
Archivierung dieses Abschnittes wurde gewünscht von: Skyscraper1996 (Diskussion) 16:41, 23. Jan. 2016 (CET)

Zeilenabstand Word

Hallo! Meine Frage hat nicht direkt mit Wikipedia zu tun, aber vielleicht kann mir dennoch jemand helfen! Ich habe eine Hausarbeit mit Open Office angefangen. Dann konnte ich Microsoft Word herunter laden und habe damit weiter gemacht. Zwischendurch habe ich auch an der Uni mit Word damit gearbeitet. Jedanfalls... ist es jetzt so, dass wenn ich eine Fußnote mache, automatisch 2 Zeilen Abstand entstehen, die den Text unleserlich machen! Und das geht auch nicht weg! Ich habe schon Stunden lang versucht, dies zu ändern. Schriftgröße der Fußnote ist acht, aber höhergestellt 12. So stehts in der Formatvorlage. Wie ändere ich das? Oder wie kann ich am besten alles in ein neues, leeres Dokument übertragen?

Verzeicht mir meine Unwissenheit...

Liebe Grüße, --134.76.62.130 20:00, 19. Jan. 2016 (CET)

Eine ganz doofe Zwischenfrage: Warum machst du nicht mit OpenOffice.org (OOo) weiter? Ich würde nichts auf Word schreiben, wenn ich auch OOo nehmen kann. Das Vorlagenkonzept bei OOo ist so ausgereift (und war es schon zu StarOffice-Zeiten), da kann Word bestenfalls dran kratzen. Zur Information: Ich schreibe beruflich Texte und habe seit Jahren kein Word mehr angefaßt. Ich arbeite ausschließlich mit LibreOffice (sozusagen der Cousin von OOo). --Kreuzschnabel 21:34, 19. Jan. 2016 (CET)
OpenOffice.org (OOo) gibt es nicht mehr. Es gibt nur noch die zwei Nachfolger LibreOffice (LibO) und Apache OpenOffice (AOO). --Rôtkæppchen₆₈ 21:57, 19. Jan. 2016 (CET)
In dem Dokument sind einige Formatierungsbefehle aus Open Office, die MS-Word nicht kennt. Also leeres Worddokument aufmachen, den Text in Open Office markieren und in den Zwischenspeicher kopieren. Dann im Word die Einfügeoption "Nur Text übertragen" wählen. Allerdings musst du dann alles neu formatieren, ggf. die Option "Formatierungen zusammenführen" probieren.--2003:75:AF16:8F00:B9F1:DA6:F30D:B8A7 22:04, 19. Jan. 2016 (CET) Da fällt mir ein, in Word gibt es auch irgendwo die Funktion Formatierungen entfernen.
„gibt es nicht mehr“ ist eine Formulierung, die mir in solchen Zusammenhängen immer etwas merckwürdig vorkömmt. Natürlich gibt es die Software noch, existierende Installationen lösen sich ja mit ihrer Abkündigung nicht in Wohlgefallen auf. Sie wird halt nicht mehr weiterentwickelt, und es ist fraglich, ob man sie benutzen möchte, aber es gibt sie schon noch :-) --Kreuzschnabel 07:12, 20. Jan. 2016 (CET)
Abgesehen davon dürften die meisten Leute "Apache OpenOffice" genauso "OpenOffice" nennen, wie sie es vorher mit "OpenOffice.org" gemacht haben. --Eike (Diskussion) 10:45, 20. Jan. 2016 (CET)
Jetzt mal 'ne doofe Frage, auch wennich mich damit unbeliebt mache: Warum wird hier schon wieder versucht, jemanden auf OO u. ä. zu eichen, wenn die Frage sich auf Word bezieht? --Gwexter (Diskussion) 17:14, 21. Jan. 2016 (CET)
Siehe Frage... Weil er mit OpenOffice angefangen hat und das Problem vermutlich gar nicht auftreten würde, wenn er einfach damit weiterarbeiten würde. --Eike (Diskussion) 20:09, 21. Jan. 2016 (CET)
Sieht ganz so aus, als wolle er mit Word weiterarbeiten und das sollte man akzeptieren. --Gwexter (Diskussion) 22:05, 21. Jan. 2016 (CET)
Hier wird übrigens mitneffen versucht, „jemanden auf OO o.ä. zu eichen“. Ich habe lediglich gefragt, wieso nicht einfach mit OO weitergearbeitet wird, wenn schon damit begonnen wurde und die Probleme erst beim Umstieg auf Word auftraten. Ich habe die Frage deshalb gestellt, weil immer noch einige Leute meinen, Word sei die einzige Textverarbeitung und alles andere nur ein Notbehelf. Und ich habe dies witzigerweise ebenfalls als doofe Frage gekennzeichnet – sogar als ganz doofe. Das mir als böswillig-krampfhaft-missionarisches Abweichen vom Thema auszulegen ist schon etwas 3st. --Kreuzschnabel 23:16, 21. Jan. 2016 (CET)
Aber man kann ihm doch erklären, dass der Umstieg anscheinend sein aktuelles Problem verursacht hat, und dass Nicht-Umstieg wohl eine Möglichkeit wäre, es zu vermeiden. Microsoft Word ist halt nicht hundertprozentig OpenOffice-kompatibel... --Eike (Diskussion) 07:15, 22. Jan. 2016 (CET)
@Fragesteller, welches Word benutzt du? Es gibt unter »Absatz« die Möglichkeit, Zeilenhöhe etc. einzustellen. Findest du bspw. bei Word 2007 im Menü »Start«. Bemerkung zur Version: Tut mir leid, mit der arbeite ich und das wird auch so bleiben. --Gwexter (Diskussion) 17:19, 21. Jan. 2016 (CET)
Der Fragestellung entnehme ich, daß all diese naheliegenden Möglichkeiten schon Stunden lang durchprobiert wurden. --Kreuzschnabel 23:16, 21. Jan. 2016 (CET)

Feinstaubbelastung

Ernstgemeinte Frage: Könnte eine Stadt wie Stuttgart eigentlich nicht die Luft mit so einer Art Staubsauger reinigen, also Luft durch Filter saugen? (Wobei das wohl eher Elektroabscheider als Filtermatten sein müßten - alternativ auch irgendwas mit feuchten Wänden, an denen sich die Staubpartikel abscheiden.) Kann mal jemand grob den erforderlichen Aufwand abschätzen?

Wahrscheinlich wäre es aber schon sinnvoll, den Staub vor allem gleich an der Quelle einzufangen, also z. B. bei Holzheizungen Staubabscheider vorzuschreiben, denn je verdünnter Schadstoffe auftreten, desto aufwendiger wird es, sie wieder einzufangen.

(Aus den Autoauspuffanlagen soll übrigens gar nicht mal soviel Feinstaub rauskommen, der verkehrsbedingte Feinstaub kommt wohl weitgehend aus Reifen- und Bremsenabrieb.) (nicht signierter Beitrag von 92.224.241.193 (Diskussion) 03:52, 20. Jan. 2016 (CET))

Das ist sicher zu aufwändig, es würde ja reichen den Luftaustausch in dem Kessel durch größere Ventilatoren anzuregen.--2003:75:AF11:2E00:998F:A665:4B1E:AFB3 07:14, 20. Jan. 2016 (CET)
Dem Vernehmen nach wurde dieses offensichtlich aussichtslose Verfahren angeblich dennoch ausprobiert, wie zu erwarten ohne jeglichen Einfluss auf die Feinstaubimmission. --Rôtkæppchen₆₈ 07:21, 20. Jan. 2016 (CET)
Wie fast zu erwarten musst du dich mit vagen und sich krümmenden Aussagen gegnerisch zu Wort melden.--2003:75:AF11:2E00:998F:A665:4B1E:AFB3 07:38, 20. Jan. 2016 (CET)
Kann man dazu Näheres erfahren?
Ist halt die Frage, ob das weniger aufwendig wäre. Aber der Ansatz ist schon richtig: Die Feinstaubbelastung tritt bei austauscharmen Inversionswetterlagen auf, und die ließen sich durch große Kamine zerstören. Fragt sich, wie energieaufwendig das ist - an sich wäre die Abwärme der Heizungen und diverser industrieller Quellen nutzbar, nur muß man die auch erst einmal "unter die Haube" kriegen.
Mal abschätzen: Der Kessel hat eine Fläche von ca. 100 km^2 = e8 m^2, davon soll innerhalb eines Tages eine Bodenschicht von 10 m Höhe abgesaugt werden. Das wären dann e9 m^3 Luft in 25 h, also ein Durchsatz von 4e7 m^3/h oder rund 10.000 m^3/s. Wenn die Luftgeschwindigkeit in den Absaugtürmen 10 m/s betragen soll, dann müssen die insgesamt einen Querschnitt von 1000 m^2 haben - das ist einigermaßen übersichtlich, z. B. zehn runde Türme mit jeweils etwas über 11 m Innendurchmesser. (Irgendwo verrechnet?)
Fragt sich, ob die Stuttgarter mitten in der Stadt Aufwindkraftwerke haben wollen - das wäre nicht so dumm, weil durch die erzielbare Energiegewinnung die "Stadtreinigung" sogar kostenlos erfolgen würde, d. h. ohne Energiekosten. Aber diese "Türmle" müßten schon ziemlich hoch werden (300 m oder mehr) und über die Alb drübergucken, und damit werden die nicht so ganz billig. Aber die vermiedenen Krankheitskosten kann man natürlich gegenrechnen. (nicht signierter Beitrag von 92.224.152.169 (Diskussion) 07:56, 20. Jan. 2016 (CET))
Hier ein Artikel von damals über einen voodoobasierten Feinstaubsauger. In diesem Artikel wird ebenfalls Voodoo angewendet. --Rôtkæppchen₆₈ 08:09, 20. Jan. 2016 (CET)
Im kleinen Deutschland natürlich unbekannt kennen unsere amerikanischen Freunde natürlich den Spary tower]. Ich kann die Kosten für so etwas im Großstadtformat allerdings nicht abschätzen. Vermeiden von Feinstäuben an der Quelle ist sicherlich billiger, als das Sammeln weit von der Quelle entfernt. Yotwen (Diskussion) 08:30, 20. Jan. 2016 (CET)
Nachtrag
Yotwen (Diskussion) 08:36, 20. Jan. 2016 (CET)
Gegen solche „Türmle“ hätte vermutlich die Deutsche Flugsicherung gewisse Vorbehalte, da der Luftraum über Stuttgart wegen des nahen Flughafens nicht ganz verkehrsfrei ist. Ob man sich da auch noch Turbulenz-Erzeuger hinstellen will … --Kreuzschnabel 08:45, 20. Jan. 2016 (CET)
Erinnert mich daran, wie sich einer unserer Profs aufregte: „Selbst die Holländer, die ebene Städte und genug Wind haben, bauen ihre Großfeuerungen weit außerhalb. Die Stuttgarter sind so schlau, sich ein großes Heizkraftwerk in ihren Talkessel zu bauen. Andererseits sind da immerhin schon 700.000 Feinstaubfilter installiert: die Lungen der Stuttgarter …“ --Kreuzschnabel 08:49, 20. Jan. 2016 (CET)
 
Vielleicht funktioniert sowas auch umgekehrt? --Optimum (Diskussion) 12:44, 20. Jan. 2016 (CET)
Bestimmt, irgendwo muss doch die Grundlast hin, also wirf die Vogelguillotine an, damit die Netzfrequenz bei 50 Hz bleibt. --Hans Haase (有问题吗) 14:32, 20. Jan. 2016 (CET)
@Kreuzschnabel: Gegen den Fernsehturm auf dem Hohen Bopser hatte die Flugsicherung anfangs auch etwas. Heute gibt es mindestens vier Sendetürme in Stuttgart (Stuttgarter Fernsehturm, Stuttgarter Fernmeldeturm, Stuttgarter Funkturm, Vodafone-Funkturm Stuttgart-Vaihingen, …) --Rôtkæppchen₆₈ 09:01, 20. Jan. 2016 (CET)
@Yotwen: wir haben den Artikel Gaswäscher und die dort verlinkten Rotationswäscher, Sprühwäscher, Strahlwäscher, Venturiwäscher und Wirbelwäscher. Wer verinterwikilinkt? --Rôtkæppchen₆₈ 09:25, 20. Jan. 2016 (CET)
Vielleicht schaffen es mehrere große Hubschrauber eine Kaminwirkung zu erzeugen, dass die warme Feinstaubbelastete Glocke am Boden darin nach oben steigt. Hat noch keiner untersucht und probiert?--2003:75:AF11:2E00:907A:984:77B3:E77A 09:40, 20. Jan. 2016 (CET)
Ich fürchte, die Dicke der Wirbelschleppe der Rotorblätter ist geringer als die der zu überwindenden Inversionsschicht. --Rôtkæppchen₆₈ 09:51, 20. Jan. 2016 (CET)
Wirbelschleppe beim Hubschrauber ist wahrlich gering, aber der Rotorabwind ist da um so heftiger. Wenn man die in Höhe und Länge staffeln würde, könnten die zusammen einige Luftmassen bewegen.--2003:75:AF11:2E00:D5A1:17BE:5C09:BB1A 16:44, 20. Jan. 2016 (CET)
Gegen den Feinstaub wird nichts gemacht, außer Ampel rot gestellt, was wiederum mehr Staub an anderer Stelle erzeugt. Das ganze ist nur eine Verbrauchsdiktatur. Da mehr als die Hälfte der Neufahrzeuge nicht von Pivatkunden gekauft werden, spricht der Endverbraucher auch bei der Entwicklung nicht mehr mit. Stuttgart hat keine Stadtautobahnen. Es ist nicht möglich ohne Stop durch Stuttgart zu fahren. Dadurch werden auch Benziner und Elektrofahrzeuge mit ihren Bremsen zu Feinstaubverursacher. Der öffentliche Nahverkehr ist in Stuttgart bundesweit der teuerste. Auch ist er im Fahrtakt nicht attraktiv. Möglicherweise hat der Feinstaub bereits die ein oder andere Bluthirnschranke passiert, so dass er sich auch die Verwaltung einsetzt, ihn zu erhalten. Feinstaub wird eingeschränkt durch Wasser gebunden. Er lässt sich auch elektrostatisch binden. Das funktioniert in Kopiergeräten und an der Hochspannungsleitung von Bildröhren. --Hans Haase (有问题吗) 10:01, 20. Jan. 2016 (CET)
"Der öffentliche Nahverkehr ist in Stuttgart bundesweit der teuerste." - Wie kommt man zu dieser Einschätzung? Ein Einzelticket für eine Fahrt im gesamten Stadtgebiet kostet 2,80 Euro, eine Tageskarte 6,80 Euro und eine Monatskarte 83 Euro. Zum Vergleich: In Köln kostet zwar die Einzelfahrt ebenfalls 2,80 Euro, die Tageskarte aber 8,50 Euro und die Monatskarte 92,90 Euro. (Und bevor jemand darauf hinweist, dass Köln auch größer als Stuttgart ist: Im deutlich kleineren Bonn gelten dieselben Preise.) --91.221.58.20 16:27, 20. Jan. 2016 (CET)
Hier wird das Thema aus der verbreiteten reversinnovatien Sicht thematisiert.[7] --Hans Haase (有问题吗) 11:10, 20. Jan. 2016 (CET)
https://www.google.de/search?q=reversinnovativ --Eike (Diskussion) 11:13, 20. Jan. 2016 (CET)
en:Reverse_innovation 78.52.220.164 19:52, 20. Jan. 2016 (CET)
Im Gegenteil: die meisten Städte versuchen, die Schadstoffbelastung dadurch zu senken, dass sie mehr grüne Wellen installieren. Was natürlich schief geht, da mehr grüne Wellen eine höhere Attraktivität für den Autoverkehr bedeuten und dann noch mehr Leute mit dem Auto fahren, statt andere Verkehrsmittel zu benutzen. Maßnahmen die dazu dienen "ohne Stop durch Stuttgart zu fahren" sind das unsinnigste, was man in einer solchen Situation machen kann (die Stuttgarter versuchen es - wie auch andere Städte - trotzdem). Was es bräuchte, wäre eine Abkehr von der Priorisierung des Flüssigkeit des Autoverkehrs vor der Attraktivität der anderen Verkehrsmittel. Aber das ist in einer Stadt wie Stuttgart vermutlich illusorisch.--141.30.184.93 15:21, 20. Jan. 2016 (CET)

Wenn Du keinen Feinstaub haben möchtest, musst Du dir ein Umweltzonenschild vor dein Haus stellen. Hinter diesen Schildern gibt es bekanntlich keinen Feinstaub. VG Thogru (Diskussion) 11:48, 20. Jan. 2016 (CET)

Da ist Stuttgart das prominenteste Gegenbeispiel. --Rôtkæppchen₆₈ 11:51, 20. Jan. 2016 (CET)
Google Maps, Streetview und die Websuche machen es möglich, dass sich jeder vom Zustand überzeugen kann. Die Baumaschinen von den aktuellen Großbaustellen liefern auch ihren Beitrag dazu. --Hans Haase (有问题吗) 16:32, 20. Jan. 2016 (CET)
Streetview? In Deutschland? Dazu hatte Uschi viel zu viel Angst um ihre Familie. Sieht man ja an Großbritannien, wo Streetview so ziemlich jede öffentliche Straße erfaßt hat: dort sind Familien inzwischen praktisch unbekannt. --Kreuzschnabel 21:11, 20. Jan. 2016 (CET)
Stuttgart ist in Google Streetview präsent. --Rôtkæppchen₆₈ 22:35, 20. Jan. 2016 (CET)

Haase, Elektroautos kann man so fahren, dass man die Bremsen kaum benutzen muss. Das hängt davon ab, wie stark rekuperiert wird. Bei vielen Fahrzeugen kann man einstellen, wie stark die Reku sein soll. Das fühlt sich an wie die "Motorbremse" beim Verbrenner. Du regst mich langsam echt auf, Haase. Welche Elektroautos bist du denn schon gefahren? Je weniger Verbrenner auf den Straßen fahren, desto besser. Dann verringert sich nicht nur der Feinstaubanteil in der Luft, sondern auch die schädlichen NOx-Verbindungen etc. und nicht zuletzt der LÄRM. Feinstaub kommt aber nicht nur aus den Auspuffrohren der Verbrenner, sondern auch von den "Auspuffen" der Häuser. Und das nicht zu knapp. Wer meint, dass er sich und seiner Umwelt mit dem Verbrennen von Holz und/oder Heizöl einen Gefallen tut, der irrt. Insbesondere die Holzverbrennerei erzeugt viel Feinstaub. Leider gibt es da ja bislang nur wenige Alternativen. Das ist schade. Bei den Fahrzeugen hingegen bin ich optimistisch, dass sich das bis 2035 erledigt hat. Bis dahin werden Verbrenner nahezu unverkäuflich sein. Und Rotkäppchen, Elon Musk verheimlicht uns gar nichts. Auf die Akkus gibt Tesla 8 (acht!) Jahre Garantie. Lies lieber mal bisschen im TFF-Forum die Erfahrungsberichte von den Leuten, die die Autos seit über 100.000 km fahren, statt hier Menschen zu diskreditieren, die dafür sorgen wollen, dass die Welt in Zukunft lebenswerter wird. --Kapitän Turnstange ¯\_(ツ)_/¯ 21:51, 20. Jan. 2016 (CET)

Der Feinstaub aus dem Straßenverkehr stammt großteils aus dem Reifenabrieb. Daran ändert ein Elektroauto, tja, gar nichts. Fahr Straßenbahn, Mann. -- Janka (Diskussion) 22:02, 20. Jan. 2016 (CET)
Lüge. "25 Prozent der verkehrsbedingten Feinstaubemissionen sind auf die Entstehung durch Abrieb und Aufwirbelungen zurückzuführen", 50 % durch Verbrennungsmotoren, vor allem Dieselmotoren. "Aerosole entstammen im Wesentlichen dem Auspuff von Dieselmotoren...", so die Deutsche Umwelthilfe. Medical Tribune: Etwa die Hälfte der Feinstaub-Belastung stammt aus den Auspuffen von Dieselfahrzeugen wie Lastern, Bussen und Pkw. --Kapitän Turnstange ¯\_(ツ)_/¯ 22:35, 20. Jan. 2016 (CET) "Die einzelnen Verursacher von PM10-Emissionen im Straßenverkehr sind schwere Diesel-Nutzfahrzeuge 26 %, Diesel-Pkw 22 %, leichte Diesel-Nutzfahrzeuge 7 %, Pkw mit Ottomotor 12 % sowie Reifenabrieb 19 %, Bremsabrieb 8 % und 6 % Sonstige." --Kapitän Turnstange ¯\_(ツ)_/¯ 22:44, 20. Jan. 2016 (CET)
Erstens bezieht sich gleich die erste Studie auf die Zeit vor 2007, als es noch keine Umweltzonen gab. Und das Beispiel Stuttgart war gleich zu Anfang dabei. Zwei Varianten: Entweder die Feinstaubplakettiererei ist sinnlos oder der Feinstaub aus Dieselabgasen hat erheblich nachgelassen und das was uns heute noch umbringt ist das Zeug aus dem Reifenabrieb. Zweitens sind die Dieselabgase vor allem "Grobstaub" (sonst könnte man die nämlich gar nicht sehen) und was die Gesundheitsschädlichkeit angeht ist der unsichtbare wirkliche Feinstaub der Feind. Und auch der stammt im Verkehrsbereich wiederum hauptsächlich aus dem Reifenabrieb. -- Janka (Diskussion) 23:21, 20. Jan. 2016 (CET)
Wo sind deine Quellen? So kannst du bei Papa in der Wirtschaft argumentieren, hier nicht. --Kapitän Turnstange ¯\_(ツ)_/¯ 00:40, 21. Jan. 2016 (CET)
Junge, es sind deine Quellen! Man muss natürlich auch verstehen, was drinsteht, und warum. Nimm dir zum Beispiel auch mal deine letzte Quelle dazu. Da sind die Aufwirbelungen durch den Straßenverkehr einfach aus diesem Posten rausgerechnet. Nimmt man sie mit hinein, wird der Anteil der Abgase an der Gesamtbelastung durch den Verkehr glatt halbiert. Was wohl in diesem Aufgewirbelten so drin ist? Meine Vermutung: Alter Reifenabrieb. Würde ich zumindest auf einer Straße suchen.
Manchmal hilft aber auch nur die pure Logik: Wenn in deiner Zukunft von 2035 alle mit Elektroautos rumfahren, ist der Feinstaubanteil aus dem Reifenabrieb selbstverständlich 100%. Und die Aufwirbelungen bleiben auch identisch. Und dann werden die Leute endlich einsehen, dass Elektroautos das Feinstaubproblem auch nicht lösen. Hoffentlich passiert das früher, bevor die Umwelt durch die Fertigung unnützer Waren unnötig zerstört wurde. -- Janka (Diskussion) 01:17, 21. Jan. 2016 (CET)
Du gibst überhaupt keine Quelle an. Ich hab 2 Min. gegooglet und dir ein paar Zahlen hingeklatscht, die deine Zahl widerlegen. Ach so, und weil der Reifenabrieb auch bei Elektroautos nicht verhindert werden kann, sollen Verbrenner, die noch so einige andere schädliche Dinge emittieren, nicht durch Elektroautos ersetzt werden? Das entbehrt jedweder Logik. Und komm mir nicht wieder damit, dass dann irgendwelche schmutzigen Kohlekraftwerke den Strom für die Fahrzeuge produzieren müssen. Es ist viel einfacher, die Emissionen WENIGER Kohlekraftwerke in den Griff zu bekommen als die von MILLIONEN einzelner Fahrzeuge. Das muss doch auch dir einleuchten. Und der Anteil regenerativer Energien am Strommix wird kontinuierlich ansteigen. Ich bin übrigens nicht dafür, alle Fahrzeuge durch EVs zu ersetzen. Ich bin für nachhaltige Mobilitätslösungen. Verbrenner sind jedenfalls nicht die Lösung. Du machst dir Gedanken um die Umweltzerstörung durch die Herstellung von EVs? Sonst bist du gesund, ja? Wo kommt denn der Treibstoff her, mit dem Papas Auto getankt wird, hm? Und das ganze alte Getriebe- und Motoröl kommt auch nicht in den Salat, weisst du... Schon die ganzen Ölkatastrophen vergessen? Durch die Förderung und Verbrennung von Öl ist die Umwelt schon genug zerstört worden, oder nicht? Entweder du bist abhängig von einer dieser Industrien oder du hast so einiges nicht verstanden. Aber wer pauschal behauptet, dass alle Menschen dumm sind, die Luxusgüter wie iPhones und das Tesla Model S kaufen, ist wohl eher langsam in der Auffassung und fehlbar in den Rückschlüssen... --Kapitän Turnstange ¯\_(ツ)_/¯ 11:14, 21. Jan. 2016 (CET)
Ich lese deine Quellen. Nur eben anders als du. Habe ich dir bereits geschrieben.
Aber auch so, liebe Turnstange: Ich habe mein Auto vor 5 Jahren abgeschafft. Pro Woche fahre ich zweimal 25km zum Einkaufen, dafür lohnt sich ein eigener Wagen nicht. Ich leihe mir daher einen und wenn ich das nicht könnte würde ich ein Taxi rufen. Das ist billiger als der Unterhalt eines eigenen Wagens. In die Stadt zu ziehen wegen kürzerer Wege kommt für mich nicht in Frage, du verstehst sicher - der Feinstaub. Aber wer das unbedingt will, der sollte ein striktes Autoverbot für seine Stadt fordern, und um den Leuten die Sache schmackhaft zu machen kostenlosen Bus- und Bahnverkehr. Und siehe da, plötzlich wird die Luft besser, selbst in Stuttgart würde das funzen, wenn Daimler und Porsche nicht dagegen wären. Wahrscheinlich weil die Leute dann merken würden, dass man gar keine Autos braucht und Elektroautos schon gar nicht.
(Und ja, wer sich einen Tesla oder ein iPhone kauft - und damit dann womöglich noch hausieren geht - hat gewaltige Komplexe. Ob du dir diesen Schuh selbst anziehst, ist deine Sache. Weiß ich ja nicht, ob du solchen Mist benötigst.) -- Janka (Diskussion) 01:15, 22. Jan. 2016 (CET)


Vom CO2-Ausstoß her sind Fossilwärmekraftewerke, egal ob Gas, Erdöl oder Kohle, und Elektro-Kfz den Fossil betankten Kfz deutlich unterlegen und Motor- und Getriebeöl ist nicht wirklich das Problem und war es auch nicht. Elektro-Kfz sind nur mit Atom- oder Ökostrom sinnvoll, wobei beim Atomstrom bei vielen Leuten ein ökologisches Brett vorm Kopf ist. --Rôtkæppchen₆₈ 11:23, 21. Jan. 2016 (CET)
Die Partei, die damals die Regierung aus einem Automobilkonzern heraus bestellt hatte, ist doch stets bemüht, ihre Diesel zu verkaufen und das Lohnniveau abzusenken. Da leistet sich niemand einen Tesla. Die Realität sieht so aus, dass die Familienkutsche das abschriebene Vertreterauto ist. Da es vom Erstbesitzer für Langstrecke gekauft wurde, ist es ein Diesel. Die Reparaturen wie Turbo und Injektoren soll dann der Nachbesitzer investieren, der nach Anschaffungspreis kaufen muss. Mit den Batterien ist das E-Fahrzeug ein Kostenrisiko und damit nur etwas für Besserverdiener. Der Rest ist indertat so dumm um sich diesen Blödsinn diktieren zu lassen und dafür braucht Deutschland Flüchtlinge, die aus Ländern kommen, in denen nur Fossile Energie benutzt wird, weil es dort billig war und sie deswegen nichts anderes kennen, und von Demokratie noch weniger wissen als die Deutschen heute. Autokauf ist nur auf Glück und Kosten auf Schmerzgrenze. Hinterher war es natürlich teurer, nur die Leute werden arm gehalten, damit sie das Geld anders ausgeben. Das ist auch die einzige ernstzunehmende Innovation. --Hans Haase (有问题吗) 12:54, 21. Jan. 2016 (CET)

Hervorragender Abschluss für diese Diskussion, Hans Haase. Unverständlich, aber hervorragend. Danke. 178.82.72.225 13:39, 21. Jan. 2016 (CET)

Elektro-PKW sind kaum leiser als herkömmliche PKW, eine relevante Lärmreduktion wird es dadurch nicht geben. Ab ca. 30 km/h ist das Reifen-Fahrbahn-Geräusch maßgeblich für den Lärmpegel der PKW, womit Elektroautos nur in Straßen mit weniger als 30 km/h zu einer Lärmreduktion führen werden.--141.30.184.92 16:17, 21. Jan. 2016 (CET)

Wenn man diversen Medienberichten glaubt (willkürlicher Google-Fund - die Bremsen sind's) sind auch U-Bahnen für Feinstaub verantwortlich. Und bei Entlüftungsanlagen der Tunnel könnte man schon Filter einbauen. Ebenso bei Tunneln für den Straßenverkehr (davon gibt es in Stuttgart auch ein paar). Kleinvieh macht ja auch Mist. ;) Man könnte natürlich auch einfach die ÖPNV-Preise senken, zumindest Aktionsweise. Solange eine Autofahrt günstiger ist... --StYxXx 04:35, 22. Jan. 2016 (CET)

Eine Autofahrt ist in den meisten Fällen nicht günstiger - je nach Ausgangs- und Zielort, Parkplatzverfügbarkeit und -kosten, PKW-Modell etc u.U. sogar sehr viel teurer. Und mit Einbezug des volkswirtschaftlichen Kosten sogar immer deutlich teurer. Das Problem sind deshalb auch weniger die Preise des ÖPNV (der wird sowieso schon subventoniert), sondern die noch niedrigeren (weil noch stärker subventionierten) Preise des PKW-Verkehrs. Die Lösung für Stuttgart (wie auch für andere Städte) wäre eine Maut, analog bspw. der London Congestion Charge - die erreicht gleich in mehrfacher Hinsicht das, was man braucht: Einen Rückgang der KFZ-Verkehrsmenge, dadurch weniger Abgase und weniger Staus (wodurch diejenigen, die tatsächlich das Auto benötigen, auch schneller voran kommen), und mit den Einnahmen aus einer solchen Maut lassen sich dann auch notwendige Investitionen in den Nahverkehr tätigen. Die Lösung wäre also vorhanden - man muss sie nur umsetzen.--92.228.244.170 09:18, 22. Jan. 2016 (CET)
"Eine Autofahrt ist in den meisten Fällen nicht günstiger" - Ich weiss ja nicht wo du wohnst, aber an einer deutschen Grossstadt ist das Unsinn. Beispiel 1: Ich muss heute ploetzlich noch von Koeln nach Bern: Kostet mich mit der Bahn 300zequetschte Euros (ja, passiert mir gelegentlich). Was? Mit meinem Auto mach ich das fuer die Haelfte, und brauch die selbe Zeit (Flexibilitaet nicht so ausschlaggebend, ich koennte nen Zug aller 20 Min kriegen). Beispiel 2: Ich fahr mit meiner Familie in die Stadt einkaufen. Kostet mich hin und zurueck mit 3 Personen etwa 12 Euro Strassenbahn. Was? Mit dem Auto (30 Cent/km gute Pauschale) komme ich auf etwa 6 Euro, ich parke fuer 2 Stunden kostenlos, ich muss mein Zeug nicht schleppen. Nee, eine Autofahrt ist nur halbsoteuer wie die Oeffies. --195.37.46.254 10:44, 22. Jan. 2016 (CET)
Wie ich schon schrieb: Je nach Ausgangspunkt, Uhrzeit, usw. Deine Beispiele sind zumindest nicht repräsentativ: Wie oft kommt es denn vor, dass jemand plötzlich erfährt, dass er am selben Tag noch eine 500 km lange Fahrt machen muss - statt dies, wie üblich, zumindest einige Tage vorher zu wissen? Mal abgesehen davon, dass der Anteil von Fahrten mit >200 km im Promillebereich liegt (und ich aufgrund dre Fragestellung sowieso eher den Stadtverkehr im Auge hatte). Auch dein Beispiel mit Familie ist bei drei Personen im Auto nicht repräsentativ: Der Besetzungsgrad von PKW (also die Zahl der Personen pro Auto) liegt bei unter 1,3, im Berufsverkehr bei gerade noch 1,1.
Ich könnte dir jetzt beliebig viele Beispiele (auch im Fernverkehr) heraussuchen, bei denen selbst die reinen Spritkosten des PKW deutlich über den Fahrpreisen des ÖPNV liegen. Nur: Der Standardfall sind nicht plötzlich anfallende, lange Fahrten, sondern regelmäßige, eher kurze Fahrten. Wenn du wirklich wissen willst, was insgesamt (also nicht nur in willkürlichen Beispielen) günstiger ist, muss du wohl auf Statistiken zurückgreifen. Wobei auch ohne Statistik die beiden zuverlässig und konkurrenzlos günstigsten Verkehrsmittel sowieso die eigenen Füße und das Fahrrad sind...--141.30.184.99 15:28, 22. Jan. 2016 (CET)
Leute, das sind Äpfel und Birnen. Wenn man nur den reinen Ort-zu-Ort-Transport betrachtet, ist das Auto aufs Jahr gesehen immer teurer, weil es auch Geld kostet, wenn es zuhause in der Garage steht, z.B. in der Zeit, in der man im Urlaub ist. Dafür dauert die Fahrt mit Bus und Bahn grob geschätzt immer doppelt so lange. Ja, jeder kennt eine Strecke, wo das nicht so ist, aber die meisten Leute wohnen nicht neben einem Bahnhof und wollen auch nicht zu einem anderen Bahnhof, müssen also i.d.R. zweimal umsteigen, was mit entsprechender Wartezeit verbunden ist. Und Bahnfahren verliert enorm an Reiz, wenn man drei, vier Aktenordner mitnehmen muss. Oder in der Stadt ein kleines Schränkchen kauft. Das Auto ist gleichzeitig sowas wie ein Tresor: man kann seine Einkäufe zwischenlagern und unbeschwert weiter shoppen. Die Preise zwischen Auto und ÖP(N)V sind unterschiedlich, man bekommt aber auch ganz unterschiedliche Sachen. --Expressis verbis (Diskussion) 15:56, 22. Jan. 2016 (CET)

Vogelbeobachtung

Hat jemand einen sandfarbenen Feldstecher? Reagieren Vögel darauf anders als bei Schwarzen oder Dunkelgrünen? --85.4.233.141 17:02, 20. Jan. 2016 (CET)

Sichtbarmachung...

Wenn eine bestimmte Farbe geeigneter wäre als eine andere, würden bird watching Clubs vor allem diese anpreisen. Da sie aber in Schwarz, Dunkelgrün, Braun, Beige und Tarnfarbe verfügbar sind, scheint dem nicht so zu sein und der Mark macht es sich zum Vorteil. Tönung oder Coating (o.ä.) des Glases könnte wichtiger sein, da es Reflektion (Blitzen) vermeidet. Play It Again, SPAM (Diskussion) 17:18, 20. Jan. 2016 (CET)
(BK)Ich kenne vorallem einen anderen Vorteil von hellen statt dunklen Objektiveanstrichen, die werden weniger schnell warm um nicht zu sagen heiss. Starke Erwährmug mag kein wirklich gutes optisches Gerät, und darunter fallen auch Feldstecher. Tarnfarbe ist nicht alles, denn es ist auch eine Frage wie stark sich etwas vom Hintergund abhebet. Wenn du beim Vogelbeobachten helle Kleidung trägts, ist ein heller Gegenstand im deinen Händen schlechter erkennbar, somit auch deine Bewegungen damit. Bei dunkler Kleidung -nach möglichkiet noch mit dunklen handschuhen) wäre der helle Gegenstand erst recht gut erkennbar. Ob die Frontlinse "aufblitz" oder nicht, ist dann schon eher ein Störgrund, der sit aber von der Gehäusefarbe unabhängig. Klar ein matter Anstich ist generell angesagt, damit eben nicht was hinten spiegelt und "aufblitzen" kann.--Bobo11 (Diskussion) 17:22, 20. Jan. 2016 (CET)
Ich dachte mir, das beige augenförmige Gehäuse im obigen Link könnte vielleicht als natürlicher Feind wahrgenommen werden.--85.4.233.141 22:17, 20. Jan. 2016 (CET)
Ich glaube, dass es bei Ferngläsern dieser Größe, wenn überhaupt, dann auf die Farbe der Handschuhe ankommt. --Wicket (Diskussion) 23:28, 20. Jan. 2016 (CET)
nach meinen Erfahungen mit Vögeln, oder auch anderen Tieren,ist es viel wichtiger wie schnell man sich bewegt, wenn es keine hektischen Bewegungen (und somit aus keine hektischen Reflexionen) gibt, sondern sich das alles seeehhhr langsam abspielt, und man sich ausserhalb des jeweiligen (bei den einzelnen Tieren sehr unterschiedlichen) Fluchtabstandes aufhält, sollten Farben relativ egal sein. --Jmv (Diskussion) 00:12, 21. Jan. 2016 (CET)
Richtig, sich langsam bewegen wird viel wichtiger sein, als die Farbe des Feldstechers. Welches die richtige Farbe ist, ist wie oben geschrieben, viel mehr von deiner üblichen Bekleidung abhängig, denn von einer ganz bestimten Farbe. Ob eben der Vogel das ansetzen des Feldsteckers (kippbewegung) erkennen kann oder nicht. Und natülich auch -wenn er es erkennt-, ob er das als Bedrohung auffast oder nicht. --Bobo11 (Diskussion) 05:19, 21. Jan. 2016 (CET)
Verstehe. Wobei ich den Olivgrünen dann als Mensch dann doch als sehr viel unauffälliger empfinde, der verschwindet förmlich in der Wahrnehmung, warum das Militär wohl hierzulande auch diese Farbe benutzt. Auch die schwarzen Okularabdeckungsaufsätze sind etwas Kontrastreicher beim "zivilen Modell", es hat es auch noch chromfarbene Ausziehenstulpen, die reflektieren könnten. Nun, ja, ich habe dann wohl die Qual der Wahl. Schön sind beide. Keine leichte Entscheidung. --85.4.233.141 11:30, 21. Jan. 2016 (CET)
Vögel reagieren auf "Augen" und finden nur das Objektiv bedrohlich. Die Farbe der Hülle drum rum ist ziemlich wurscht. Dieses machen sich auch Schmetterlinge zu nutze, indem sie Augen auf ihre Flügel malen um ein größeres Tier dahinter vorzutäuschen.--2003:75:AF10:700:30CF:D031:125D:6EA8 11:35, 21. Jan. 2016 (CET)
Der Sinn dieser "Schmetterlingsaugen" ist m.W. nach wie vor strittig. Aber Gerade die würden für die These sprechen, da gibt es nämlich keine Linse oder was meinst du mit dem Objektiv?--85.4.233.141 13:27, 21. Jan. 2016 (CET)
Der amerikanische Philosoph und Wahrnehmungsfachmann G. Larson hat sich damit beschäftigt, wie Vögel Menschen wahrnehmen. Play It Again, SPAM (Diskussion) 14:24, 21. Jan. 2016 (CET)
:)--85.4.233.141 15:32, 21. Jan. 2016 (CET)
Das Objektiv ist bei einem Fernglas der Teil, der zum Objekt zeigt (daher der Name). Das ist groß und rund und könnte durchaus für Augen eines großen Tieres gehalten werden. Ob ein Tier mit so großen Augen schwarz, sandfarben oder dunkelgrün ist, ist dem Tier, das sich bedroht fühlt, egal.
Mir persönlich wär's übrigens auch egal, wenn ich ein Vogel wär. --Snevern 16:16, 21. Jan. 2016 (CET)
Die Frage ist aber, ob die eine Farbe dem entsprechenden Tier vielleicht eher auffällt. Camouflage ist ja auch im Tierreich ein ziemlich verbreites Phänomem -> Signalspiele. --85.4.233.141 19:07, 21. Jan. 2016 (CET)
Ja, das stimmt sicher. Ich habe aber nun mal leider nur ganz wenig Ahnung von Vogelkunde und Vogelbeobachtung - ich habe nur die bis dahin unbeantwortete Frage "was meinst du mit dem Objektiv?" im hier vorliegenden Zusammenhang beantworten wollen. --Snevern 19:51, 21. Jan. 2016 (CET)
Nimm gescheckt wie ein Sperber, der fängt bei mir öfters Rotkehlchen an der Futterstelle. Komischerweise hab ich noch nicht beobachtet, dass er einen Sperling oder ne Meise erwischt.--2003:75:AF11:E900:141F:D72E:1813:B6C5 09:33, 22. Jan. 2016 (CET)
Hab' mich für die Tan-Version entschieden, sieht etwas ziviler aus und man wird nicht gleich mit einem Pegidaanhänger verwechselt.--85.4.233.141 15:16, 22. Jan. 2016 (CET)

Lied "Darling" (1970er Jahre)

Hallo, ich suche den Sänger und/oder Komponisten (am besten Orginal) von diesem Song ab 2'24.

Laut Tracklist (A6.c) heißt er nur "Darling".

Danke herzlichst, --93.133.76.134 15:24, 21. Jan. 2016 (CET)

Hier zum hinterher gegenchecken. Play It Again, SPAM (Diskussion) 16:18, 21. Jan. 2016 (CET)
Mein Tipp geht in Richtung des Liedes von Rolf Soja, ursprünglich von Baccara (Band). Die grobpixelige Komponistenangabe hier (2. Spalte ganz unten) könnte doch R. Soja heißen. --Rôtkæppchen₆₈ 16:36, 21. Jan. 2016 (CET)
[8]? --Hans Haase (有问题吗) 16:45, 21. Jan. 2016 (CET)
Uraltes Zeuch: Amor (1943) und Quando (1962). In der en-Wp steht mehr --RobTorgel 16:47, 21. Jan. 2016 (CET)
Auf der Scheibe wurde wohl in Deutschland populäre Musik des Jahres 1977/78 für Elektronische Orgel neu arrangiert. --Rôtkæppchen₆₈ 16:52, 21. Jan. 2016 (CET)
Steht aber doch eh auf de o.a. Tracklist ??? A4.a u. A4.b --RobTorgel 17:00, 21. Jan. 2016 (CET)
Wie kommt ihr auf die zwei spanischen Fuchteln ? Die singen doch was anderes, aber sowas von --RobTorgel 17:04, 21. Jan. 2016 (CET)
Welches andere in Frage kommende Lied des in Frage kommenden Zeitraums trägt den genannten Titel? --Rôtkæppchen₆₈ 17:21, 21. Jan. 2016 (CET)
Nochmal auf Null: Ab den gefragten 2.24 läuft bei mir eine gewöhnungsbedürftige Fassung von Amore mio (Original Amada mia, amore mio von Paolo Zavallone, genannt El Pasador). Ansonsten verstehe ich einfach nicht, worum es hier geht... Grüße Dumbox (Diskussion) 17:28, 21. Jan. 2016 (CET)
Danke@Dumbox, Volltreffer: https://www.youtube.com/watch?v=6_DnQ-NqF04 --93.133.76.134 20:05, 21. Jan. 2016 (CET)
Hat sich Ray Manzarek eigentlich je zu dieser Art des Orgelmissbrauch geäußert? --Rôtkæppchen₆₈ 23:34, 21. Jan. 2016 (CET)
In D wurde das Lied bekannt als Titelsong des Jugend-Krimis Die Vorstadtkrokodile, der 1977 erstausgestrahlt und später immer wieder mal (z. T. auch als Zweiteiler) wiederholt wurde. Als Kind war ich begeistert von dem Film. Gut gespielt mit vielen Laiendarstellern und schön spannend. --Blutgretchen (Diskussion) 09:16, 22. Jan. 2016 (CET)

Bild von der Messe

Wie bekomme ich mein bild von der messe am 21.01.2016 ? --78.43.50.228 10:21, 22. Jan. 2016 (CET)

Zuallererst wäre es hilfreich zu erfahren, welches Bild und welche Messe du meinst. --j.budissin+/- 10:25, 22. Jan. 2016 (CET)
Messe an einem Donnerstag?--Wikiseidank (Diskussion) 13:20, 22. Jan. 2016 (CET)
Wieso nicht? Sowohl Warenschauen als auch Gottesdienste gibt es auch an Donnerstagen. --MrBurns (Diskussion) 13:22, 22. Jan. 2016 (CET)
Mancherorts werden auch kirmesartige Volksfeste als Messe bezeichnet. Googelei stellt schnell klar, dass am 21. Januar 2016 einige (Handels)Messen stattgefunden haben. --Rôtkæppchen₆₈ 16:08, 22. Jan. 2016 (CET)

Wo liegt das Problem mit einer PDF-Buchseite?

Es geht um diese Buchseite bei Hathi trust. Mit dem Internet Explorer betrachtet, scheint alles normal. Mit dem Firefox aber ist kein Text zu sehen. Er ist aber doch da (der Text), weil man die weiße Seite nach Wörtern (OCR) durchsuchen kann.

Nachdem man die Seite herunterlädt und abspeichert, und sei es vom Firefox, ist das ganze wieder eine ganz normal sicht- und lesbare PDF-Seite. Was geht hier vor? Liegt das Problem bei Hathi Trust, beim Firefox, oder wo? --Ratzer (Diskussion) 13:14, 22. Jan. 2016 (CET)

Der PDF-Viewer vom Firefox ist nicht sonderlich gut, da kommt es öfters zu Darstellungsfehlern. In dem Fall kann man den Downlaodbutton drücken (schaut aus wie eine Blatt Papier mit einem Pfeil nach unten) und "Öffnen mit" wählen, um die Seite mit einem normalen PDF-Programm zu öffnen. --MrBurns (Diskussion)

Unterliegt Sangria dem Alkopopsteuergesetz (Deutschland)?

Müsste es eigentlich. --Tastaturtest Belgisch (Diskussion) 08:05, 20. Jan. 2016 (CET) --Tastaturtest Belgisch (Diskussion) 08:05, 20. Jan. 2016 (CET)

Durch geschicktes Decodieren der sog. Buchstaben in den Artikeln Alkopopsteuergesetz (Deutschland) und Sangría lässt sich herausfinden, dass das individuell von der Zusammensetzung der Sangría abhängt, je nachdem, ob Branntwein enthalten ist oder nicht. --Rôtkæppchen₆₈ 08:14, 20. Jan. 2016 (CET)
Wenn du keine hilfreiche Antwort geben möchtest, steht es dir frei, alternativ gar nichts zu sagen. Hab' einen schönen Tag. --Tastaturtest Belgisch (Diskussion) 08:26, 20. Jan. 2016 (CET)
Der Beitrag beantwortet in meinem Augen die Frage vollumfänglich und gibt dazu auch noch Quellen an. Ich halte den Beitrag für hervorragend. --BlackEyedLion (Diskussion) 08:41, 20. Jan. 2016 (CET)
Die richtige Antwort wäre gewesen: "Nein, ist kein Alkopop." Inzwischen hab ich es wikipediaextern nachgelesen. Wollte mir aber eigentlich Zeit sparen, weil ich eine Frage wikipediaextern schnell hätte beantworten müssen. (nicht signierter Beitrag von Tastaturtest Belgisch (Diskussion | Beiträge) 08:47, 20. Jan. 2016 (CET))
 
Sangriazutaten
Falsch. Schau Dir mal die Flaschenetiketten genau an. --Rôtkæppchen₆₈ 08:54, 20. Jan. 2016 (CET)
(BK) Die Antwort ist falsch: Sangría unterliegt der Alkopopsteuer, wenn sie ein Getränk ist, das
  • „aus einer Mischung von Getränken mit einem Alkoholgehalt von 1,2 % vol oder weniger oder gegorenen Getränken mit einem Alkoholgehalt von mehr als 1,2 % vol mit Erzeugnissen nach § 130 Abs. 1 des Gesetzes über das Branntweinmonopol bestehen,
  • einen Alkoholgehalt von mehr als 1,2 % vol, aber weniger als 10 % vol aufweisen,
  • trinkfertig gemischt in verkaufsfertigen, verschlossenen Behältnissen abgefüllt sind und
  • als Erzeugnisse nach § 130 Abs. 1 des Gesetzes über das Branntweinmonopol der Branntweinsteuer unterliegen.“
Die Antwort gilt im Übrigen für Deutschland (da bleibt die Frage leider unpräzise), wobei das letzte Kriterium Büsingen und Helgoland ausschließt. --193.174.230.68 08:55, 20. Jan. 2016 (CET)
Danke IP. Also: mal so, mal so, je nachdem ob Weinbrand beigemischt ist oder nur Rotwein als Alkohollieferant eingesetzt wird. --Tastaturtest Belgisch (Diskussion) 08:58, 20. Jan. 2016 (CET)
Yup, genauso wie Rotkäppchen am Anfang sagte :-) -- southpark 09:08, 20. Jan. 2016 (CET)
Einfache Fragen zu beantworten, dauert immer länger.  ;) --Heletz (Diskussion) 13:50, 21. Jan. 2016 (CET)
Obwohl man 9 Minuten nicht wirklich als lange bezeichnen kann.--Expressis verbis (Diskussion) 15:39, 22. Jan. 2016 (CET)
Beim Fragesteller hat es immerhin 32 Minuten gebraucht, wie die jeweilige Erwiderung auf die erste und die zweite korrekte Antwort zeigen. --Rôtkæppchen₆₈ 16:57, 22. Jan. 2016 (CET)

Was heißt das genau? Bezieht sich das nur auf die Sprache (sächsisch)?

Wiktionary: Sassisch – Bedeutungserklärungen, Wortherkunft, Synonyme, Übersetzungen

--93.134.184.198 13:03, 20. Jan. 2016 (CET)

Hier lesen. Play It Again, SPAM (Diskussion) 13:35, 20. Jan. 2016 (CET)
siehe auch hier. PS: etymologisch könnte da ahd. sāzzo = Bewohner dahinterstecken, vgl etwa Artikel Elsass--in dubio Zweifel? 16:23, 20. Jan. 2016 (CET)
Wolke versteht das Wort „sassisch“ als Urform des Wortes „sächsisch“ [9] und deutet es in seinen Düdischen oder sassischen Singedichten etc. im Jahr 1816 als „ansässig“ [10]. Weder Wolkes Wortgeschichte noch seine Wortetymologie scheinen allgemein akzeptiert zu sein wie auch der Begriff selbst anscheinend wenig verbreitet ist. Vielleicht – jetzt spekuliere ich – suchte Wolke nach einem Begriff für Platt oder Niederdeutsch oder Niedersächsisch, der Volk und Sprache nicht abwertete (Platt und Nieder- haben abwertende Konnotation). --Pp.paul.4 (Diskussion) 00:44, 21. Jan. 2016 (CET)
Das Wort bezieht sich auf die Sprache, aber nicht - um das noch mal explizit zu sagen - auf das, was man heute landläufig unter "sächsisch" versteht, sondern ausschließlich auf die Niederdeutsche Sprache. Die Sachsen waren im Nordwesten Deutschlands Träger des Altsächsischen, worauf das heutige Westniederdeutsche zurückgeht. Sprecher des Mittelniederdeutschen waren an der Ostsiedlung beteiligt und haben so die Grundlage gelegt für das Ostniederdeutsche. Das Ostniederdeutsche ist Teil des Niederdeutschen, allerdings herrscht im ostniederdeutschen Sprachraum kaum Gefühl dafür, dass die Sprache auf den Stamm der Sachsen zurückgeht.
Die etymologische Ableitung von sāzzo ist falsch. Sassisch ist einfach die niederdeutsche Form von sächsisch (vgl. Voss: „Fuchs“, wassen: „wachsen“ etc.; Mittelniederdeutsches Wörterbuch zu Sasse (Schiller-Lübben)). Zur Etymologie des Wortes Sachsen steht etwas unter Sachsen (Volk)#Name.
Scheller und Wolke haben hier keinen neuen Begriff erfunden, sondern einfach das alte Endonym übernommen. Ein Endonym, das zu ihrer Zeit langsam ins Vergessen geriet, aber trotzdem noch verständlich war (zumindest die Form neddersassisch war allgemein bekannt). Ziel war dabei natürlich auch, einen Begriff zu verwenden, der die Sprache in ihr eigenes Recht setzt und sie nicht als Appendix ihrer großsüchtigen Schwester erscheinen lässt. --::Slomox:: >< 07:40, 21. Jan. 2016 (CET)
Und noch mal nachgereicht als Nachweis, dass das Wort tatsächlich so schon lange vor Scheller und Wolke verwendet worden ist: Luthers Bibel wurde im 16. Jahrhundert auch ins Niederdeutsche übersetzt und wurde unter anderem gedruckt als: Biblia. Dat ys: De gantze hillige Schrifft Sassisch. --::Slomox:: >< 07:54, 21. Jan. 2016 (CET)
Als Ergänzung: In mittelniederdeutscher Zeit gibt es auch Nachweise (neben dudesch) für "sassesch"/"sassisch" im ostniederdeutschen Kolonialgebiet, z. B. Nikolaus Gryse: "dat dorch Jochim Lowen in unse Sassische Sprake gedruckedes Bock" (Historia Van der Lere / Leuende vnd Dode M. Joachimi Slüters) oder "das Düdesche Sassesche wordt" (Wedewen Spegel). Gryse ist nach eigener Aussage "Anno 1543. an S. Catharinen dage [25. November] alhyr tho Rostock gebaren".--IP-Los (Diskussion) 18:03, 22. Jan. 2016 (CET)

Lied, dass ein Leben im Schnelldurchlauf erzählt

Es gibt ein Lied von den Ärzten oder Tote Hosen oder irgendeiner ähnlich bekannten deutschsprachigen Band, die von einem Mann erzählt und wie sein ganzes Leben durchläuft. Hat so ein leicht(?) melancholisches Ende, weil er ja stirbt und es auch so schnell vorbei ist. Hat jmd. eine Idee? (Jede Assoziation willkommen, wer weiß wieviele Fehler meine Beschreibung enthält :) )-- 141.30.146.64 03:09, 21. Jan. 2016 (CET)

Wel du es so offen formuliert hast: Mike Krüger: Mein Gott, Walther. --Hachinger62 (Diskussion) 07:47, 21. Jan. 2016 (CET)
http: // www.songtexte.com/songtext/die-fantastischen-vier/geboren-13da6d5d.html <= Fanta 4 - Geboren Play It Again, SPAM (Diskussion) 10:19, 21. Jan. 2016 (CET)
Immer dabei von Farin Urlaub? --slg (Diskussion) 20:23, 21. Jan. 2016 (CET)
Alles gute Vorschläge, aber leider nicht dabei. Mal sehen, ob mein Unterbewusstsein noch was ausgräbt. Es war wohl doch HipHop. Blumentopf? Youtube hat leider nur Schwarze-Rote Verbote für mich und kaum Tracks. -- 141.30.146.64 00:47, 23. Jan. 2016 (CET)

Die Grenzsteine des Hambacher Haagwaldes

ich möchte die Geschichte des Haagwaldes kurz erläutern, und anschließend eine Bilderliste der Grenzsteine einfügen.

--Fischer.H (Diskussion) 09:49, 22. Jan. 2016 (CET)

Das ist lobenswert. Aber was ist deine Frage? --j.budissin+/- 09:52, 22. Jan. 2016 (CET)
wie bringe ich die Bilderliste hinter die Geschichte? (nicht signierter Beitrag von Fischer.H (Diskussion | Beiträge) 10:04, 22. Jan. 2016 (CET))
Hilfe:Galerie, aber auch beachten: Wikipedia:Artikel illustrieren, besonders den Abschnitt Ziel: Sinnvolle Textergänzung. Wikipedia ist keine Plattform zum Erstellen von Bilderlisten. Bilder sollten immer dem besseren Verständnis des Textes dienen. 217.230.88.120 10:09, 22. Jan. 2016 (CET)
Oder als informative Tabelle etwa wie in Liste der Rittersteine. Es werden aber Einwände kommen, ob der Hambacher Haagwald und dessen Grenzsteine enzyklopädisch relevant sind. Bereite es daher in deinem Benutzernamensraum vor und/oder mache einen Wikipedia:Relevanzcheck. --Pp.paul.4 (Diskussion) 14:10, 22. Jan. 2016 (CET)
@Pp.paul.4: Warum sollte der Hambacher Haagwald nicht relevant sein? Was die Grenzsteine angeht: siehe da. --j.budissin+/- 15:41, 22. Jan. 2016 (CET)
Zum Beispiel weil er 0 Google-Treffer und 0 Google-Buch-Treffer hat, bei Google-Scholar habe ich ihn nicht gesucht. --Pp.paul.4 (Diskussion) 15:45, 22. Jan. 2016 (CET)
Gut gesehen. GROSSARTIG! Jetzt sind wir quitt! :-)))) Play It Again, SPAM (Diskussion) 16:20, 22. Jan. 2016 (CET)

Bilderliste!

Wie erstelle ich eine Bilderliste, z.b. von den Grenzsteinen einer Gemarkung?

--Fischer.H (Diskussion) 17:06, 22. Jan. 2016 (CET)

Was passt Dir denn an den Antworten weiter oben nicht? -- Gerd (Diskussion) 17:34, 22. Jan. 2016 (CET)

Startseite (Firefox, WINDOWS 10)

Ich möchte meine e-mail-Seite (arcor) als Startseite haben. Wie? Gruß -- Dr.cueppers - Disk. 11:36, 22. Jan. 2016 (CET)

about:preferences in die Adressleiste einfügen und unter "Startseite" die Startseite festlegen. --Komischn (Diskussion) 11:48, 22. Jan. 2016 (CET)
Oder so: Im Menü, das ist das Symbol mit den drei parallelen Linien ganz rechts oben, Einstellungen, Allgemein ganz oben. -- Gerd (Diskussion) 11:51, 22. Jan. 2016 (CET)
Hat funktioniert; Dank und Gruß -- Dr.cueppers - Disk. 14:01, 22. Jan. 2016 (CET)
Jetzt habe ich aber ein nerviges Schild und muss das immer ausfüllen, wie kriege ich das weg? 15:33, 22. Jan. 2016 (CET)

 

Offenbar hast du dir eine unschöne Toolbar eingefangen. Gehe ins Menü "Add-ons" und entferne EasyPDFDingens. Grüße Dumbox (Diskussion) 15:42, 22. Jan. 2016 (CET)
ich habe mir ask eingefangen; das hätte ich auch geerne wieder weg. Und: Bei meinen Add-ons gibt e kein EasyPDF! Gruß -- Dr.cueppers - Disk. 17:04, 22. Jan. 2016 (CET)
Vielleicht hilft: Rechtsklick auf das Startsymbol, dann Programme und Features und dort dann unerwünschtes deinstallieren. Vorher alle Programme schließen. --Rôtkæppchen₆₈ 17:33, 22. Jan. 2016 (CET)
Diese Ungeziefer installieren sich als Programme. Gehe auf Programme und Funktionen,(Win7, bei anderen analog) suche ask.com und deinstalliere das und den anderen nicht gewünschten PDF-Schrott auch. Und das nächste Mal beim Java-Update das Häkchen Ask entfernen, denn das ist der Wirt. --2003:75:AF11:E900:9C90:2406:3042:7A3F 17:43, 22. Jan. 2016 (CET)
ask ist hartnäckiger. Deinstallieren und anschließend das Prog drüber hilft. 78.51.118.239 18:14, 22. Jan. 2016 (CET)
HIer noch ein paar tips:http://www.browserdoktor.de/ask-toolbar-entfernen/ gruß 217.251.205.123 22:16, 22. Jan. 2016 (CET)

Fahrzeugpanne - Mietwagen über Schutzbrief

Nach einer Fahrzeugpanne würde mir meine Autoversicherung (Schutzbrief) einen Mietwagen bezahlen, bis das Auto repariert ist. Die Reparatur wäre frühestens am Dienstag fertig, sie wurde noch nicht begonnen. Es müssen erst Teile bestellt werden und der Meister war heute beschäftigt.

Allerdings überlege ich jetzt, ob ich das Auto überhaupt noch reparieren lassen soll. Die akut notwendige Reparatur zusammen mit einigen anderen sinnvollen Arbeiten wäre etwa so teuer wie der Restwert des Autos.

In den Versicherungsbedingungen habe ich von einer Schadensminderungspflicht gelesen, aber nichts von einer "Reparaturpflicht" o.ä. Wenn ich mich nun entschließen sollte, das Auto an einen Verwerter zu verkaufen statt es zu reparieren, kann die Versicherung dann die Übernahme der Kosten für den Mietwagen verweigern?

Konkret heißt es Wenn das Fahrzeug [...] nicht fahrbereit ist [...], helfen wir ihnen, ein gleichwertiges Fahrzeug anzumieten. Voraussetzung ist, dass das Fahrzeug weder am Schadentag noch am darauf folgenden Tag wieder fahrbereit gemacht werden kann. Wir übernehmen die Kosten des Mietwagens bis [...] Euro. [...] Sobald Ihnen Ihr Fahrzeug wieder fahrbereit zur Verfügung steht, endet Ihr Anspruch auf Kostenerstattung. Ich finde da keine Einschränkung, dass es zwangsläufig repariert werden muss, wenn die Analyse in der Werkstatt lange dauert und dann hohe Kosten ergibt.

Und was wäre, wenn die Werkstatt ein Original-Ersatzteil vorrätig hätte, ein günstigeres Gebrauchtes aber erst bestellen müsste? Könnte das auch Probleme mit der Versicherung geben?

Vielleicht gibt es auf diese Fragen keine uneingeschränkt gültigen Antworten. Aber wenn hier jemand vom Fach ist, bin ich um Tipps dankbar. Die Versicherung möchte ich nicht ohne Not kontaktieren.--89.15.238.110 22:33, 22. Jan. 2016 (CET)

„am Festland“ (erl.)

In Alexanderarchipel#Bevölkerung lesen wir: „Die größte Stadt in der Region und Hauptstadt Alaskas, Juneau, liegt am Festland.“ Sollte es in diesem Zusammenhang nicht „auf dem Festland“ heißen? Die Zusammenziehung von „auf dem“ zu „am“ kenne ich nur als Bavarismus/Austriazismus, aber ein Österreich- oder Bayernbezug (in dem Falle würde ich so etwas nicht beanstanden) besteht bei einer nordamerikanischen Inselgruppe ja nicht. --Florian Blaschke (Diskussion) 17:10, 22. Jan. 2016 (CET)

+1 "Husum, graue Stadt am Meer" liegt auf dem Festland. "Am Festland" wäre also im Wasser. --Optimum (Diskussion) 17:31, 22. Jan. 2016 (CET)
In „am Boden“ ist es idiomatisch auch im Standarddeutsch. --Chricho ¹ ² ³ 19:19, 22. Jan. 2016 (CET)
"Juneau liegt am Boden"? --Eike (Diskussion) 21:02, 22. Jan. 2016 (CET)
Das klingt wie schlecht aus dem Englischen übersetzt? --Pölkkyposkisolisti 21:08, 22. Jan. 2016 (CET)
@Eike Nein, eher Sätze wie: "Er liegt am Boden." oder übertragen "Er ist am Boden." "am Boden zerstört sein". Ich kenne auch noch "an der Erde" - neben "auf der Erde": Das Tuch liegt an der Erde, heb es auf!; so auch niederdeutsch: Dat Dauk liggt anne Ierd', böhr dat up!--IP-Los (Diskussion) 00:25, 23. Jan. 2016 (CET)
Das passt aber doch gerade zu dem, was Florian gesagt hat: "Am Festland" ist (direkt) neben dem Festland - das Wasser. "Am Boden" ist (direkt) neben dem Boden. Wer "am Boden ist", ist nicht zum Boden geworden, was am Festland ist, ist nicht Teil des Festlands. --Eike (Diskussion) 12:40, 23. Jan. 2016 (CET)
+1, "am" ist ja eine Kurzform von "an dem". Weiterhin dient die Wahl des Begriffs "Festland" ja der Unterscheidung zu einem Ort,der eben nicht auf dem Festland ist - auf See also, auf einem Schiff, auf dem Eis oder auf einer Insel usw. Die Benutzung von "auf" unterstreicht hier, dass vor allem der Untergrund und seine Beschaffenheit (hier nicht zuletzt auch seine Ausmaße) von Bedeutung sind. "Auf einer Eisscholle" oder "auf einer Insel" kündet von frühen Begrenzungen des Aktionsradius, "auf dünnem Eis" betont das Risiko und "auf dem Festland" läßt etwas Beruhigendes mitschwingen. Der Kollege Blaschke liegt mit seiner Vermutung also richtig, besser: er befindet sich auf dem richtigen Weg bzw. auf der richtigen Spur und nicht auf dem gefürchteten Holzweg. --2003:45:4656:300:4081:24D3:529F:76F0 00:39, 23. Jan. 2016 (CET)
"Am Boden" bedeutet ja, dass der Boden von außen berührt wird, das ist also nicht "im Boden". Äquivalent würde "am Festland" eine Berührung von außerhalb bedeuten. - "An der Erde" halte ich für eine falsche Übernahme/Übersetzung, hauptsächlich von nativen Sprechern des Niederdeutschen.--Optimum (Diskussion) 01:02, 23. Jan. 2016 (CET)
Hier dürfte wohl nichts weiter als österreichische Ausdrucksweise vorliegen. Mangels Bezugs zu Österreich kann das aufgrund der Missverständlichkeit im größten Teil des deutschen Sprachraums korrigiert werden. Diese Diskussion zeigt ja, wohin eine solche Ausdrucksweise führt. MBxd1 (Diskussion) 12:31, 23. Jan. 2016 (CET)
Ich war mal so mutig. --Jossi (Diskussion) 13:51, 23. Jan. 2016 (CET)
Nein Eike, mit "am Boden" meinte ich "auf dem Boden", denn das ist mit der Berührung des Bodens ja gemeint. Wenn jemand am Boden liegt, liegt er nicht neben, sondern auf dem Boden. Das Tuch liegt am Boden = Das Tuch liegt auf Boden. Im Bairischen Sprachgebiet ist der Gebrauch noch sehr viel ausgedehnter, z. B. "am Tisch liegen" = "auf dem Tisch liegen". In diese Kategorie dürfte auch "am Festland" fallen wie Florian eingangs schon schrieb. "an der Erde" ist deshalb nicht genuin niederdeutsch, sondern sollte nur verdeutlichen, daß diese Konstruktion selbst noch im Norden gebräuchlich ist, vgl. mittelhochdeutsch "in deme himile unde an der erden" oder "an dirre welt auf dieser welt". Adelung bemerkt z. B. "In manchen Fällen vertritt es in Oberdeutschland auch die Stelle der Präposition auf." Das Goethe-Wb. schreibt: "auch für die Berührung von oben her: auf  Sie findet Ottilien an der Erde 20,363,20 Wv II 14  am Boden AA146,27 Werth II  da sie allein sas am [auf dem AA137,28 Werth II Ossian] [on the] Hügel AAJw3,60,7 GesängeSelma  am Lande 12,100,19 Fischerin  an [about] der Küste" Vgl. dazu auch die zahlreichen Nachweise im Grimm, wo eben jenes "auf" gemeint ist: "er liegt am boden" "wann man zu vil bonen iszt und am rucken ligt." Während für mich "am Festland auch eher etwas wie "neben dem Festland" impliziert, kenne ich aber eben "am Boden" und "an der Erde", ganz so wie Chricho geschrieben hat.--IP-Los (Diskussion) 00:52, 24. Jan. 2016 (CET)
Wie gesagt, die Kontraktion „auf dem“ → „am“ ist charakteristisch für den Südosten, was wohl daran liegt, daß „auf“ im Bairischen (Ostoberdeutschen) als „aff“ erscheint („au“ wird bair. vor Labialen allgemein zu „aa“ monophthongiert, aber auch in „auch“). Deshalb verfiel ich auf den Gedanken, daß dies der Hintergrund sein könnte; ich war mir aber unsicher, welche Präposition in diesem Zusammenhang üblich ist, daher die Nachfrage.
Dank anJossi2. --Florian Blaschke (Diskussion) 13:17, 24. Jan. 2016 (CET)
@IP-Los: Ich verstehe jetzt, was ihr meint: Die Stadt liegt am Festland - vertikal (angrenzend) daneben. Ich glaube, mir kommt das im Gegensatz zu "am Boden" komisch vor, weil bei Letzterem das Daneben nur vertikal sein kann (der Boden ist grenzenlos), während beim Festland meines Erachtens das naheliegende Daneben ein horizontales ist. Wenn ich "liegt neben dem Schrank" (oder auch "liegt am Schrank") sage, meine ich nie, das etwas dadrauf liegt - da würde ich "auf dem Schrank" sagen. --Eike (Diskussion) 10:22, 25. Jan. 2016 (CET)
Archivierung dieses Abschnittes wurde gewünscht von: Florian Blaschke (Diskussion) 13:17, 24. Jan. 2016 (CET)

Betriebsruhe auf Bahnstrecken??

Es gibt wohl etliche Nebenstrecken wo Betriebsruhe herrscht abends und teilweise Sonntag früh morgens weil kein Zug kommt, ich bin Hobbyfotograf und wollte mit 2 Mädels dort Bilder machen auf einem Bahnhof (Lauterbach Nord), in Hessen, jetzt hat die Bahn mir das untersagt! Gibt es dafür einen Grund es ist doch eh Betriebsruhe also darf ich mich dort doch aufhalten was soll da passieren?--GEdacht und gebaut (Diskussion) 14:26, 22. Jan. 2016 (CET)

Es fahren auch immer wieder mal außerplanmäßig Züge über Strecken, selbst wenn da sonst nichts fährt. Das Betreten und Überqueren der Gleise ist eine Ordnungswidrigkeit nach § 62 iVm. § 64b Eisenbahnbetriebsordnung. Das kostet dann etwas Geld. Aber darum geht es mir gar nicht. Entscheidend ist was Anderes: Gleise zu betreten ist schlicht und einfach lebensgefährlich. --87.123.42.156 14:33, 22. Jan. 2016 (CET)
(BK) Bedeutet "Betriebsruhe" auch, dass garantiert keine Sonderzüge, Güterzüge oder (eventuell massiv) verspäteten Züge kommen? Manchmal kommt es auch zu mehrstündigen Verspätungen, ich selbst war mal in einem ICE, der >3 Stunden gestanden ist. Grund: es hat sich jemand überfahren lassen und das ganze war in der "Pampa", der Lokführer wird nach solchen Zwischenfällen immer ausgetauscht und allein das Taxi, mit der der Ersatz gebracht wurde, hat 2 Stunden gebraucht. Im Zusammenhang mit winterlichen Verhältnissen aber auch schon wegen Hitze (Zug durfte wegen Klimaanlagentotalausfall und 50°C im Innenraum nicht mehr weiterfahren) gabs laut Medienberichten auch schon sehr viel längere Verspätungen bei der DB... --MrBurns (Diskussion) 14:39, 22. Jan. 2016 (CET)
Es KANN KEIN ZUG FAHREN. Weil die Stellwerke und die Schrankenwärterposten nicht besetzt sind also kann keiner die Signale auf grün stellen folglich kann kein Zug fahren. Warum blockt die Bundesbahn dann so?--14:48, 22. Jan. 2016 (CET)
Warum blockt die Bundesbahn dann so? Das musst du die Bundesbahn fragen. --Engie 14:52, 22. Jan. 2016 (CET)
Ich will ja die Bundesbahn verstehen, und wissen warum sie so blockt, Fragen werden bei der Behörde ja nicht beantwortet.-14:54, 22. Jan. 2016 (CET)
Es ist für jede Behörde jederzeit einfacher, nein zu sagen. Kostet nix, bringt keinen Ärger. Für ein Ja brauchst du Argumente, Beziehungen, Charme... 89.12.27.66 15:02, 22. Jan. 2016 (CET)
Es gibt auch Strecken, die sind stillgelegt und mehr oder weniger zurückgebaut. Oft sind die dann zugänglich, und teilweise genauso romantisch wie aktive Strecken. --213.98.84.12 15:03, 22. Jan. 2016 (CET)
Mit dem Rückbau stimmt, aber auf der anderen Seite dort ist alles zugewachsen ist soll so viel Realität wie möglich dargestellt werden. Daher muss es eine Strecke mit aktiven Betrieb sein zur Betriebsruhe so verlangt es der Auftraggeber.--15:07, 22. Jan. 2016 (CET)
Wenn es einen Auftraggeber gibt, dann kann man auch eine ganz offizielle Anfrage an die Bahn stellen, dann wird die Strecke eben mal für eine Weile gesperrt und das kostet dann noch ein paar Euronen und dann wars das auch schon. Außerdem gibt es noch eine ganze Reihe nichtbundeseigener Eisenbahnen, die machen das vielleicht mal schneller. Und beachte bitte das schlechte Vorbild, das du in solchen Fällen abgibst. --GeorgDerReisende (Diskussion) 15:14, 22. Jan. 2016 (CET)
Überleg dir einfach, was sie zu gewinnen und was sie zu verlieren haben. --Eike (Diskussion) 15:17, 22. Jan. 2016 (CET)
  • Man betritt kein in Betrieb befindliche Gleise, als Laie sowiso nicht. Wenn das Gleis nicht stillgelegt ist, muss es gespert sein damit Personen ohne Sicherheitsprüffung es betreten dürfen. Gerade auf Strecken mit Betriebsruhe wird in der Zeit gerne mal das Gleis unterhalten. Und dann wird halt dafür das Stellwerk usw. ausserplanmässig besetzt. kurzum du musst immer davon ausgehen, dass ein Zug Fahren kann, es sei denn du hast dich -vor Ort- vom Gegenteil übezeugt.
Und eine Bewilligung erhälst du auch nur, wenn du die Spielregeln kennst und erwähnst. Also das erste Schreiben sollte als das Ziel nicht das Erreichen einer Bewillgung haben, sondern sollte nur ausloten was zum erhalten der Bewilligung notwendig ist.--Bobo11 (Diskussion) 15:17, 22. Jan. 2016 (CET)
Dann erkläre mir mal bitte welche Zug Sonntag Morgens um 7:00 Uhr fahren soll???? Das Stellwerk ist nicht besetzt das erkennt man daran ob das Auto des Fdl vor dem Gebäude steht.--15:27, 22. Jan. 2016 (CET)
Denkfehler, die Betriebsruhe und die Anwesenheit des FDL bezieht sich auf den rollenden, den fahrplanmäßigen Verkehr der allein während der Betriebsruhe eingestellt ist, die Schiene hat damit keine Betriebsruhe.--2003:75:AF11:E900:C500:881F:9639:5F11 15:52, 22. Jan. 2016 (CET)
Ja, es gibt etliche Strecken wo auch bei unbesetzten Stellwerken befahren werden können (Stellwerk durchschalten nennt sich das). Dann ist der Zug aber eben der einzige der sich auf der gesamten Strecke zwischen den bedienten Stationen befinden darf. Oder wenn es eine Stichstrecke ist eben ab der Einmündung. Oder auch anders Rum. Auf einer für den Baudiest gesperten Strecke dürfen Ragierfahrten ausgeführt werden (auch über dazwischen liegende Bahnhöfe). Dann läst man eben den Bauzug rein, stellt als FDL die Sperrtaflen usw. auf und geht in denn Feierabend. Der Frühdienst hebt die Sperre auf und lässt den Bauzug wieder raus. --Bobo11 (Diskussion) 16:07, 22. Jan. 2016 (CET)
Wenn du die offizielle Seite nicht zur Kenntnis nehmen willst, musst du eben auf die andere – illegale – wechseln. Dann bist du allerdings selbst für etwaige Konsequenzen verantwortlich. Mehr gibt es dazu eigentlich nicht zu sagen. --j.budissin+/- 15:38, 22. Jan. 2016 (CET)
Du sagst, du hast die Bahn gefragt. Sowas wird ziemlich zentral und weit "oben" entschieden und hat eine längeren bürokratischen Vorlauf. Ich glaube nicht, dass du das richtig eingetütet hast. Die Bahn muss sicherstellen, dass nichts passieren kann und keine Regressansprüche aufkommen. Sie muss durch eine totale Sperrung über die Betriebsruhe hinaus sicherstellen, dass tatsächlich kein Zug fährt. Bei Umleitungen z.B. können auch solche Strecken kurzfristig befahren werden. Um die Bahn zu verstehen stell dir vor, das wäre dein eigenes Gelände, das erste wäre, dass du eine Verzichtserklärung gegen jegliche Forderungen und einen Haftungsausschluss unterschreiben lässt und der Fotograf für die Kosten aufkommt, die dir durch Sicherungsmaßnahmen entstehen. Die Bahn macht da sicher mit, nur geht das über die Kompetenz und Verantwortungsbereich eines Bahnhofvorstehers hinaus. Mit Geduld geht das sicher, ist aber sicher auch nicht kostenlos.--2003:75:AF11:E900:C500:881F:9639:5F11 15:40, 22. Jan. 2016 (CET)
Es gibt zum Beispiel diese Gleismesswagen, die eher in der Zeit fahren, in der kein normaler Zugverkehr stattfindet. Möglicherweise auch Triebwagen, die zum neuen Einsatzort fahren. Bei Bautrupps am Gleis gibt es immer ein oder zwei Leute, die nicht mitarbeiten, sondern nur gucken, ob ein Zug kommt. Das gilt meines Wissens auch nachts. --Expressis verbis (Diskussion) 16:11, 22. Jan. 2016 (CET)
Ja, genau da liegt ja der Hacken. Ist das Gleis nicht gesperrt, ist immer mit einem Zug zu rechnen. Und selbst in gesperten Gleisen können, -wenn so angeordnet- Ragierbewegungen stattfinden. Es gilt da eben, solange du dich nicht vom Gegenteil überzeugt hast, kann immer ein Zug kommen. Und das „du dich nicht vom Gegenteil überzeugt hast“, ist schon für einen Bahnmitarbeiter nicht einfach so möglich. Für einen Laien ist es schlichtweg unmöglich. Da dieser selber keine Gleissperre bestätigen lassen kann, und den Gleisabschnitt auch nicht -wenn notwendig- regelkonform mit Sperrtafel absichern kann. Bei der SBB kannst du das eigentlich vergessen, dass du als Laie ohne Sicherheitswärter -oder zumindest Einweisung durch einen Fachperson- in Gleis darfst (und es würde mich erstaunen wenn das bei der DB anderes wäre). Auf Bewilligung sind Sicherheitswärter die Regel, oder das Gleis kann so gesichert werden, dass man "nur" eine Freigabe benötigt. Aber auch da wird bei Beginn und Ende ein Bahn-Mitarbeiter auf Platz sein, der dir erlaubt ab jetzt das Gleis zu betretten, udn am Schluss eban nachschaut ob das Gleis wieder frei udn Fahrbar ist. Und weil da so gut wie immer Personal auf Platz sein muss, kann das endsprechend teuer werden. Wie oben schon geschrieben, es kommt auch darauf an wie gut der Antragsteller die Sicherheistregeln und Abläufe kennt, und welche Sicherheitsvorkehrung er selber treffen kann. Darauf baut auch eine Bewillgung auf, neben den lokalen Gegebenheiten natürlich (Kann überhaupt eine -für Laien- unmissverständliche Bewilligung ausgestellt werden?). --Bobo11 (Diskussion) 17:32, 22. Jan. 2016 (CET)
Es gibt einzelnen Fälle, in denen man wohl die Augen zudrück, solange die Leute nicht quer über die Gleise latschen, in der Nähe meines Stellwerks gibt's da so einen Punkt. Ansonsten wurde das Wesentliche schon gesagt. Unbefugte dürfen nicht ohne Weiteres den Gleisbereich betreten, entweder gibt's vorher eine Einweisung oder eben eine Berechtigte Person an die Seite. Formal ist das Gleis in Betrieb, ergo müsste man dich vor Gefahren aus dem Schienenverkehr schützen, hieße das Gleis sperren. Das wiederum ist Aufgabe des Fahrdienstleiters, der ja nicht anwesend ist. Wird also nichts. Da macht man auch keinen Unterschied, ob nun ein Zug kommen könnte oder nicht. Ruhezeiten sind zudem nicht in Stein gemeißelt, wenn Arbeiten anstehen kann es durchaus vorkommen, dass der Fdl eine Extraschicht einlegen darf. -- Platte ∪∩∨∃∪ 18:11, 22. Jan. 2016 (CET)

Angenommen, die Bahn erteilt eine Ausnahmegenehmigung zum Betreten der Gleisanlagen: Wenn nun kurzfristig die Strecke - aus welchem Grund auch immer - doch befahren werden muss, müsste sie wissen, wer sich den potenziell auf den Gleisen aufhalten könnte, die Ausnahmegenehmigung widerrufen und hoffen, dass alle erreicht werden und sich daran halten. Da ist es doch viel einfacher, grundsätzlich und ausnahmslos das Betreten zu untersagen. Ein Einzelfällen sind Ausnahmen sicherlich nach Absprache möglich, aber dann bestimmt nicht kostenlos und nur gut begründet.--Vertigo Man-iac (Diskussion) 16:53, 22. Jan. 2016 (CET)

Such dir eine dieser Strecken wo Privatleute auf Loren ran dürfen oder nur Oldtimer unterwegs sind. Da haste eher die Chance, dass jemand mal den Verkehr anhält damit du deine Fotos machen kannste oder ein bisschen gerade nicht benutztes Gleis für eine Zeit übrig hat. --87.148.92.241 17:36, 22. Jan. 2016 (CET)
Und dann will die Bahn sicher wissen, was du da ablichtest. Stell dir das Machwerk mit den 4 Buchstaben in großen Lettern vor:"Porno auf den Gleisen statt pünktlicher ICE." Auch darauf müssen die achten.--2003:75:AF11:E900:9C90:2406:3042:7A3F 18:53, 22. Jan. 2016 (CET)
Das fiel mir auch gerade auf, als ich von den „2 Mädels“ las. Sollten die „Mädels“ dabei mehr oder weniger nackig gewesen sein, möchte die Bahn damit vielleicht auch nicht gerne in Verbindung gebracht werden. --Jossi (Diskussion) 13:49, 23. Jan. 2016 (CET)

Und auf den Gleisen fährt nicht nur die DB, im Güterverkehr gibts angeblich an die 400 Bahnunternehmen, die die Strecken buchen... z.B. https://www.youtube.com/watch?v=oZ3MGgk0Jk4 --Btr 22:14, 22. Jan. 2016 (CET)

Rail 4U --Hans Haase (有问题吗) 00:26, 24. Jan. 2016 (CET)

was ist Stress mit 5 buchstaben?

--91.34.244.10 19:07, 22. Jan. 2016 (CET)

Die Schreibweise in der alten Rechtschreibung (Streß), jedenfalls laut dem. --MrBurns (Diskussion) 19:12, 22. Jan. 2016 (CET)
Jedenfalls schreibt man in der Schweiz "Stress", auch in der alten Rechtschreibung.--Skyscraper1996 (Diskussion) 19:21, 22. Jan. 2016 (CET)
Ärger? --Heimschützenzentrum (?) 19:39, 22. Jan. 2016 (CET)
Krach, Stunk Play It Again, SPAM (Diskussion) 19:49, 22. Jan. 2016 (CET)
Frust, will man Stress negativ sehen. --92.196.121.121 23:31, 22. Jan. 2016 (CET)
Strass mit einem Tippfehler? --2003:45:4656:300:4081:24D3:529F:76F0 00:20, 23. Jan. 2016 (CET)
Schon mal in die Onlineenzyklopädie mit 9 Buchstaben geguckt? Unter Stress? --MannMaus 00:24, 23. Jan. 2016 (CET)
Schau lieber in das Onlinewörterbuch desselben Herausgebers: wikt:Stress liefert zwei Synonyme mit fünf Buchstaben. --Rôtkæppchen₆₈ 00:34, 23. Jan. 2016 (CET)
Da, wo man öfter mal Wörter mit einer bestimmten Anzahl Buchstaben sucht, hat aber
A E R G E R
1 2 3 4 5 6
normalerweise sechs Buchstaben. Außerdem stand es oben schon. Trotzdem kann man in solchen Situationen natürlich gerne in dieses Onlinewörterbuch gucken. --MannMaus 14:59, 23. Jan. 2016 (CET)
Für diese Situationen gibt es spezielle Nachschlagewerke, auch online. --Rôtkæppchen₆₈ 16:04, 23. Jan. 2016 (CET)
Das habe ich jetzt mit unserem Beispiel ausprobiert. :) --MannMaus 17:15, 23. Jan. 2016 (CET)

Bleierne Zeit

Wofür steht heutzutage "bleierne Zeit", was bedeutet eine "bleierne Zeit", was ist eine "bleierne Zeit". Wofür und wann wird diese Metapher verwendet? Gemeint ist jetzt nicht nur der Film Die bleierne Zeit oder die RAF-Episode. --31.17.105.147 00:18, 23. Jan. 2016 (CET)

Dieser GBS-Treffer, 108 Jahre vor der RAF, macht klar, dass es den Begriff vorher schon gegeben haben muss. --Rôtkæppchen₆₈ 00:44, 23. Jan. 2016 (CET) Hier ein literarischer Treffer desselben Jahres. --Rôtkæppchen₆₈ 00:48, 23. Jan. 2016 (CET)
Ist das nicht selbsterklärend? Bleiern = schwer wie Blei. "Bleierne Füße" kann man körperlich nachempfinden, "bleierne Müdigkeit" dann ins Ungegenständliche übertragen. "Bleierne Zeit" "fließt nicht dahin" oder "vergeht im Flug", sondern kriecht nur schwerfällig voran. --Optimum (Diskussion) 01:09, 23. Jan. 2016 (CET)
Es gab im Humanismus und später noch den Topos, die antiken Weltalter fortzuschreiben, wobei nach der eisernen Zeit die bleierne aetas plumbea kam (auch metaphorisch, etwa zur als Verfall angesehenen Entwicklung der lateinischen Sprache im Mittelalter). Grüße Dumbox (Diskussion) 08:52, 23. Jan. 2016 (CET)
Genau.
Also methaphorisch "schwer(fällig)", "arm", "unspektakulär", "stumpf" (=unscharf)(plumbum, plump (stimmt das?) Play It Again, SPAM (Diskussion) 09:56, 23. Jan. 2016 (CET)

Der Gang aufs Land. An Landauer#Vision. Salomo Glassius 1657 „Diese hundert Jahre sind zu achten [...] vor eine bleyerne Zeit wegen der Unfruchtbarkeit zu allen Guten und überquellenden Heßlichkeit alles Bösen.“ Und Beutelschneider 1641 „Wir leben/ die warheit zu sagen/ in einer eysernen ja bleyernen Zeit / oder vielmehr in einer unbarmherzigen Tyrannischen Blutzeite [...]“. Beides doch eher „grausam“ und „böse“ als bloß "schwer(fällig)", "arm", "unspektakulär", "stumpf". Plump ist Lautmalerei und hat nichts mit lat. plumbum zu tun, Grimm Wörterbuch und http://zwei.dwds.de/wb/plump. --Vsop (Diskussion) 11:00, 23. Jan. 2016 (CET)

Bin mir nicht sicher - lasse es offen.
Bleibe bei den Metaphern. Gold ist "glänzend", "strahlend", "wertvoll"; Eisen ist "hart", "kalt", "unbeugsam"; Blei ist vor allem "schwer" (Doppelbedeutung), "dumpf" (in der Farbe). Kann man andere Stellen finden, in denen Blei/bleyern als "grausam" und "böse" metaphorisiert wird? Bei "grausam/böse" fällt mir "Blut" ein, aber kein chemisches Element. Play It Again, SPAM (Diskussion) 11:24, 23. Jan. 2016 (CET)
Weit hergeholt: "Blei" wird auch im Zusammenhang mit Schusswaffen verwendet. --RobTorgel 11:40, 23. Jan. 2016 (CET)
>>>> Nein, durchaus nah, nämlich in Die bleierne Zeit#Der Titel. --Vsop (Diskussion) 12:52, 23. Jan. 2016 (CET)
Naheliegend ist wohl Blei als Material für Falschmünzen. Das Bild spielt etwa in der Scholastik eine Rolle, wenn es um den Wert des Menschen und seiner Taten für Gott geht: intrinsischer (geringer) Wert gegen per Vertrag auferlegten (hohen) Wert. Grüße Dumbox (Diskussion) 12:00, 23. Jan. 2016 (CET)
Noch ein Hinweis: Blei wird in der Astrologie dem Planeten Saturn zugeordnet. Zitat aus unserem Artikel Saturn (Mythologie): In der mittelalterlichen Astrologie stand Saturn [...] für Unglück: Sorgen, Melancholie, Krankheiten und harte Arbeit, jedoch auch für Ordnung und Maß. In der Antike war das noch anders. Da galt Saturn als Gott des Ackerbaus und ausgerechnet als Herr des "Goldenen Zeitalters". Geoz (Diskussion) 12:53, 23. Jan. 2016 (CET) P.S. @ SPAM: Manche denken bei grausam/böse an das chemische Element Schwefel. Geoz (Diskussion) 14:22, 23. Jan. 2016 (CET)
Schwefel ist - menschbezogen - in erster Linie einmal essenziell: Auch der Papst enthält etwa 150 g Schwefel! So'n Pech! Vielleicht kommt das böse/grausam ja auch von der (relevanten) Bleizeit: Unter dem, was so alles gedruckt wurde, ist auch viel Böses und Grausames. Ich erweitere metaphorisch auf schwer/bedrückend/erdrückend/belastend. Mit bleiernen Lidern Play It Again, SPAM (Diskussion) 15:14, 23. Jan. 2016 (CET)
Man sollte mal Sixtus IV. exhumieren lassen. Vielleicht enthält der 300 oder 600 g Schwefel? Geoz (Diskussion) 16:22, 23. Jan. 2016 (CET)
Der Schwefel wurde von den Destruenten längst verstoffwechselt. --Rôtkæppchen₆₈ 18:18, 23. Jan. 2016 (CET)

Parkettkleber und Kleinkinder

Unser Bauträger möchte den Kleber Pallmann P5 für Fertigparkett in verschiedenen Räumen einzusetzen. In den Räumen werden sich täglich auch mehrere Kleinkinder.

Soweit wir sehen, ist Pallmann P5 ein Silanmodifizierter Polymerklebstoff. Er ist laut Datenblatt "EMICODE EC 1 R PLUS/Sehr emissionsarm" und "GISCODE RS10" zertifiziert und erscheint uns daher recht unbedenklich.

Allerdings weist das Bundesministerium für Umwelt, Naturschutz, Bau und Reaktorsicherheit darauf hin, dass Silanmodifizierte Polymerklebstoffe teilweise auch Weichmacher enthalten. ( http://www.wecobis.de/bauproduktgruppen/klebstoffe/silanmodifizierte-polymerklebstoffe.html)

Leider habe ich auch nach Recherche nicht herausfinden könnnen, ob in Pallmann P5 nun Weichmacher enthalten sind oder nicht,- und wenn ja, welche und in welchen Mengen. Wikipedia meint, dass laut EU gesundheitlich bedenkliche Weichmacher DEHP, DBP und BBP sind, bei Bisphenol A teilweise gesundheitsgefährdend angesehen.

Kennt sich hiermit jemand aus? Ich bitte um begründete Einschätzung dieses Produkt. Bitte nur begründet einen konkreten anderen Kleber stattdessen empfehlen. Erstens ist bei Änderungswünschen mit erheblichen Mehrkosten zu rechnen und zweitens stellt sich ggf. dann dieselbe Frage. --85.177.16.44 10:23, 23. Jan. 2016 (CET)

Die exakte Zusammensetzung kennt vermutlich nur der Hersteller, insofern wird hier keiner helfen können - schon gar nicht mit belastbaren Aussagen. Generell: Wenn Du Fertigparkett verwendest (mit welchem die Kinder viel eher als mit dem darunter befindlichen Kleber in Kontakt kommen) dann würde ich erst mal über die Zusammensetzung des Parketts nachdenken (was ist da an Klebern, Lösungsmitteln etc. drin), vermutlich wird der komplette Boden nach Verlegung noch "Eingelassen" (Lasiert etc.) - was wird dazu verwendet? Und wenn Du preissensitiv bist - klar sind emmissionsarme Baustoffe teurer! Unbehandelte, gehobelte Eichenbohlen, festgenagelt, sind dagegen emmissionsfrei.--Mabschaaf 10:42, 23. Jan. 2016 (CET)
Im Prinzip hast Du ja recht, ich störe mich lediglich an den Eichenbohlen. Da hat noch keiner untersucht, ob die wirklich keine Wirkung auf den Menschen haben. Mir ist ein Schreiner bekannt, der regelmäßig nur (ausschließlich) beim Sägen von Eichenholz Asthmaanfälle bekommt und nicht umsonst nimmt man Eichenholz für Wein- und Wiskeyfässer um einen besonderen Geschmack aus dem Holz zu saugen. --2003:75:AF37:7900:F173:5944:E542:48F6 11:02, 23. Jan. 2016 (CET)
Wie Vorredner. Schätze die Belastung durch Weichmacher aus anderen Haushaltsgegenständen, die Kinder gerne mal in den Mund nehmen, als höher ein.
Auf dieser Seite kannst du dir links das zweiseitige Data Sheet des Klebers herunterladen. Es gibt keine detaillierten Inhaltsstoffe an, weist aber auf Vorsichtsmassnahmen bei der Verarbeitung hin.
Hinterher halt gut lüften ... sich vorstellen, was die Arbeiter, die das Parkett verlegen, tagtäglich einatmen ... und entspannen.
Man kann auch an Keramikkacheln denken: Da brennt nichts, wenn ein Scheit aus dem Kamin kullert, das lässt sich mit heissem Wasserdampf reinigen (wenn jemand gegen Putzmittel allergisch ist), und wenn die Kinder mit 15 ihre erste Party machen, während die Eltern die Schwiegereltern besuchen, und da „Bioflüssigkeiten“ (auch die der Haustiere) auf den Boden gelangen, lächelt der vorausschauende Familienvater nur... Play It Again, SPAM (Diskussion) 11:08, 23. Jan. 2016 (CET)
Eichenbohlen haben einen Einfluß auf Menschen. Die Gerbsäure ist giftig. --Pölkkyposkisolisti 12:29, 23. Jan. 2016 (CET)
Eichenbohlen, die tot auf dem Boden liegen, halte ich für völlig unbedenklich. Es geht hier nicht um die Verarbeitung selbiger (Stichwort: Einatembare Stäube) oder um das Trinken von Flüssigkeiten die vorab als Lösungsmittel verwendet wurden, um dem Holz Inhaltsstoffe zu entziehen. Eiche ist nun ihrerseits auch nicht wirklich für ätherische Öle bekannt, die bei üblichen Zimmertemperaturen einen nennenswerten Dampfdruck aufweisen und daher in höheren Dosen über Lunge oder Haut aufgenommen werden könnten. Das würde mich alles sehr ruhig schlafen lassen. (Äh, schlafen - worauf: Matratze?? Oh weh, was da alles rauskommt...) --Mabschaaf 13:24, 23. Jan. 2016 (CET)
Auf der oben schon verlinkten Seite gibt es auch das MSDS, dort steht, dass für die Einstufung noch Trimethoxyvinylsilan (tja, haben wir leider noch keinen Artikel dazu) relevant war, ein Sicherheitsdatenblatt dazu findest du hier.--Mabschaaf 13:33, 23. Jan. 2016 (CET)
Gerbsäure ist nicht giftig. Gerbsäure ist in sehr vielen herb schmeckenden Lebensmitteln vorhanden, ob das jetzt Tee, Wein oder bestimmte Früchte sind. Solange die Kinder nicht substanzielle Mengen von zerkleinertem Eichenholz essen (ergo Sägemehl von Eichenholz), geht davon keine Gefahr aus. Wenns anders wäre, müssten z. B. Barique-Weine ein ernsthaftes Gesundheitproblem darstellen. Aber vielleicht sind ja die Eltern irgendwie pervers und zwingen die Kinder dazu täglich mehrere Quadratmehter Fußboden abzulecken. Aber selbst dann haben wahrscheinlich die Eltern bereits den größeren Schaden als die Kinder jemals durch die Aufnahme von geringen Mengen Tannine aus dem Eichenholz.--Giftzwerg 88 (Diskussion) 16:48, 23. Jan. 2016 (CET)
Ich würde mir über die Gerbsäure keine Gedanken machen, denn sie wird ja auch zur Leder- und Tintenherstellung verwendet. Viel eher fände ich reproduktionstoxische Weichmacher oder zinnorganische Katalysatoren problematisch, die in obigen Klebstoffen enthalten sein könnten. Das oben verlinkte MSDS erwähnt sie aber nicht. --Rôtkæppchen₆₈ 17:53, 23. Jan. 2016 (CET)

Seite: Anton Günther Gedenksteine

Auf der Seite "Anton-Günther-Gedenkstein" ist bei der Aufzählung der Orte, an denen Gedenksteine stehen, hinter dem Ort Karlstein am Main in Klammer "(Hessen)" gesetzt. Meines Wissens liegt die Gemeinde Karlstein am Main in Bayern und gehört zum Regierungsbezirk Unterfranken.

--80.128.22.211 16:58, 23. Jan. 2016 (CET)

Danke für den Hinweis. --Mauerquadrant (Diskussion) 17:02, 23. Jan. 2016 (CET)

Vielen Dank DU hast rescht siehe Karlstein am Main, das gehort zum Kreis Aschaffenburg. Ich habe es geändert. Wenn Du nochmal etwas findest oder etwas zuergänzen hast, kannst DU das auch selbst ändern. Dafür ist es ein Wiki. Gruß und ein schönes Wochenenden Catrin (Diskussion) 17:02, 23. Jan. 2016 (CET)

Du warst schneller. --Mauerquadrant (Diskussion) 17:04, 23. Jan. 2016 (CET)

Zwei Schönheiten gesucht

Charles Pears hat auf einem Bild Taeping und Ariel in grüner See beim Teerennen von 1866 dargestellt. Das Bild hing im Londoner Tea Centre, das es wohl nicht mehr gibt (?). Im Netz kann ich es, das Bild, nicht finden, was aber nicht heisst, dass es nicht zu finden ist; wer macht das Rennen? Windige Grüsse --Dansker 16:59, 23. Jan. 2016 (CET)

Müsste dort stehen: The Great Tea Race of 1866 Catrin (Diskussion) 17:04, 23. Jan. 2016 (CET)

Danke Catrin, aber gesucht ist dies bestimmte Gemälde von Pears. Ops, meine Formulierung oben ist wohl missverständlich. --Dansker 17:09, 23. Jan. 2016 (CET)
Du bist ganz sicher, dass es ein Pears war? Ich hätte leider nur Foster und Dutton anzubieten. --Jossi (Diskussion) 00:12, 24. Jan. 2016 (CET)
Jes. Es gibt einige Bilder Taeping -Ariel, nur meines finde ich leider nicht wieder. --Dansker 00:25, 24. Jan. 2016 (CET)
Ich halte ein Bild von en:Charles Pears des en:Great Tea Race of 1866 für einen mehrfachen Anachronismus. Charles Pears wurde ein Jahr nach dem letzten Teerennen und vier Jahre nach Eröffnung des Suezkanals geboren. Er begann sein künstlerisches Schaffen 1890, als der chinesisch-britische Teehandel längst per Dampfschiff durch den Suezkanal lief. Es gibt aber zahlreiche Gemälde zeitgenössischer Künstler, die das Teerennen darstellen. --Rôtkæppchen₆₈ 00:58, 24. Jan. 2016 (CET)
Hej Rotkæppchen. Ja, Pears ist kein zeitgenössischer Künstler und somit schiffshistorisch nicht wesentlich; dennoch gibt es sein Bild, und genau das suche ich. Die mir drucktechnisch vorliegende Reproduktion lässt alters- und abnutzungsbedingt qualitativ eben etwas zu wünschen übrig. --Dansker 01:07, 24. Jan. 2016 (CET)
Hast Du das Bild schon eingescannt und per Google Image Search oder TinEye gesucht? --Rôtkæppchen₆₈ 01:09, 24. Jan. 2016 (CET)
Nö; darf man das? Da fehlen da nicht noch zwölf Jahre? --Dansker 01:16, 24. Jan. 2016 (CET)
Da bin ich überfragt, aber vielleicht wissen die Urheberrechtsfachleute unter wp:UF weiter. --Rôtkæppchen₆₈ 01:32, 24. Jan. 2016 (CET)

Hohles Eis

Wenn es so kalt ist, wie in den letzten Tagen, sieht man oft Pfützen, deren Oberfläche durchgehend gefroren ist. Wenn man dann jedoch die Oberfläche durchstößt, stellt man fest, dass das Eis darunter hohl ist. In der Senke, in der sich vor dem Frost die Pfütze gesammelt hatte, ist unter der oberen Eisdecke also weder ein massiver Eiskörper, noch flüssiges Wasser, sondern ein hohler Raum. Wie entsteht so etwas? Mir ist ja klar, dass eine Wasserfläche an der Oberfläche gefriert, aber wie verschwindet dann das Wasser darunter? Es kann ja weder verdunsten, weil es erstens zu kalt ist und zweitens nach oben ja auch die Eisdecke das verhindert, und das Wasser kann auch kaum verdunsten[Edit: versickern--85.178.254.231 20:32, 23. Jan. 2016 (CET)], denn der Boden ist durchgefroren, oder gleich Asphalt oder ähnliches. Wohin verschwindet also das Wasser bzw. Eis, wie kommen diese hohlen Eispfützen zustande? --85.178.254.231 17:26, 23. Jan. 2016 (CET)

Eine mögliche Erklärung findet sich hier (gekennzeichnet als "Hilfreichste Antwort"): http://www.gutefrage.net/frage/pfuetzen-innen-hohl-warum --Buchling (Diskussion) 17:54, 23. Jan. 2016 (CET)
Es wäre gleichermaßen interessant zu erfahren, woher die Luft kommt, die zwischen Wasser und Eis ist. Der Boden kann ja nicht gefroren sein, wenn flüssiges Wasser darauf steht. Also könnte Wasser im Boden versickern und Luft aus dem Boden aufsteigen. Das könnte man falsifizieren, wenn man einen Wassereimer beobachtet, der per Definition wasserundurchlässig ist. Also liebe Frostgeschädigte im Osten: Bitte experimentiert! Hier (PDF-Download, 103 kB) eine Geistererscheinung im Eis. --Pp.paul.4 (Diskussion) 18:52, 23. Jan. 2016 (CET)
Es kann ja weder verdunsten, weil es erstens zu kalt ist [...] – Ähm, doch. Kommt ganz drauf an, wie kalt – und trocken – die Luft ist. --j.budissin+/- 18:59, 23. Jan. 2016 (CET)
Zu letzterem: →Sublimation (Phasenübergang) --Hans Haase (有问题吗) 19:55, 23. Jan. 2016 (CET)

Was ist Puddingpulver?

Oder sollte ich besser fragen Pudding-Pulver? 77.235.178.3 20:07, 23. Jan. 2016 (CET)

Das ist ein Pulver zur Zubereitung eines Puddings. Es besteht aus (modifizierter) Stärke, Armoa- und Farbstoff. Zucker, Milch oder Sahne und ggf. Speisefett müssen selbst hinzugefügt werden. --Rôtkæppchen₆₈ 20:09, 23. Jan. 2016 (CET)

Senf

Hallo, ich bin auf der Suche nach einem wirklich richtig scharfen Senf. Der Löwensenf extra ist doch recht laff und irgendwelche Chili-Senf-Mischungen sind nicht mein Geschmack. Gibt es einen Senf, der eine wirklich große Senfschärfe hat, also ohne Capsaicin?--Emergency doc (D) 18:24, 21. Jan. 2016 (CET)

Wie steht es mit dem klassische helle Dijon-Senf, ist der dir dir zu schwach? Dann belibt eigentlich nur noch der englische übrig, wenn es reiner Senf sein soll. --Bobo11 (Diskussion) 18:41, 21. Jan. 2016 (CET)
Laut Senf ist traditioneller Englischer Senf besonders scharf. --Blutgretchen (Diskussion) 18:54, 21. Jan. 2016 (CET)
Hmmm, Löwensenf ist doch Dijonsenf. Ich hatte vor ein paar Jahren mal einen Dijonsenf, der mit Körnern war und recht scharf. Leider weiss ich nicht mehr, wo ich den her hatte. Englischer Senf, da werd ich mal nach gucken, danke für den Tip.--Emergency doc (D) 19:28, 21. Jan. 2016 (CET)
Wie wäre es denn mit Meerrettichsenf, ist auch schön scharf. -- Frila (Diskussion) 19:34, 21. Jan. 2016 (CET)
Colman's Mustard - für franz. Senf: bis morgen warten... Play It Again, SPAM (Diskussion) 19:43, 21. Jan. 2016 (CET)
Ausser dem Dijon-Senf scheint es wirklick keine schärferen in Frankreich zu geben - "Milde" sind aber Legion. Play It Again, SPAM (Diskussion) 11:40, 22. Jan. 2016 (CET)
Die Schärfe im Senf liegt in erster Linie an den Senfölglycosiden. Empfehlen kann ich dir den "Echt Düsseldorfer scharf". Bei Aldi Süd gibt es den in der 200-ml-Tube, bei Aldi Nord im 200-ml-Glas als "GenussKrone Düsseldorfer Scharfer Senf". Dieser Senf kommt aus dem selben Werk wie der Löwensenf und unterscheidet sich vom "Original" nur minimal in der Zusammensetzung. --Bratwurm (Diskussion) 20:10, 21. Jan. 2016 (CET)
Hast du schonmal echten Düsseldorfer Mostert probiert? --91.221.58.28 10:23, 22. Jan. 2016 (CET)

Ich bin nicht grad so der Senfkenner und Scharfliebhaber, aber Colman's find ich schon recht scharf. Okay, auf den wärst du nach Blutgretchens Tipp vermutlich ohnehin gestossen. --King Rk (Diskussion) 19:43, 21. Jan. 2016 (CET)

Übrigens ist nicht der Löwensenf schlaff, sondern deine Geschmacksknospen sind ruiniert. Aber das weißt du sicher, sonst würdest du nicht nach immer härterem Stoff lechzen, wie es auch jeder Junkie tut. --Snevern 19:55, 21. Jan. 2016 (CET)
Hihihi... nein, wirklich nicht. Ich esse zwar gerne scharf, aber unheimlich selten. Früher war der Löwensenf extra für mich das Nonplusultra in Sachen Senfschärfe. Ich habe aber die Vermutung, daß die Rezeptur geändert wurde, weil sich einfach nicht mehr dieses schöne Nasefreipustaugenträn-Gefühl von scharfem Senf einstellen will. Bei Wasabi ist es ja ähnlich, da hat sich für mich geschmacklich auch nichts geändert. Den Colman-Senf muss ich mal probieren, danke.--Emergency doc (D) 19:59, 21. Jan. 2016 (CET)
Stimmt, jetzt, wo du's sagst, fällt's mir auch auf – den Löwensenf meiner Kindheit und den von heute trennen Welten. Und ich glaube, das liegt nicht nur am altersbedingten Abstumpfen meiner Geschmackspapillen. :) --Jossi (Diskussion) 11:50, 22. Jan. 2016 (CET)
Deshalb ist die Bezeichnung "Senfgas" für eine chemische Waffe heutzutage auch nicht mehr so unmittelbar nachvollziehbar. Wobei ich mich allerdings frage, was Menschen daran schätzen, eine Gewürzsoße zu konsumieren, bei deren "Genuß" sich eingestandenermaßen genau dieses Grabenkriegsgefühl einstellt. (Wobei der Name angeblich vom Geruch abgeleitet worden sein soll.) (nicht signierter Beitrag von 92.224.159.236 (Diskussion) 17:42, 22. Jan. 2016 (CET))

Es stellt sich erst mal die Frage, wie scharf Senf überhaupt maximal sein kann, wenn keine weiteren »Scharfmacher« verwendet werden sollen. Weißer und gelber Senf sind milder als brauner und schwarzer. Du könntest also einen Versuch machen, selbst Senf herzustellen. Im Prinzip werden die Samen gemahlen, mit Wasser bzw. einer wässtrigen Flüssigkeit vermischt und müssen dann noch einige Zeit fermentieren, wodurch sich die Schärfe erst richtig entwickelt. Macht man das mit braunem/schwarzem Senf, bekommt man die größtmögliche Schärfe. Handelsüblicher Senf dürfte einerseits durch die Michung von hellem und dunklem Senf, andererseits durch weitere streckende Zutaten absichtlich milder eingestellt sein. Gut möglich, dass auch Löwensenf mittlerweile entschärft wurde, um breitere Akzeptanz zu finden. Mir würde es auch gefallen, wenn Löwensenf zusätzlich einen noch schärferenSenf anböte – mit Tränengarantie. Rainer Z ... 14:17, 22. Jan. 2016 (CET)

Vielleicht ist Löwensenf heutzutage nicht mehr so scharf, weil aus Tierschutzgründen keine wildlebenden Löwen mehr verarbeitet werden dürfen. Bleiben also nur die Zoo-Löwen, die an Altersschwäche gestorben sind. SCNR --Expressis verbis (Diskussion) 16:02, 22. Jan. 2016 (CET)

Gibt es eigentlich Wasabi-Senf? Das stelle ich mir echt scharf vor. --muns (Diskussion) 00:46, 24. Jan. 2016 (CET)

Es gibt jedenfalls-Meerrettich-Senf. In beiden Fällen kommt dann zum Senf noch eine andere scharfe Substanz hinzu, wobei die Schärfe von Meerrettich und Wasabi der des Senfs ähnlicher ist als die von Chili. Rainer Z ... 14:28, 24. Jan. 2016 (CET)
Bei Senf und Meerrettich inkl. Wasabi sind Senföle bzw Senfölglycoside der scharfe Wirkstoff, bei Chilischoten ist es Capsaicin. --Rôtkæppchen₆₈ 15:15, 24. Jan. 2016 (CET)

Was bedeutet das lemno bei Lemnozyt?

Hallo!

Weiß jemand, was das lemno bei den Lemnozyten (Schwann-Zellen) eigentlich bedeutet?

--2A02:908:1062:FAA0:E421:5B88:4C2A:F095 22:42, 23. Jan. 2016 (CET)

Hier, also Mantelzelle, Hüllzelle. Play It Again, SPAM (Diskussion) 23:24, 23. Jan. 2016 (CET)

Und wieso einmal Lemma und einmal Lemno? --= (Diskussion) 01:16, 24. Jan. 2016 (CET)

Mediziner ... ? ;-) Play It Again, SPAM (Diskussion) 10:35, 24. Jan. 2016 (CET)
Na ja, lemma und lemmo würd ich den Medizinern noch zutrauen, aber wie kommt man von lemma nach lemno? Altgriechen ran (im Wörterbuch hab ichs nachgesehen, aber keine Lösung gefunden)! --= (Diskussion) 14:55, 24. Jan. 2016 (CET)

neues Handy, altes verloren

bekomme ich mein Guthaben des alten Handys gutgeschrieben wenn ich ein neues beim gleichen Anbieter ( e+) anmelde? mfg --2A02:908:F358:46C0:9C26:3394:B057:5173 09:05, 24. Jan. 2016 (CET)

Ich würde da mal den Anbieter fragen. --Mauerquadrant (Diskussion) 10:18, 24. Jan. 2016 (CET)
Du kannst auch eine neue Simkarte mit deiner alten Nummer bestellen. ber ob da eventuelle Gebühren teuerer sind als das Restguthaben kann ich nicht sagen, aber der Vorteil besteht darin, dass du nicht jedem ne neue Nummer mitteilen musst.--84.179.174.118 11:15, 24. Jan. 2016 (CET)
Vor dem Bestellen unbedingt in das Kleingedruckte des Anbieters schauen: Bei manchen Anbietern gibt es Fälle, in denen der SIM-Tausch kostenlos ist. In anderen Fällen kann es vorkommen, dass der SIM-Tausch teurer als eine neue Karte mit neuer Nummer und Startguthaben ist. Dazu kann man dann noch die alte Nummer auf die neue Karte portieren, auch wenn man den Anbieter wechselt. Aber das lässt sich der Anbieter meistens großzügig bezahlen, sodass ein Anbieter- und Rufnummernwechsel mitunter am allergünstigsten ist. --Rôtkæppchen₆₈ 12:41, 24. Jan. 2016 (CET)

Yps Eierbaum

Hi, vielleicht kennt einer von euch ja auch noch die Kinderzeitschrift Yps. Dort gab es mal als Gimmick einen Eierbaum. Welche Pflanze war das? --Sascha1971 (Diskussion) 13:34, 24. Jan. 2016 (CET)

Aubergine --80.219.124.55 13:39, 24. Jan. 2016 (CET) Falsch.
Wieso löscht Du fremde Beiträge?
Schlecht gegoogelt. 84.153.89.115 13:44, 24. Jan. 2016 (CET)
Nein, wieso? Natürlich nicht die lange, schwarze Aubergine, die wir aus dem Supermarkt kennen, sondern eine Form mit kleinen, weissen Früchten, siehe http://www.ypsfanpage.de/gimmicks/eierbaum.php --King Rk (Diskussion) 14:40, 24. Jan. 2016 (CET)
Peinlich. Stimmt. Also, Aubergine, eine der vielen Sorten. 84.153.89.115 15:23, 24. Jan. 2016 (CET)

Fürs Tablet brauch ich Tabletten

Irgendwie macht mir mein neues Denver-Tablet Kopfschmerzen. Ich hab mir ein Acer-Tablet kaputt gemacht und mir von Denver ein neues geholt mit den gleichen technischen Details. Jetzt habe ich versucht zwei Apps zu installieren. In meinem Fall "Clash of Clans" und die moblie Version von "World of Tanks". Beides funktionierte auf dem Acer problemlos. Für das Denver wird mir angezeigt, dass die Apps nicht mit dem Gerät kompatibel seien. Im Google-Play-Store. Beide Apps sind kostenlos. Was brauche ich oder was hat mein neues Tablet anderes als das alte Acer? Hat da jemand eine Idee oder kann ich es morgen zurückbringen, weils mir zu doof ist? LG --Rischtisch (Diskussion) 17:55, 22. Jan. 2016 (CET)

Poste mal die genauen Bezeichungen oder google selbst anhand der Bezeichnung nach dem verbauten Prozessor oder SoC. ARM ≠ ARM, da gibt es mittlerweile hunderte Varianten. --Rôtkæppchen₆₈ 18:06, 22. Jan. 2016 (CET)

Das Tabelet selber heißt Denver TAQ-10153. Und um ehrlich zu sein bin ich ein Computernoob was sowas betrifft. Hier steht was von 1,3 GHz Quad Core Prozessor. --Rischtisch (Diskussion) 18:50, 24. Jan. 2016 (CET)

Drin ist ein Mediatek MT8127 SoC mit Cortex A7 CPU und ARM Mali GPU. Es läuft mit Android 4.4 oder 5.0. Deine Apps brauchen Android 4.0 (World of Tanks) bzw 4.0.3 (Clash of Clans). Ich weiß aber nicht, ob die Apps eine Cortex-A8-CPU brauchen oder ob sie mit Cortex A7 auskommen. Wo verrät Google Play das? --Rôtkæppchen₆₈ 19:27, 24. Jan. 2016 (CET)
Gemäß diesem Dokument filtert Google Play die angebotenen Apps gemäß den Eigenschaften des Endgeräts. Man müsste die Apps runterladen und deren AndroidManifest.xml untersuchen. --Rôtkæppchen₆₈ 20:05, 24. Jan. 2016 (CET)

Rotlichtviertel am Kölner Hauptbahnhof

Gibt es in der Zwischenzeit aktuellere Zahlen? --77.10.238.198 21:58, 22. Jan. 2016 (CET)

Für was? --Jossi (Diskussion) 11:19, 23. Jan. 2016 (CET)
Für die 'Sorgenkinder'. --77.10.254.153 16:51, 23. Jan. 2016 (CET)
Hm. Ich vermute es geht um folgenen Sachverhalt: Nach diesem Artikel des Kölner Stadtanzeigers vom 22. Juni 2014 gab es damals eine Kontroverse, weil neun Gebiete in Köln von der Polizei intern zu „gefährlichen Orten“ im Sinne des Polizeigesetz NRW erklärt wurden. Die Folge sind erweiterte Rechte der Polizeibeamten in diesen Gebieten (bzw. aus einem relevanteren Blickwinkel Einschränkungen von Freiheiten und Rechten der Bürgerinnen und Bürger). Die Kontroverse entstand (aus meiner Sicht völlig berechtigt), weil die Kölner Polizei nicht veröffentlichte, um welche Gebiete es sich konkret handelt. Auch die Grundlagen für das Erklären der „gefährlichen Orte“ sind nicht zugänglich. Das polizeiliche Handeln ist also nicht öffentlich überprüfbar. Offenbar geht man allgemein davon aus, dass der Eigelstein zu den sogenannten „gefährlichen Orten“ gehört. Es liegt mir fern, hier nun stellvertretend für den Fragesteller diesen Teil der Kölner Stadtgeschichte zwischen Juni 2014 und Januar 2016 zu recherchieren. Da ist auf die Kölner Tagespresse und die jährliche Polizeiliche Kriminalstatistik zu verweisen (hier die Statistik für 2014). Ob die Zahlen für das vergangene Jahr bereits publiziert sind entzieht sich meiner Kenntnis. Die Kriminalstatistik weist jedoch keine regionalen Schwerpukte aus und belegt dies. Wenn es überhaupt Zahlen gibt (der Fragesteller schreibt ja etwas lakonisch von „aktuellere(n) Zahlen“), dann sind diese Teil des regionalen politischen Diskurses und wohl dort zu suchen. --2003:45:460F:B000:DC78:A331:6D0A:188A 13:12, 24. Jan. 2016 (CET)
Die Frage lautet: Wieviele Personen verdienen im Gebiet des Kölner Hauptbahnhofes (inkl. Eigelstein) heute ihr Geld mit Prostitution? (eine ältere Zahl (5000), die von 2009 bis zum 23. Jan. 2016 in der Wikipdia kursierte, wurde gestrichen). --77.10.192.21 13:48, 24. Jan. 2016 (CET)
Aha. Warum schreibst du das denn nicht gleich? Die Zahl war offenbar nicht ordentlich belegt und es geht also um valide Belege in dieser Frage. Ich finde hier, S. 20 eine Zahl von ca. 150 Prostituierten im Eigelstein vor Oktober 2001. Die Zahl basiert auf „Schätzungen des SkF und des Gesundheitsamtes Köln, 2001 und Gespräch mit Herrn Rüenaufer nach Aussagen des KK 12, Polizeipräsidium, Köln, Februar 2002“. In dieser Stellungnahme des Sozialdienstes katholischer Frauen e.V. Köln (Mäc Up Geestemünder Straße und Rahab), des Sozialdienstes katholischer Frauen e.V. Dortmund (Kober) und des Sozialdienstes katholischer Frauen e.V. Essen (StrichPunkt) zum Referentenentwurf des Bundesministeriums für Familie, Senioren, Frauen und Jugend – Entwurf eines Gesetzes zur Regulierung des Prostitutionsgewerbes sowie zum Schutz von in der Prostitution tätigen Personen, Bearbeitungsstand 29.07.2015 sind auf S. 3 einige m.E. differenzierende und zitierfähige allgemeine Ausführungen zu der konstatierten „Ungewissheit über die tatsächlichen Zahlen“ des bundesdeutschen Prostitutionsgeschehens. Damit haben wir keine aktuellen Schätzungen (mehr gibt es wohl nicht), aber einen Weg: Der SkF e.V. scheint mir der sinnvollere und kompetente Ansprechpartner für deine Frage zu sein. --2003:45:460F:B000:DC78:A331:6D0A:188A 15:21, 24. Jan. 2016 (CET)
Das bezieht sich aber doch überwiegend auf den 'erfolgreich' an die Geestemünder Straße verlegten Straßenstrich. Ich bin mir aber nicht sicher, ob der in Artikeln wie z.B. diesem (vom Kölner Stadtanzeiger) gemeint war. --77.10.192.21 16:59, 24. Jan. 2016 (CET)
Du hast einen der beiden Artikel schon gestern Nachmittag in deinem zweiten Beitrag angeführt. Ich beziehe mich auf garnichts sondern habe grob recherchiert, ob es (möglichst aktuelle) Zahlen zu Köln und zum Eigelstein gibt. Wie bereits geschrieben gibt es offenbar kaum seriöse statistische Erhebungen und die Schätzung „von ca. 150 Prostituierten im Eigelstein vor Oktober 2001“ berücksichtigt, wie in der Studie nachzulesen ist, nicht den späteren Beitritt von osteuropäischen Ländern in die EU und selbstverständlich auch nicht die Darstellung aus den beiden hier im Thread verlinkten Zeitungsberichten, die ebenfalls seit Ende 2001 eine Verschärfung der sozialen Spannungen vermuten lassen. Wenn man das ernsthaft untersucht sind für das Untersuchungsgebiet nicht nur der Straßenstrich sondern alle Erscheinungsformen von Prostitution zu berückichtigen. Weil es aber offenbar keine Untersuchungen gibt oder wir sie nicht kennen sehe ich (und jetzt wiederhole ich mich leider) keinen anderen sinnvollen Weg als eine Nachfrage beim SkF e.V., weil die Leute dort vor Ort die realistischste Einschätzung über ihre Klientel haben und natürlich auch die Publikationen zu ihrem Arbeitsfeld kennen werden. Aus dieser Nachfrage ergibt sich dann entweder eine Literaturangabe mit einer validen, seriösen Belegstelle für Zahlen und Trends im Kölner Raum oder eben nichts. Frei nach Wittgenstein würde ich beim Fehlen seriöser Zahlen für die de.wp sagen: Was nicht anständig untersucht worden ist, darüber muß man schweigen. --2003:45:460F:B000:DC78:A331:6D0A:188A 19:26, 24. Jan. 2016 (CET)

Ab wann hat ein Politiker Personenschutz?

Gibt es da eine Regel? Ich frage mich das gerade weil ich in Videos gesehen habe die zufällig aufgenommen habe wie Trittin ganz alleine durch Berlin radelt, in einem anderen habe ich Claudia Roth gesehen wie Sie mit ner Freundin zusammen allein einkaufen ist. Haben solche Leute tatsächlich keinen 24/7 Personenschutz oder werden die nur auf Anforderung geschützt. z.B. wenn Sie eine Rede halten? --84.149.232.44 07:51, 23. Jan. 2016 (CET)

Art und Umfang des Personenschutzes und ggf. weiterer Familienmitglieder wird zusammen mit den Betroffenen vom Sicherheitsdienst des Bundestages/ seiner Dienststelle festgelegt. Da werden auch die Wünsche des Betroffenen nach "Freiheit" und normalem Leben berücksichtigt. Das gilt in der Regel nur für Politiker der ersten Reihe, für andere nur bei besonderer Gefährdungslage, die im Einzelfall geprüft wird. Eine Regel gibt es nicht. Des weiteren ist der beste Personenschutz, der nicht auffällt. Es wäre schon möglich dass da einer in gebührenden Abstand mitschleicht oder mitfährt. Lückenhaft fand ich es, dass der hiesige Abgeordnete, wir hatten immer Montags den gleichen Flieger, ohne Schutz zum Flughafen kam, öfters auch neben mir saß und erst auf dem Flughafen in Köln/Bonn vom Personenschutz vereinnahmt wurde. Da spielte wohl auch das Ego vor den anderen mit.--2003:75:AF37:7900:AD56:24D0:C18D:18E3 08:35, 23. Jan. 2016 (CET)
Erstmal danke für die Antwort. Aber mich würde das schon etwas präziser interessieren. Gibts da so was wie "Anrecht auf permanenten Personenschutz" z.B. jeder Minister hat Anrecht auf permanenten Personenschutz und Landtagsabgeordnete nur bei Gefährdungslage. Oder jeder Minister wird grundsätzlich geschützt und das wird nur auf Wunsch ausgesetzt? Da gibts doch bestimmt irgend ein Regelwerk. Ich frage das übrigens nicht weil ich ungestört einen Minister verprügeln will :) sondern mich interessiert das wegen den Kosten. --84.149.232.44 09:08, 23. Jan. 2016 (CET)
BKA, Gefährdungsstufe(n), nicht automatisch (ausser ganz oben). Play It Again, SPAM (Diskussion) 09:39, 23. Jan. 2016 (CET)
Eine Regel --> "Rechtsanspruch" gibt es für Politiker nicht, jedenfalls nicht mehr wie für jeden Bundesbürger auch, der in eine Gefährdungslage gerät, z.B. Kronzeuge oder gegen den aus anderen Gründen Morddrohungen vorliegen. Bei Politiker wird das in der Öffentlichkeit mehr beachtet, als bei einem Industrieführer. Die haben häufig auch Personenschutz, wie hohe Beamte des Staates auch. Letztendlich ist das für den Betroffenen aber lästig, sodass öfters welche auf einen angebotenen Personenschutz verzichten oder ihn reduzieren. --2003:75:AF37:7900:F173:5944:E542:48F6 10:48, 23. Jan. 2016 (CET)
(BK)24/7 Personenschutz ist den D sehr unüblich, wenn ich mich recht erinnere gibt es den nur für Bundespräsident, Kanzler, Innen- und Außenminister sowie die Ministerpräsidenten der Länder. Ansonsten wird nach Bedrohungslage entschieden, Ulla Schmidt soll etwa jahrelang voll beschützt worden wg. ihrer Gesundheitsreformen. Bei öffentlichen Auftritten sind aber Bewacher dabei. @2003.75: "Lästig" ist eine Frage des Empfinden. Staatlicher Personenschutz ist erst einmal die maximale Form des Prestiges die einem der Staat zukommen lässt. Manch zweitrangiger Ministerpräsidnet geneißt die Aufmerksamkeit, aber je nach Persönlichkeit kann die freilich auch lästig sein. Günther Beckstein, der ja Feinde ohne Ende hatte, soll nicht selten seinen Bewachern nichts gesagt haben und alleine in die U-Bahn gesteigen sein.--Antemister (Diskussion) 11:01, 23. Jan. 2016 (CET)
@84.149.232.44, wie sollte man daraus auch noch auf die realen (Zusatz-) Kosten schließen können? Ich würde mir an deiner Stelle eher Gedanken wegen der Kosten zu Großveranstaltungen (Fußball, Konzerte, Demos, G20-Gipfel usw.) machen, als bei diesem - im Verhältnis dazu - Kleckerkram. Wobei dort noch volkswirtschaftliche Zusatzkosten durch eventuelle Sach- und Personenschäden entstehen. --Dontworry (Diskussion) 11:22, 23. Jan. 2016 (CET)
Das ist nicht vergleichbar, bei Großveranstaltungen werden tausende Teilnehmer beschützt, die auch alle schon mit ihrem Besuch zu den Steuereinnahmen beitragen (über Eintrittskarten, Getränkeverkauf, etc.), also muss da<s für den Staat nicht mal ein Verlustgeschäft sein, wenn man die wenigen Politiker nicht beschützen würde, würde das dem Staat nur kosten sparen und Politiker würde es trotzdem geben (in den USA war selbst nach dem Attentat auf Abraham Lincoln noch kein Personenschutz für Präsidenten vorgesehen (man hat erwartet, dass die Präsidenten wenn sie Personenschutz wollen sich das selbst organisieren, was Lincoln auch tat, jedoch war das damals im Theater nicht ausreichend), erst 1901 nach der Ermordung von William McKinley wurde der vorher als Finanzgeheimdienst gegründete Secret Service damit beauftragt, trotzdem gabs immer genug Präsidentschaftskandidaten) weil viele bereit sind, für Macht viel zu riskieren (das hat man ja auch unter Josef Stalin gesehen, der immer neue Leute für seine Regierung gefunden hat, obwohl sie wussten, dass die Überlebensrate dort nicht sehr hoch ist). Also rein wirtschaftlich betrachtet ist der Personenschutz völlig anders zu betrachten, von diesem Standpunkt macht er höchstens deshalb Sinn, weil ein erfolgreicher Anschlag auf einen Politiker allgemeine Unsicherheit verbreitet und damit doch wirtschaftlichen Schaden anrichten würde. Das ist aber nicht Vergleichbar mit Großveranstaltungen, die direkt Einnahmen bringen. Viel teurer sind allerdings die Staatsbesuche und Gipfel, die in einer Zeit, in der man verschlüsselte Videokonferenzen machen kann eigentlich auch überflüssig sind, daher sie deinen wohl heute hauptsächlich der Machtdemonstration. --MrBurns (Diskussion) 12:07, 23. Jan. 2016 (CET)
In einigem hast du ja recht, aber die von dir gelobten (Fußball-)Großveranstaltungen kosten dem Steuerzahler nur Geld. Da geschieht kein ausreichender Mittelrückfluss aus den Eintrittskarten. Da stoßen sich nur die vielen kleinen Blatters gesund und wenn man noch einrechnet was die Besucher anschließend in den Straßenbahnen, Omnibussen und Zügen kaputt machen und vollkotzen ist das ein weiteres Zuschussgeschäft für die Allgemeinheit. Aber ich gebe dir recht, zur Abwehr noch größerer Schäden muss man diesem Klientel ein Forum geben, sonst gehen die nur noch fummeln.--2003:75:AF37:7900:F9F4:215:D12B:3DD3 14:06, 23. Jan. 2016 (CET)
Wenn man naiv rechnet, könnte das sogar stimmen, obwohl ich Zweifel daran habe, dass das je jemand genau durchgerechnet hat. Wenn man noch die Steuereinnahmen, die in weiterer Folge entstehen dazurechnet aber wohl kaum: die Einnahmen aus den Eintrittskarten werden teilweise für Gehälter von Fußballern und anderen Vereinsangestellten verwendet (der Anteil ist relativ hoch, bei den meisten Vereinen werden allein für die Spielergehälter deutlich mehr als 50%, da die Vereine üblicherweise keine Gewinne machen gilt langfristig, dass alles Eingenommene wieder ausgegeben wird), dass bei keinen Zuschauern oder weniger Zuschauern im Stadion die Attraktivität geringer wäre (keine, falls mans ohne Zuschauer würde, weniger falls man trotz nicht gewährleisteter Sicherheit Zuschauer reinlässt lässt) und somit auch die Einnahmen aus TV-Geldern geringer wären, dass es dann auch weniger Sky-Abos geben würde usw. kommt man sicher auf ein Ergebnis, das für den Steuerzahler insgesamt positiv ist. Der Einfluss des Vandalismus wird mMn überschätzt, es passiert da nicht bei jedem Match was gröberes, im Gegensatz zu dem was manche denken verhalten sich die meisten Fußballfans nicht wie wilde Affen, nur über die, die sich zivilisiert verhalten steht halt nichts in der Zeitung... --MrBurns (Diskussion) 15:31, 23. Jan. 2016 (CET)
Dass ein Abgeordneter im Flieger keinen Personenschutz braucht, lässt sich schon ungefähr an den Sicherheitsmaßnahmen bei der Fliegerei ablesen. Es ist nahezu ausgeschlossen, dass jemand mit einer potentiell gefährlichen Waffe in den Flieger kommt und es gibt inzwischen auch Sicherheitspersonal an Bord, das bei bedrohlichen Situationen eingreifen kann, also z. B. wenn ein Fluggast randaliert oder Personen bedroht als Geißel nehmen will etc. Dieser allgemeine Schutz gilt natürlich für alle Passagiere nicht nur für den einen. Ich denke es gibt da auch irgendwelche Abkommen zwischen Flughafenbetreiber, Bundestag und Fluglinien, die dafür sorgen, dass es keine Lücken im Schutz gibt.--Giftzwerg 88 (Diskussion) 16:35, 23. Jan. 2016 (CET)
Ich vermute es geht um den Kommentar von 2003:75:AF37:7900:AD56:24D0:C18D:18E3. Laut dem kam der Abgeordnete bereits ohne Personenschutz zum Flughafen. --MrBurns (Diskussion) 16:38, 23. Jan. 2016 (CET)
In Bayern haben nur der Ministerpräsident und der Innenminister dauernd Personenschutz. Andere Politiker können aufgrund besonderer Hinweise zeitweilig in eine höhere Schutzklasse genommen werden und dann zB für einzelne Veranstaltungen oder für bestimmte Zeit Personenschutz bekommen. Die persönlichen Fahrer von Ministern haben auch eine rudimentäre Ausbildung für Verhalten in Gefahrensituationen, sind aber Fahrer, keine Personenschützer. Außerdem gibt es noch Objektschutzmaßnahmen, so geht zB vor Auftritten des MP oft - aber nicht immer - vorher ein Sprengstoffhund durch die Halle.
Vor Jahren (als er noch lange nicht Ministerpräsident war) bin ich einmal zufällig mit Bodo Ramelow Zug gefahren. Damals war ich erstaunt, dass er ganz allein reiste (während ich wiederum noch etwas früher Zeuge gewesen war, wie der damalige MP Dieter Althaus in Begleitung mindestens zweier besonnenbrillter Personenschützer die Bundesgartenschau 2007 besichtigt hatte), und sprach ihn auch darauf an. Er meinte, er habe einmal eine Weile unter Polizeischutz gestanden, jetzt nicht mehr. Im Einzelnen kommt es also wohl wirklich auf die konkrete Gefährdungslage an. Ob er jetzt als Ministerpräsident dauerhaften Schutz qua Amt genießt, weiß ich nicht; interessieren würde mich die Antwort darauf aber auch. --slg (Diskussion) 19:55, 23. Jan. 2016 (CET)
Jetzt wird klar, warum Horst von Kreuth so auf die Kanzlerin einschlägt, der will sich unbeliebt machen und seinen vollen Personenschutz wieder haben.--2003:75:AF4A:600:18E1:FC32:350:7612 10:06, 24. Jan. 2016 (CET)
@MrBurns, ich empfinde deine Ansicht: '"...wenn man die wenigen Politiker nicht beschützen würde, würde das dem Staat nur kosten sparen und Politiker würde es trotzdem geben..-" als zynisch und absolut unhaltbar. Du solltest diesen und auch diesen Schwachsinn: "...weil viele bereit sind, für Macht viel zu riskieren (das hat man ja auch unter Josef Stalin gesehen, der immer neue Leute für seine Regierung gefunden hat, obwohl sie wussten, dass die Überlebensrate dort nicht sehr hoch ist). Also rein wirtschaftlich betrachtet ist der Personenschutz völlig anders zu betrachten, von diesem Standpunkt macht er höchstens deshalb Sinn, weil ein erfolgreicher Anschlag auf einen Politiker allgemeine Unsicherheit verbreitet und damit doch wirtschaftlichen Schaden anrichten würde..." einfach ersatzlos entfernen! Was deine Ansichten zu Fußballspielen und deren Schutz betrifft kannst du dich ja auch nochmal anderweitig: [11] informieren, bevor du noch mehr Kaffeesatzleserei betreibst und daherfaselst! --Dontworry (Diskussion) 07:13, 24. Jan. 2016 (CET)
+1--2003:75:AF4A:600:1CE5:CECF:2769:6033 08:19, 24. Jan. 2016 (CET)
Warum sollte er das entfernen? Er hat doch ganz offensichtlich recht. --84.149.232.44 13:26, 24. Jan. 2016 (CET)
@2003:75:AF37:7900:AD56:24D0:C18D:18E3, einfache Abgeordnete sind keine wirklich bedeutenden/mächtigen Personen, auch wenn so mancher vllt. das Prestige des Personenschützers genißen würden. Ich habe den lokalen MdL auch schon im Zug getroffen. Die Bedrohung für die hohe Politiker geht weniger von terroristischen Anschlägen oder Mordkomplotten aus - will man so was abwehren braucht es schon heftigere Maßnahmen, mit dutzenden bis hunderten Bewachern und dem Absperren der Umgebung. Die größere Gefahr sind Geisteskranke, die sich vom Staat verfolgt fühlen und meinen sie müssten einen Spitzenpolitiker töten. Die Attentate auf Schäuble und Lafontaine sind zwar schon etwas länger her, aber das sind eher die Fälle in den Personenschützer eingreifen können.--Antemister (Diskussion) 14:36, 24. Jan. 2016 (CET)
In meinem Kommentar vorher ging es nicht darum, zu argumentieren, dass es generell keinen Personenschutz geben soll, sondern darum, dass Personenschutz wirtschaftlich gesehen anders zu betrachten ist als der Schutz von Großvernastaltungen. Dennoch möchte ich jetzt etwas weiter ausholen: Das Leben ist generell gefährlich, daher man muss auch wenn es darum geht, Leben zu schützen, eine Kosten/Nutzen-Rechnung anstellen. Man kann eben nicht jeden vor jeder Gefahr schützen. Das hat nichts mit Zynismus zu tun. Das politische System funktioniert jedenfalls auch ohne Personenschutz, Personenschutz für hohe Politiker in demokratischen Systemen ist ein relativ neues Konzept, bis ~1900 gabs das selbst in den USA kaum. Das Leben jedes Menschen sollte ja grundsätzlich gleich viel wert sein, daher muss man für eine Kosten/Nutzenanalyse betrachten, was der Tod eines Politikers durch einen Anschlag noch für Auswirkungen hat außer dass ein Mensch sein Leben verliert. Ich sehe durchaus ein, dass man für den Schutz von Politikern auch in Relation zur Gefahr etwas mehr ausgibt, also für einen Durchschnittsbürger, weil ein Anschlag wie schon erwähnt ein gewisses Maß an Unsicherheit verbreitet. Aber gerade bei Spitzenpolitikern auf Bundesebene bin ich der Meinung, dass man mal überprüfen sollte, ob da das Verhältnis noch stimmt oder ob man nicht zu viel für deren Schutz ausgibt. Ich habe den Eindruck, dass eher letzteres stimmt.
Zu den Fußballspielen: im Link von Dontworry steht aber auch, dass ein Programm gestartet wurde, um die Polizeipräsenz bei nicht-Risikospielen zu reduzieren. Ich habe auch schon länger den Eindruck, dass die Polizeipräsenz gerade bei nicht-Risikospielen oft übertrieben ist. Des Weiteren muss man natürlich auch die andere Seite sehen, also die Steuereinnahmen. Ich weiß nicht, wie viel genau 2 Millionen Einsatzstunden kosten, aber ich denke, es werden wohl nicht viel mehr als 100 Mio. € sein, eventuell auch weniger. Laut Wikipedia war der Gesamtumsatz allein der deutschen Fußball-Bundesliga schon 2006/07 (Wikipedia hat leider keine aktuelleren Zahlen) bei 1,4 Mrd. €. ich bin mir sicher, dass es heute noch etwas mehr ist. Um dabei auf 100 Mio. € steuern zu kommen, braucht man eine extrem niedrige Steuerquote, es ist eher davon auszugehen, dass die deutsche Bundesliga allein schon mehrere hundert Millionen Euro an Steuereinnahmen bringt (auch dürfen wegen einer EU-Richtlinie Profivereine bald nicht mehr als gemeinnützig gelten, wodurch viele Steuervorteile wegfallen). Dazu kommen noch Einnahmen aus der 2. Bundesliga (die Angaben beziehen sich laut der Quelle auf "Fußballspiele der Vereine der 1. und 2. Liga"). Es ist nicht klar, wie viel der einnahmen wegfallen würde, wenn man die Spiele ohne Zuschauer oder auch nur ohne Polizeischutz austragen würde, aber man kann das nicht nur über die Ticket- und Gastroeinnahmen rechnen, weil leere Ränge eben auch die Attraktivität fürs TV reduzieren. --MrBurns (Diskussion) 16:18, 24. Jan. 2016 (CET)

Altersnachweis contra fehlende Ausweispflicht

Hallo! Nein, das soll hier keine Diskussion über Freiburgs Nachtclubs oder persönliche Erfahrungen an der Tanke werden, wo man als 17jähriger bereits Alkohol bekam. Ganz einfache Frage,

  • Welche gesetzlichen Vorschriften gibt es, wie das Alter darzulegen ist?

Einschlägig für das Allgemeine der §2 Jugendschutzgesetz [12]. Also Kernfrage, ist es dem Verkäufer, Kellner, Einlass überlassen, ob er strikt Pass, Personalausweis oder Führerschein verlangt, oder auch Kopien davon akzeptiert? Ich höre immer wieder von Touristen, welche mit Absicht nur mit Kopien auf Kneipentour gehen, weil sie Angst vor Verlust und Diebstahl haben. Genauso die Frage, ob ein Schüler/Studentenausweis oder gar Bahncard ausreicht. Immer vorausgesetzt, die Person ist es auch wirklich finde ich es eigentlich für sehr schwammig formuliert, in beide Richtungen.Oliver S.Y. (Diskussion) 10:51, 24. Jan. 2016 (CET) Und war es nicht so, daß man sich in Deutschland nur gegenüber der Polizei auf Verdacht einer Straftat ausweisen muss, also für gewöhnlich auch keine Personaldokumente mit sich führt, außer man holt mal eben Zigaretten.

Gegenüber Privatleuten reicht aus, was Privatleute als ausreichend ansehen. --Heletz (Diskussion) 10:58, 24. Jan. 2016 (CET)
So ist es. Es kann daher auch ganz ohne Vorlage von irgendwelchen Papieren (oder Kopien davon) ausreichend sein, wenn ein Dritter das Alter des Betreffenden bestätigt. --Snevern 12:13, 24. Jan. 2016 (CET)
Ausweispflicht: Du bist nicht verpflichtet, Dich gegenüber dem Verkäufer auszuweisen. Andererseits ist er nicht verpflichtet, Dir was zu verkaufen. --95.112.50.5 12:15, 24. Jan. 2016 (CET)
Letzteres stimmt im Regelfalle - ersteres nicht. Die Verpflichtung ergibt sich aus der Regelung "Personen, bei denen nach diesem Gesetz Altersgrenzen zu beachten sind, haben ihr Lebensalter auf Verlangen in geeigneter Weise nachzuweisen. Veranstalter und Gewerbetreibende haben in Zweifelsfällen das Lebensalter zu überprüfen." Diese Pflicht entfällt nur dann, wenn du darauf verzichtest, die Ware oder Dienstleistung weiterhin in Anspruch nehmen zu wollen. --Snevern 12:21, 24. Jan. 2016 (CET)

Ganz einfach: der Clubbesitzer hat das Hausrecht in seinem Haus und entscheidet (neben dem Jugenschutzgesetz), in "eigener Machtvollkommenheit" nach eigenem Ermessen, wen er in sein Etablissement hinein lässt und wen nicht. Wenn eine Person nicht persönlich bekannt ist, nach dem Augenschein minderjährig und keinen Ausweis zeigen möchte: nun da bleibt er /sie halt eben aussen vor. Einer Begründung dafür bedarf es nicht. 188.195.182.67 20:30, 24. Jan. 2016 (CET)

Ging um was anderes, komme ja aus der Gastronomie, wenn auch nicht dem Clubbereich. Was da an "Altersnachweisen" vorgezeigt wird ist teilweise ne Beleidigung, und wenn man freundlich ist, wird man für blöd gehalten. Selbes Spiel übrigens in letzter Zeit in Supermärkten, wo Elfjährige versuchen an Energydrinks zu kommen. Im Gespräch kam es aber auch raus, daß zumindest in Bars keine Kopien mehr akzeptiert werden wegen den Fälschungsmöglichkeiten, und da im Zweifelsfall das Ordnungsamt gegen den Besitzer und nicht den Jugendlichen das Ordnungsgeld verhängt. Darum nur noch Original only, was für Asylbewerber, welche ja aktuell ausschließlich mit dem Begriff "Flüchtling" gleichgesetzt werden, unmöglich ist, da sie nur irgendwelche Blätter von Ämtern haben, aber keinen Pass oder ID-Card. Somit gerade bei der Gruppe der "jugendlichen Flüchtlinge" ein Abweisungsgrund ohne Diskriminierungsvorwurf. Alte werden ja jeglicher Herkunft nicht reingelassen :) Oliver S.Y. (Diskussion) 21:10, 24. Jan. 2016 (CET)
Für Energydrinks gibts ein Mindestalter? Das wäre mir neu... --MrBurns (Diskussion) 21:38, 24. Jan. 2016 (CET)
Schon seit längerem in der Diskussion [13] Rossmann und DM sind zentrale Vorreiter. Mein Edeka hat das auch eingeführt, samt zentralen Hinweisschildern, ist aber als Inhabergeschäft diesem überlassen, ohne generelle Regelung durch die Zentrale.Oliver S.Y. (Diskussion) 21:55, 24. Jan. 2016 (CET)
Das ist aber eine private Aktion dieser Läden ohne gesetzlichen Hintergrund, was aufgrund der Vertragsfreiheit möglich ist. Da steht es im Ermessen der Läden, was sie akzeptieren. Wo gesetzlicher Jugendschutz dahintersteht (z. B. bei Alkohol), gibt es diese Freiheit nicht. Diese Panikmache gegen Energydrinks muss man sowieso nicht ernst nehmen. MBxd1 (Diskussion) 22:03, 24. Jan. 2016 (CET)
Ich hab nichts von gesetzlichen Mindestalter geschrieben. Und mit dem fehlenden "gesetzlichen Hintergrund" wäre ich vorsichtig :) - gibt nur keine Klagen, welche bekannt wurden. Aber da sind nicht nur Foodwatch und Ärzte dran, nur funktioniert da noch die Lobby ziemlich gut. Ansonsten fördern solche Beiträge wie Dein Panikmache nur meinen Widerstand. Es ist allgemein bekannt, daß Alkaloide negative Auswirkungen auf den Organismus haben. Und nun kann man Red Bull sicher keine "Panikmache" vorwerfen, oder? [14] Selbst als Hersteller rät er vom Konsum durch Schwangere und die Abgabe an Kindern (allgemein bis 14) ab. Und auch das BfR sagt: "Diese sollten beinhalten, dass bestimmte Verbrauchergruppen (Kinder, Schwangere, Stillende, coffeinempfindliche Personen) auf den Verzehr von Energiegetränken verzichten sollten." hier ist also eher die Truppe der Verharmloser das Problem, welche das Problem nicht ernst nehmen, und sich dann über gesundheitliche Probleme des Nachwuchses wundern. Oliver S.Y. (Diskussion) 22:18, 24. Jan. 2016 (CET)
Es macht aber einen entscheidenden Unterschied, ob da gesetzlicher Jugendschutz dahintersteht oder nur eine hauseigene Restriktion.
Die selektive Hetze gegen die Energydrinks zeigt nur, dass vor allem die Lobbyarbeit der Kaffeeindustrie noch hervorragend funktioniert. Jugendschutz mit Verkaufsverbot für Kinder (unter 14) ist sicher sinnvoll. Aber der verlinkte Artikel ist einfach nur geistiger Durchfall. Wenn ein 16- oder 18-jähriger täglich eine Dose von dem Zeugs trinkt, dann wird das als besorgniserregende Sucht hingestellt. In dem Alter wird aber auch schon in größerem Ausmaß Kaffee getrunken, mit eher noch mehr Koffein. Macht aber alles nix, ist ja kein böser Energydrink. Und rät man Schwangeren vom Kaffeekonsum ab? Wäre mir neu. MBxd1 (Diskussion) 22:29, 24. Jan. 2016 (CET)
Ja, das mit denn „high cronic consumers“ die "4-5 mal wöchentlich zu einem Energy-Drink greifen" (ich denke, damit ist die übliche Dosierung von 250ml mit 80mg Koffein gemeint), ist natürlich schon etwas übertrieben, ich kenne aber Leute, die trinken so viel pro Tag und das ist sicher nicht mehr gesund, bei 4-5 Kaffee pro tag trinken aber auch manche und das ist sicher auch nicht mehr gesund... --MrBurns (Diskussion) 22:47, 24. Jan. 2016 (CET)
Nur interessiert letzteres niemanden, und das ist das Verlogene an der Aktion. Und wenn Jugendliche Kaffee trinken, ist auch alles in bester Ordnung. Tja, so funktioniert Lobbyismus. MBxd1 (Diskussion) 22:58, 24. Jan. 2016 (CET)

Vertragsschluss per eMail

Bei Miet- und Arbeitsverträgen dürfte das heute ja überwiegend der Fall sein: Man erhält die Zusage per eMail, mit dem Hinweis man bekäme den richtigen Vertrag zugeschickt. Jetzt die Frage, ist damit der Vertrag schon geschlossen wenn man die eMail erhält? Kann dann eine Seite einseitig die Unterschrift des eigentlichen Vertrags verweigern, etwa wenn im eigentlichen Vertrag eine nicht so erfreuliche Klausel steht, der Arbeitgeber oder Vermieter etwas neues über den Bewerber erfährt, man an der Wohnung etwas findet was einem nicht so passt, oder einfach weil man es sich anderes überlegt hat/etwas besseres gefunden hat?--Antemister (Diskussion) 13:52, 24. Jan. 2016 (CET)

Kommt drauf an.
Manche Verträge kommen bereits durch eine bloße Einigung zustande, also zwei sich inhaltlich aufeinander beziehende Willenserklärungen, ohne dass es dabei auf deren Form ankäme. Andere Verträge erfordern eine besondere Form (zum Beispiel Schriftform oder Textform - letztere wird durch eine E-Mail erfüllt). Formmängel können aber unterschiedliche Folgen haben: Entweder der Vertrag ist überhaupt nicht zustande gekommen, oder der Formmangel führt zu inhaltlichen Änderungen (so gibt es zum Beispiel ein Schriftformerfordernis für Mietverträge über mehr als ein Jahr - die Verletzung dieser Vorschrift führt aber nicht zur Nichtigkeit des Vertrages, sondern dazu, dass der Vertrag als auf unbestimmte Zeit geschlossen gilt, § 550 BGB).
Daher: Es kommt drauf an. --Snevern 14:10, 24. Jan. 2016 (CET)
Spielt nicht auch noch eine Rolle, dass in der E-Mail zwar die Zusage gegeben wird, aber oft nicht alle Vertragsbedingungen bekannt sind? --= (Diskussion) 14:29, 24. Jan. 2016 (CET)
Das spielt selbstverständlich eine Rolle, denn danach richtet sich, ob tatsächlich eine inhaltliche Einigung auf einen Vertrag zustande kommt. Zumindest die wesentlichen Bedingungen müssen also beiden Seiten klar sein, damit es zu einem wirksamen Vertragsschluss kommt. Ich habe schon oft erlebt, dass bei Miet- oder Arbeitsverträgen die schriftliche Ausfertigung erst nach Einzug oder Arbeitsaufnahme zur Unterschrift vorgelegt wurde - und dann von den Vorstellungen des Vertragspartners erheblich abwich. Dann hat man unter Umständen einen faktischen Vertrag - und damit weitere Probleme. --Snevern 14:43, 24. Jan. 2016 (CET)
Email kennt keine Unterschrift. Dem Papier wird heute nach wie vor vertraut. Es ist aber Üblich, Verträge zur Ansicht vorab zu überreichen. Ob Mail oder Papierform ist bei der Voransicht nicht relevant. Nachträgliche nicht abgesprochene Änderungen auf dem Papier zur Unterschrift gehören nicht zum guten Ton und können ein wichtiger Kündigungsgrund sein. Die Beweisführung ist aber fraglich, da Email nicht vertaut wird. --Hans Haase (有问题吗) 19:13, 24. Jan. 2016 (CET)
Die Unterschrift auf dem Papier ist eine Wissens- und Willenserklärung. --Hans Haase (有问题吗) 19:15, 24. Jan. 2016 (CET)
Nein, Hans. Ich würde ja sagen, halt dich an die Themen, von denen du was verstehst, aber während ich das schreibe..........
Überlest die beiden vorangehenden Postings sowie das hiernach folgende einfach. --Snevern 19:21, 24. Jan. 2016 (CET)
Im Artikel Unterschrift#Rechtliche Aspekte finde ich meine Aussage weitgehend bestätigt. Wenn ohne schriftlichen Vertrag etwas ausgemacht wird, kann es im konfliktlosen Fall– bei vollständiger Einigkeit – alles problemlos verlaufen. Im Streitfall kann das anders aussehen. Zwar ist der Ausgemachte Betrag und dessen Zahlung ein Indiz, dass es sich um die Abmachung handeln könnte. Wo der Hacken dabei liegt, wird uns Snevern bestimmt genauer erklären. --Hans Haase (有问题吗) 21:44, 24. Jan. 2016 (CET)
Für unterschiedliche Verträge gibt es unterschiedliche gesetzliche Formerfordernisse. Manche Verträge können formlos durch konkludentes Handeln erfolgen, andere erfordern Text- oder Schriftform oder müssen notariell beurkundet sein. Ein Arbeitsvertrag kann grundsätzlich auch formlos, sogar mündlich geschlossen werden. Zum Nachweis bei Sozialversicherung, Finanzamt, Unfallversicherung etc pp ist aber Schriftform sinnvoll. Auch beim Mietvertrag ist Schriftform sinnvoll, um späteren Streitigkeiten vorzubeugen und zur Vorlage beispielsweise bei der Wohngeldstelle oder dem Jobcenter. --Rôtkæppchen₆₈ 23:39, 24. Jan. 2016 (CET)

gesetze in Groß Britanien

Was für konsequenzen hat es wenn man gegen die regeln eines engländischen ladens bricht wie zum beispiel die tasche mit rein genommen hat oder mit mehr als zwei kinder rein gegangen ist? Was für konsequenzen hat das für mich einem toristen und einem normalen engländischem jugendlichen? --178.4.10.97 18:08, 21. Jan. 2016 (CET)

(In welchem englischen Laden gibt es denn solche Regeln?) Du wirst freundlich und in ruhigem Tonfall gebeten, den Laden zu verlassen (in Deutschland würdest du angebrüllt). Falls du Tourist bist, wird man sich noch dreimal dafür entschuldigen, die Regeln leider durchsetzen zu müssen. --Kreuzschnabel 18:57, 21. Jan. 2016 (CET)
In Deutschland würdest du angebrüllt!? Du gehst in die falschen Läden, Kreuzschnabel - definitiv. --Snevern 10:41, 22. Jan. 2016 (CET)
Kommt auf die Hautfarbe an.--80.129.152.205 13:00, 22. Jan. 2016 (CET)
Solange es gegen Deutschland geht, darf man wohl jeden Schwachsinn hetzen. Es kann natürlich irgendwo mal einen kuriosen Einzelfall gegeben haben, aber zumindest ich habe es noch niemals erlebt, dass in Deutschland jemand angebrüllt wird, weil er mit mehr als 2 Kindern in einen Laden geht. Noch nicht einmal, dass jemand "freundlich" gebeten wurde, den Laden aus solchen Gründen zu verlassen. Egal welche Hautfarbe. --84.119.204.210 17:55, 22. Jan. 2016 (CET)
Die 84er-IP hat mit dem ersten Satz sogar (nach umstrittener, aber herrschender Meinung) Recht. Thomas Fischer bezweifelt in seinem vielgenutzten StGB-Kommentar doch tatsächlich, dass "Opfer" einer Tat nach §130 I oder II StGB (Volksverhetzung in den verschiedenen Varianten) auch die Bevölkerungsmehrheit sein könnte. Einzig und allein ein Aufsatz von Mitsch wird als Beleg für die Gegenmeinung zitiert. Solange die herrschende Meinung mit Fischer geht, darf man juristisch tatsächlich "jeden Schwachsinn hetzen", wenn es gegen Deutsche geht... Übrigens sind "die Deutschen" oder gar "Deutschland" auch, mangels Bestimmbarkeit der beleidigten Gruppe, auch nicht (auch nicht unter einer Kollektivbezeichnung!) beleidigungsfähig.--ObersterGenosse (Diskussion) 22:30, 22. Jan. 2016 (CET)
Und das ist auch gut so, weil in Deutschland schließlich Meinungsfreiheit herrschen soll. Die sollte man sogar lieber ausweiten als einschränken, so dass im Idealfall jedermann ganz unbefangen sagen kann, was er sagen will - unabhängig von der Frage, welche Gruppe aktuell für vogelfrei "nicht beleidigungsfähig" erklärt ist. Das deutsche Strafrecht hatte ich aber gar nicht gemeint, denn WP hat seine eigenen (überwiegend ungeschriebenen) Hausregeln, die doch normalerweise zu unverzüglichen pädagogisch wertvollen Löschungen, Sperrungen oder zumindest "Erlen" führen, sobald jemand über Völker, Ethnien oder Religionen irgendeinen einen Quatsch behauptet.... --84.119.204.210 01:25, 23. Jan. 2016 (CET)
+1. Zudem: Das Strafrecht soll (unter anderem auch) die Schwächeren vor den Stärkeren schützen. Dieser Bedarf besteht bei den Stärkeren nicht, weil sie sich schon alleine schützen können. Von daher ist es durchaus sinnvoll, für den Schutz von Minderheiten und Mehrheiten unterschiediche Maßstäbe anzulegen. --Jossi (Diskussion) 11:06, 23. Jan. 2016 (CET)
Das ist offenbar sehr kurzsichtig gedacht, denn am wichtigsten ist es, die Mehrheit dabei immer auch bei Laune zu halten. Die Mehrheit ist es nämlich, die dieses Strafrecht auch ändern oder sogar den Rechtsstaat ganz in die Tonne treten kann, wenn es ihr zu blöd wird. Dass die Mehrheit sich schon "alleine", also ohne diesen Rechtsstaat, schützen kann, ist ja richtig, aber intelligenter wäre es, dafür zu sorgen, dass es nicht so weit kommen muss, indem man auch der Mehrheit ihren Schutz nicht sich selbst überlässt.. DIe deutschen "Eliten" scheinen aber gerade ein gefährliches Experiment genau in diese Richtung zu machen. --84.119.204.210 13:20, 23. Jan. 2016 (CET)
Die sogenannten Eliten sind eine „Mehrheit“ (nicht numerisch, soziologisch gesehen natürlich). Andererseits: die sogenannten „Gutmenschen“, die die armen, armen „Mehrheiten“ angeblich mit ihrer „politischen Korrektheit“ „terrorisieren“, sind definitiv keine mächtige Gruppe und haben auch keine sonderlich einflußreichen Lobbys, eher im Gegenteil, auch wenn neurechte Verschwörungstheoretiker nicht müde werden, eben dies zu behaupten. Diverse gesellschaftlich systematisch benachteiligte Gruppen haben es lediglich durch jahrzehntelangen Aktivismus und beständige Argumente wiederholt geschafft, daß der Gesetzgeber sich ihrer endlich erbarmt und ihre Interessen berücksichtigt hat. Wirklich mächtige Lobbys erreichen deutlich mehr für ihre Klientel als den Schutz von Grundrechten. Im Gegenteil: Es gibt zahlreiche einflußreiche Interessengruppen, die politisch reaktionär sind und die hart erkämpften Rechte von Minderheiten am liebsten wieder rückgängig machen würden. Nur in den Fieberträumen „besorgter Bürger“ werden „aufrechte Deutsche“ regelmäßig von „Flüchtlingen“, „Homos“, „Transen“ und ihren „linksradikalen Handlangern“ beschmipft, bespuckt, drangsaliert und gejagt anstatt umgekehrt. Und den Rechtsstaat einfach ganz abzuschaffen, nur weil ein paar Minderheiten etwas zu aufmüpfig wirken und sich nicht mehr so leicht unterdrücken lassen wie dereinst, wäre auch für die mächtigsten gesellschaftlichen Gruppen mit einigen Unannehmlichkeiten verbunden und damit nicht nur ein „gefährliches Experiment“, sondern eine gewaltige Eselei. Jammereien über die ach so unterdrückte Mehrheit funktionieren nur unter Strömen von Krokodilstränen. --Florian Blaschke (Diskussion) 14:24, 24. Jan. 2016 (CET)
Es ging nicht um angeblich unterdrückte Mehrheiten, sondern um die Behauptung von Jossi, dass die Starken bzw. die Mehrheiten angeblich nicht vom Rechtsstaat geschützt werden müssen, weil sie sich ja selbst schützen können oder sollen. Darüber soll man meines Erachtens mal nachdenken, ob diese Argumentation tatsächlich sinnvoll ist. Dass die Nichtminderheiten in Deutschland für ihren Schutz selber sorgen müssen, haben ja jetzt wohl alle begriffen, spätestens nach der Kölner Silversternacht, wie die sprunghaft angestiegenen Anträge auf Waffenscheine und die ausverkauften Selbstverteidigungsutensilien beweisen. Aber ich verstehe nicht, warum das so gewollt ist, denn m.E. überwiegen dabei ganz klar die Nachteile für alle Beteiligten - außer für die Waffenlobby vielleicht. --84.119.204.210 17:51, 24. Jan. 2016 (CET)

Du meinst sicher Kleine Waffenscheine? --Heletz (Diskussion) 18:00, 24. Jan. 2016 (CET)

Ja, jedenfalls ist über die kleinen Waffenscheine viel berichtet worden in den letzten Tagen. Aber das ist total irrelevant, wie "groß" die Waffen aktuell sind, mit denen sich die angeblich nicht schützenswerten Mehrheiten gegen die Minderheiten ausstatten soll, um für ihren Schutz selber zu sorgen. Es geht ums Prinzip. --84.119.204.210 18:14, 24. Jan. 2016 (CET)
Ok. Dennoch schreibt man "Kleine Waffenscheine"! Mit großem "K"! Weil es ein feststehender Begriff aus dem WaffG ist. Aber es ist erstaunlich, wie man von einem Laden in GB auf Kleine Waffenscheine in D kommt. Deutsche beziehen halt immer alles gleich auf sich *rofl* --Heletz (Diskussion) 18:18, 24. Jan. 2016 (CET)

Da heutzutage in der Regel keine Minderheiten insgesamt (und erst recht keine Mehrheiten) angegriffen werden, sondern nur Individuen, die bestimmten Minder- bzw. Mehr-heiten zugeordnet werden können, ist "eine Mehrheit" nicht imstande, "sich selbst zu verteidigen"- dem steht das Gewaltmonopol des Staates entgegen. Vielmehr hat jedes Individuum ein Anrecht darauf, gegen rechtswidrige Angriffe anderer Individuen geschützt zu werden, ganz unabhängig davon, welchen gesellschaftlichen Gruppierungen die Beteiligten zuzuordnen sind. (Wenn man sich ordentlich Mühe gibt, kann man IMO praktisch für jeden Einzelnen irgendeine passende Minderheit konstruieren - z.B. selbständiger Linkshänder mit Angelschein und Opernabo?). -- Zerolevel (Diskussion) 22:37, 24. Jan. 2016 (CET)

Das ist jetzt in eine ganz andere Richtung abgeirrt, als ich ursprünglich (wohl etwas missverständlich formuliert) gemeint hatte. Selbstverständlich muss das Strafrecht alle Menschen, ohne Ansehen der Person oder irgendwelcher Gruppenzugehörigkeiten, in gleicher Weise vor Angriffen auf Leben, körperliche Unversehrtheit, sexuelle Selbstbestimmung, Eigentum usw. schützen. Dazu gibt es das staatliche Gewaltmonopol, so dass Selbstbewaffnung zur Selbstverteidigung in aller Regel weder notwendig noch wünschenswert ist. Mein ursprünglicher Diskussionsbeitrag bezog sich aber auf die Ausgangsfrage, warum man „Deutschland“ oder „die Deutschen“ nicht beleidigen kann. Bei solchen Strafvorschriften, die bestimmte Verhaltensweisen gegenüber Gruppen (nicht Individuen) untersagen, macht es sehr wohl einen Unterschied, ob sie die Minderheit vor der Mehrheit oder die Mehrheit vor der Minderheit schützen (sollen). Die Volksverhetzung im deutschen Strafrecht oder die (schärfere) Bestrafung von hate crimes in den USA dient faktisch dem Schutz von Minderheiten, während etwa die Beleidigung des Türkentums und analoge Paragraphen im Strafrecht Russlands, Chinas oder arabischer Staaten der Unterdrückung von Minderheiten dienen. --Jossi (Diskussion) 12:29, 25. Jan. 2016 (CET)

Woher kommt das Licht aus einer Glühbirne

Der Draht einer Glühbirne emittiert ein thermisches Spektrum. Die Temperatur liegt bei etwa 2000K, und die Energien der Photonen irgendwo bei ein paar eV. Aber wo sind eigentlich die elektronischen Übergänge, die das Licht emittieren? Anders gefragt, wie entsteht das thermische Spektrum?

Licht kann in Metalldraht nur mehrere Wellenlängen propagieren, d.h. wir "sehen" nur die direkte Oberfläche des Drahtes. Werden im Inneren des Drahtes Photonen erst erzeugt (evtl aus Bandübergängen im Festkörper?), aber dann kontinuierlich absorbiert und emittiert, bis sie die Oberfläche erreichen? Was in dem Festkörper hat einen Skala von mehreren eV, was für das Licht relevant wäre? --186.217.213.100 19:28, 22. Jan. 2016 (CET)

Photonen entstehen doch immer dann, wenn n Elektron von einer höheren Umlaufbahn(?) zurück auf seine Original-Umlaufbahn hüpft... oda? je größer der Sprung umso höher die Energie/Frequenz des Photons? dann nehm ich mal an, dass durch die starke Erwärmung die Elektronen wild herumhüpfen... da sagen die das auch... --Heimschützenzentrum (?) 19:38, 22. Jan. 2016 (CET)
Ein Atom hat verschiedene Elektronenschichten (ausser Wasserstoff und Helium die nur die K-Schale besitzen, doch das ist hier ist unrelevant hier, da wir Metalle verwenden wie Wolfram), die K-Schale, die L-Schale usw. Diese Schalen haben verschiedene Energien. Der Draht ist zu einer Spindel geformt und erhitzt sich wenn der Strom fliesst, durch den Joul'schen Effekt (E=r*I2*dt). So erreicht der Draht hohe Temperaturen und beginnt vor Hitze zu glühen. Nun, meine Erklärung mit den Atomschalen gilt für Neon-Lampen, wo ein Kondensator Ladung speichert und sich dann ins Rohr freisetzt, was den Elektronen Energie verleiht und sie in eine höhere Schale aufsteigen (K --> L oder L --> M usw.). Dies ist jedoch ein instabiler Zustand, somit setzt das Elektron die erhaltene Energie frei in Form einer elektromagnetischen Welle, die wir als Licht sehen. So, wie hilfreich ist meine Erklärung? (Ich könnte darüber ein Video auf Youtube machen.)--Skyscraper1996 (Diskussion) 19:45, 22. Jan. 2016 (CET)
(mehrere BKs) Ja, im inneren werden Photonen emittiert und gleich wieder absorbiert, weil die Absorptionsrate und die Emissionsrate wegen der (näherungsweise) gleichmäßigen Temperaturverteilung und dem (näherungsweise) gleichmäßigen Kristall identisch sind. An der Oberfläche findet dann die Abstrahlung statt. Siehe auch Wärmestrahlung, Schwarzer Körper und Grauer Körper letzteres ist eine Verallgemeinerung des schwarzen Körpers und beschreibt den Glühdraht genauer). Das Energiespektrum geht übrigens theoretisch von 0 bis unendlich. ich denke dass die Strahlung nur von der Oberfläche weggeht ist auch der Grund, warum der Draht relativ dünn ist: damit kann man die Oberfläche im Vergleich zum Volumen und zur Masse erhöhen, da mit der Dicke des Drahtes das Volumen quadratisch ansteigt (wenn die Länge gleich bleibt), die Oberfläche aber nur linear. --MrBurns (Diskussion) 19:49, 22. Jan. 2016 (CET)
Es bildet sich auch ein Magnetfeld im Gewinde. Und ja, die Erregung der Elektronen gilt natürlich auch für die Glühbirne an sich.--Skyscraper1996 (Diskussion) 19:59, 22. Jan. 2016 (CET)
Übrigens, bei mir war bei meinem ersten Beitrag in diesem Abschnitt auf ein Bearbeitungskonflikt.--Skyscraper1996 (Diskussion) 20:02, 22. Jan. 2016 (CET)
@Homer, Skyscraper1996: entstehen die Photonen wirklich nur durch Elekronenübergänge? Und wenn dem so ist, wie erklärt man sich dann das kontinuierliche Lichtspektrum, obwohl für Elektronen nur diskrete Energieniveaus zulässig sind? --MrBurns (Diskussion) 20:11, 22. Jan. 2016 (CET) PS: ich glaub ich hab die Antwort im Artikel Bändermodell gefunden: in Festkörpern gibt es einfach wegen der Kopplung zwischen den Orbitalen der einzelnen Atome extrem viele mögliche Energieniveaus, sodass man das Spektrum nicht mehr von einem kontinuierlichen unterscheiden kann. Wie genau die Photonen bestehen ist für das Wärmestrahlermodell übrigens irrelevant, daher ist das mit den Elektronenübergängen auch kein Widerspruch zum Modell eines Wärmestrahlers. --MrBurns (Diskussion) 20:18, 22. Jan. 2016 (CET)
(BK) Ne, auch durch die Erhitzung wie oben erklärt. Die Neonlampen jedoch leuchten nur durch die Photonen der Elektronenübergänge. Daher ist das Lichtspektrum diskontinuierlich. Soll ich noch mehr sagen?--Skyscraper1996 (Diskussion) 20:24, 22. Jan. 2016 (CET)

Die Strahlung einer Glühlampe kann man doch einfach nur als eine Wärmestrahlung im sichtbaren Bereich interpretieren. Wärmestrahlung entsteht durch die thermische Bewegung der Atome, was automatisch auch zu beschleunigten Ladungen führt und damit zu einer Abstrahlung. --Engie 20:43, 22. Jan. 2016 (CET) Nachtrag: Siehe auch die Einleitung von en:Thermal_radiation, die mir deutlich besser als in unserem Artikel Wärmestrahlung gefällt. --Engie 20:47, 22. Jan. 2016 (CET)

wie jetzt? sichtbare Photonen können auch einfach nur durch Wärme entstehen? ohne dass ein Elektron wieder zurück hüpft? --Heimschützenzentrum (?) 22:58, 22. Jan. 2016 (CET)
Ja, genau. Es gibt beide Phänomäne in der Glühlampe, in einer Neonlampe aber nur die der Elektronen, da keine Erhitzung vorhanden ist.--Skyscraper1996 (Diskussion) 16:39, 23. Jan. 2016 (CET)
Wie läuft das im Metall ab? Die Atomrümpfe sind geladen, die Elektronen sind delokalisiert und können sich frei bewegen. Dazu gibt es Phonone, quantisierten Schwingungen der Atome, die thermisch getrieben sind. Die Elektronen tragen fast überhaupt nicht nicht zur Wärmekapazität bei, es gibt also wenig Scattering: innerhalb der Fermi-Fläche gibt es keine Zustände, in die die Elektronen streuen könnten. Streuen die Elektronen an den Gitterschwingungen, und dabei werden Photone frei, oder erzeugen die schwingenden Atomrümpfe die Strahlung? Vielleicht übersehe ich etwas simples, aber das Problem kommt mir ziemlich schwer vor. Ein Modell müsste ja Elektronen, Photonen und Phonen enthalten. Das würde außerdem bedeuten, dass die Elektronen fast irrelevant für die Erzeugung des Lichts sind; es von den Atomrümpfen ausgeht. Oder ist das falsch? --186.217.213.100 12:40, 25. Jan. 2016 (CET)

Woher stammen die Geräusche?

Was sind das genau für Geräusche in den zwei folgenden Videos und warum kommen sie zustande?
https://www.youtube.com/watch?v=QYHCLygZcEI & https://www.youtube.com/watch?v=Tn8oC7Mqw8U
Améliorateur (Diskussion) 16:50, 23. Jan. 2016 (CET)

Vom Eis, aber ich lasse lieber jemand anderes das genau erklären.--Skyscraper1996 (Diskussion) 16:54, 23. Jan. 2016 (CET)
Okay, danke schon mal.--Améliorateur (Diskussion) 16:56, 23. Jan. 2016 (CET)
Ich kann sie nicht hören (...), aber bei dicker Eisdecke kann man dumpfe (Ent)Spannungsgeräusche hören. Und bei noch dünner Eisdecke - wenn man einen Stein darauf wirft - ein "hohes Zwitschern" (auch Spannungsvibrationen der Eisdecke]. Play It Again, SPAM (Diskussion) 17:08, 23. Jan. 2016 (CET)
Ich schon, man muss nur die Ohren gut offen halten. Also, ich sage nun doch was ich darüber denke. Die Eisdecke ist wahrscheinlich noch dabei, sich zu schliessen, denn da wurde kein Stein geworfen, also ich sehe keinen und höre keinen, da kann das Geräusch gut so entstehen (oder eher die Geräusche).--Skyscraper1996 (Diskussion) 17:14, 23. Jan. 2016 (CET)
Ach ja, die Spannungsvibrationen von der sich schliessenden Eisdecke würde ich sagen.--Skyscraper1996 (Diskussion) 17:15, 23. Jan. 2016 (CET)
Hier nett zusammengefasst, hier Frequenz/Dichtedaten. Das mit dem Stein ist nur eine "induzierte Variante". Wir haben nur Eisstoß. Play It Again, SPAM (Diskussion) 17:33, 23. Jan. 2016 (CET)
Danke für die Antworten.--Améliorateur (Diskussion) 17:46, 23. Jan. 2016 (CET)
Manche nennen es "Eismusik". Wobei die teilweise schon eher gewöhnungsbedürftig ist. Dein See ist da sehr zurückhaltend. --j.budissin+/- 18:40, 23. Jan. 2016 (CET)
Es handelt sich um den Schwarzsee, denn im Titel steht "Lac Noir" und beide Videos stammen vom 26.12.2015, da muss man mal das Wetter von da anschauen und man stellt fest, dass es eigentlich sonnig und mild ist. Doch gerade wegen diesem warmen und ruhigen Wetter ist der Oeschinensee, der sich bei Frutigen befindet tief gefroren mit einer Eisdecke von 14 cm und er ist nicht sehr viel höher als der Schwarzsee, so denke ich mir mal um die 5 cm Eis, was begehbar ist.--Skyscraper1996 (Diskussion) 13:57, 25. Jan. 2016 (CET)

Verwandtschaftsverhältnisse nach Ehe unter Cousins

Ja, eine ungewöhnliche Frage, aber hier ist sie: In manchen Kulturkreisen sind Beziehungen zwischen Cousin und Cousine weit verbreitet. Wenn aus einer solchen Beziehung Kinder hervorgehen, sind diese Kinder untereinander dann näher verwandt als normale Geschwister? Geschwister teilen ja für gewöhnlich 50% des Erbguts, Halbgeschwister 25%. Ist der Verwandschaftsgrad hier höher als 50%? --85.183.206.146 03:01, 24. Jan. 2016 (CET)

Vielleicht hilft dir der Artikel Kreuzcousinenheirat weiter? --Joschi71 (Diskussion) 03:10, 24. Jan. 2016 (CET)

Die meisten Menschen teilen nahezu 100% des Erbguts, zumindest wenn sie dasselbe Geschlecht haben. Nur sehr wenige Gene unterscheiden sich individuell. Diese sind dann typischerweise für die optischen Unterschiede, einzelne natürlich auch für andere Unterschiede (Blutgruppe, Gendefekte etc.) zuständig. Geschwister sind meist nur näher an den 100% dran als nicht verwandte Menschen. Geschwister sind also nicht zu 50% verwandt. Sie haben jedoch 50% der Gene unmittelbar von den gleichen Vorfahren geerbt. 50% ist hier natürlich nur ein Mittelwert. Der Zufall spielt eine grosse Rolle. Wenn die Eltern nah verwandt sind (Cousin/Kusine), dann sind deren Kinder natürlich noch näher verwandt, also noch näher an 100% dran. Allerdings ist auch die Wahrscheinlichkeit für Erbschäden grösser, da ein defektes Gen, das in den meisten Fällen nur auf einem der beiden entsprechenden Chromosomen vorkommt und viele Menschen in sich tragen, dann von beiden Elternteilen vererbt wird. Viele Gene sind ja nur dann gefährlich, wenn sie auf beiden Chromosomen defekt vorliegen. Eine sexuelle Beziehung unter Verwandten ist daher meist keine gute Idee, zumindest wenn sie im Kinderkriegen mündet. 188.121.0.82 07:44, 24. Jan. 2016 (CET)

Nein, sind sie nicht. Aber die genetische Variation kann geringer sein.
Beispiel:
Bei "entfernten Eltern" hast du dasselbe Gen (XYZ) aber mit - sagen wir mal - zwei unterschiedlichen Punktmutationen. Eine "harmlose Mutation" kann also für eine "schädliche Mutation" kompensieren.
Bei familiär "nahen Eltern" ist die Chance höher, dass rezessive Bedingungen vorliegen. Beide Eltern kommen mit derselben Mutation dahergewackelt, was zu Problemen führen kann (und statistisch führt).
Bonusrunde (angeregt durch "Eine sexuelle Beziehung unter Verwandten ist daher meist keine gute Idee, zumindest wenn sie im Kinderkriegen mündet.") : In der genetische/genealogischen Island-Studie wurden 160.000 Ehepaare (innerhalb von 200 Jahren) untersucht. Cousin/Cousinen-Ehe 1. Grades waren problematisch und hatten relativ wenige erfolgreiche Kinder und Enkel; 2. Grades war da schon besser (da hat WP Artikel zu Familien, bei denen das Tradition war); optimal für die Anzahl des fortpflanzungsfähigen Nachwuchses war 3. und 4. Grades! Danach fiel es wieder ab ("zu weit entfernt" kann auch wieder (Kompatibilitäts)Probleme geben. Gynäkologen können das bestätigen, tun sie aber nicht gerne, weil das oft als politisch unkorrekt interpretiert wird [aber nur von Leuten, die die Komplexität von DNA und Genetik vermeiden...]).
Also ... ab und zu mal bei den grossen Familienversammlungen vorbeischauen und nach Cousinen 3. oder 4. Grades fragen (wenn man am Totenbett von Kindern, Enkeln und Urenkeln umringt sein will...). Play It Again, SPAM (Diskussion) 10:20, 24. Jan. 2016 (CET)
Biologisch gesehen häufen sich die genetischen Eigenschaften der gemeinsamen Vorfahren wie der Artikel es auch beschreibt. Die Wahrscheinlichkeit für Erbkrankheiten, auch wenn sie in der Natur ihren Sinn haben oder hatten, häuft sich. Andere Beispiele: Mischlinge unter Hunden haben nicht selten höhere Intelligenz. Die Greencard der USA heißt „Diversity Program“, zu Deutsch auch mit „Vielfaltskonzept“ übersetzbar.[15][16] Sieger schreiben eben Geschichte. Amerika ist erfolgreich. Russland ist so groß, dass derartige Effekte auch im Land selbst auftreten können. Das Politische Resultat: Deutschland war nicht divers und hat seine Einwanderung bisher stets ideologisiert. Auch die Teilnahme an der DV-Lotterie (Greencard) der USA schließt anteilig übervölkerte Nationalitäten nach deren Herkunftsländer aus. Die Chinatowns sollen nicht im Wildwuchs vergrößert werden, aber eine kulturelle Beziehung, die auch wirtschaftliches mit sich bringen kann, soll gegeben sein. --Hans Haase (有问题吗) 11:20, 24. Jan. 2016 (CET)
Der Fragesteller meint den Verwandtschaftskoeffizient. Nachdem Cousine und Cousin einen Verwandtschaftskoeffizient von 1/8 haben, haben die gemeinsamen Kinder einen Verwandtschaftskoeffizient von 1/2 * (1 - 1/8) + 1/8 = 9/16 = 56,25%. Usr2 (Diskussion) 13:21, 24. Jan. 2016 (CET)
Endlich eine zielführende Antwort! @Hans Haase: Die innere Logik deine verbalen Darbietung erschließt sich mir nicht. --84.135.146.132 18:34, 24. Jan. 2016 (CET)
Ich zielte darauf, ab wann es Inzest ist und wer ab wann das darunter versteht. Biologische Vererbung wurde historisch unrichtig gelehrt und dieses Unwissen hielt sich lange, so dass es heute noch vereinzelt thematisiert wird. In geschlossenen Gruppen werden mit den Generationen die Gene ähnlicher. → Die evolutionäre Gleichmacherei existiert. Ich habe den obigen Beitrag nachgebessert, danke für den Hinweis. Es sei noch angemerkt. Die, die von der DV-Lotterie ausgeschlossen sind, können ein Visum erhalten, wenn sie investieren oder Arbeitsplätze schaffen. Aber der „Ghettoisierung“ soll vorgebeugt werden. Ein Stopp für Exzellenz ist es nicht. Darin sieht erkennt man die Fehler in Deutschland und wundert sich über die fundamentale und überzogene, aber berechtigte Kritik. War es das oder was genau ist unklar? --Hans Haase (有问题吗) 22:03, 24. Jan. 2016 (CET)

Nur kleiner Einwand, aber historisch unterschied man in einigen Kulturen streng zwischen der Herkunftslinie nach Geschlecht. Das resultiert schlicht aus der Unkenntnis biologischer Abläufe aber auch der Beobachtung. So waren Cousinehen mit den Verwandten des Vaters unproblematisch, während Ehen in die Familie der Mutter ungern gesehen oder Tabu waren. Siehe dazu auch Oheim und Muhme, keine Ehe mit deren Kinder, dafür Bint ʿamm.Oliver S.Y. (Diskussion) 15:51, 25. Jan. 2016 (CET)

Die ursprüngliche Men in Black-Sage

 
Bild der schwarzen Helikopter und der zugehörigen Men in Black siehe hinter dem Herrn mit mit der Blitzdingsbrille
 
Mann in Schwarz


Hallo, so eine ähnliche Frage gab es schon einmal, dennoch stelle ich sie mal hier: Es ist ja eine mehr oder weniger verbreitete Theorie, dass sie Aussagen von UFO-Zeugen usw. sich durch popkulturelle Einflüsse verändern, dass Bild der "kleinen grünen Männchen" ("Grey", bei einigen UFO-Forschern) soll etwa durch eine künstlerische Arbeit verbreitet sein, auch häufen sich angeblich UFO-Sichtungen nach erfolgreichen Filmen über das Thema. Meine Frage wäre jetzt: Wie beschrieben eigentlich die ursprünglichen "Men in Black"-Zeugen die Begegnung mit den geheimnisvollen Männern in Schwarz, bevor das Thema durch die Medien und Filme aufgegriffen und populär gemacht wurde? Ich lese häufig davon, die Opfer haben berichtet, dass die Männer alltägliche Gegenstände wie Kugelschreiben nicht kannten und Joghurt für ein wertvolles Geschenk hielten, aber das ist wahrscheinlich aus dem Wikipedia-Artikel entnommen und dadurch verbreitet. --178.6.50.64 12:37, 24. Jan. 2016 (CET)

Du glaubst echt, es gab ursprüngliche Men-in-Black-Zeugen, auf die die Sage zurückgeht? Hat bei dir die Behandlung nicht gewirkt? --Snevern 12:44, 24. Jan. 2016 (CET)
Danke für die Antwort. Egal auf welche Behandlung angespielt wird, ob Ironisch auf das Dingsblitzen im Film oder eine andere "Behandlung" gegen Paranoia: Ich fand es nicht besonders lustig. Ich meine damit natürlich nicht, dass an den Zeugenaussagen etwas dran ist, aber im Artikel steht ja z. B. Ein möglicher Ursprung des Mythos ist am ehesten in der Zeit zu suchen, in der die ersten Berichte über „Männer in Schwarz“ veröffentlicht wurden. (Der Satz hört sich übrigens ziemlich lustig an.) (nicht signierter Beitrag von 178.6.50.64 (Diskussion) 13:01, 24. Jan. 2016 (CET))
<dazwischenquetsch> Selbstverständlich bezog ich mich auf das Blitzdingens - von irgendwelchen sonstigen Behandlungen kann ich bei einer mir unbekannten IP nichts wissen, sie gingen mich auch nichts an und ich würde mich nie darüber lustig machen. Ich hoffe, du akzeptierst meine Entschuldigung.
Beim näheren Betrachten finde auch ich die Aussage lustig, der Ursprung sei wohl am ehesten in der Zeit zu suchen, in der die ersten Berichte auftauchten. Eine andere Zeit (insbesondere die Zeit vor dem Auftauchen der ersten Berichte) wäre da schon wesentlich spannender. --Snevern 13:22, 24. Jan. 2016 (CET)
So wird es auch im en:WP-Artikel mit Jahresangabe erklärt.
Zum Prinzip: Personengruppen in einheitlich gefärbter Kleidung haben immer schon besondere Gefühle hervorgerufen (Die braunen Herrenrasse, Momos graue Herren, Hare-Krishnas, Polizistengrüne, Soutanenträger, etc...). Phase I: Dann erfinden um 1947 ein paar Herren etwas, was sich an das Aussehen von Regierungsbeamten anlehnt (die ja auch schon mal geheimnisvoll tun und merkwürdige Forderung stellen). Phase II: Andere dafür empfängliche Personen (die durch die Intensität ihrer Bezeugung sehr eindrucksvoll sein können {siehe auch Religion}) ordnen das dann in ihre phantasievoll in ihre Mustererkennung ein. Phase III: Jeder spinnt den Mythos ein kleines bisschen weiter - und eine moderne Legende ist geboren (von der die Berichter ein Teil sind (= Ansehen bekommen). Hier wird es ganz gut erklärt. Play It Again, SPAM (Diskussion) 13:12, 24. Jan. 2016 (CET)
Also mir erklärt das verlinkte Bildchen jetzt nicht so wahnsinnig viel ... --77.179.59.62 14:41, 24. Jan. 2016 (CET)
Dann hast du im entscheidenden Moment nicht genau hingesehen. --Snevern 14:44, 24. Jan. 2016 (CET)
Ahnungsloser Mensch, der ich bin, mache ich mich schlau und die Wikipedia klärt mich auf, wozu das Ding gut ist. Wieder eine Wissenslücke geschlossen. --77.179.59.62 14:53, 24. Jan. 2016 (CET)

Ich erinnere mich an die Man in Black Geschichte noch vor den Filmen. Ursprünglich waren damit Regierungsvertreter gemeint die nach UFO Vorfällen auftauchten um alles zu vertuschen. Dieses ganze Gedöns das die MIB selbst geheimnisvolle nichtmenschliche Wesen seien kam wohl aus einer Mischung von den eher esoterischen UFO Foren (Jo Conrad und so) und X Files zustande. Die Ufologie hat zwei Seiten. Auf der einen finden sich Leute die das Thema mit durchaus wissenschaftlichen Sachverstand angehen (ob Sie recht haben oder nicht sei mal dahingestellt), auf der anderen Seite finden sich Esospinner die jeden Stuss glauben so lange er nur doof genug ist. Da es bei Themen wie Ufologie aber schwer ist zu differenzieren wer jetzt zu den ernst zu nehmenden und wer zu den vollkommen Verrückten zählt ist es schwierig in solchen Artikeln nur Ansichten der vernünftigen UFOlogen wieder zu spiegeln. --87.140.192.4 15:42, 24. Jan. 2016 (CET)

Zum guten Schluss findet sich dann noch eine nicht ganz unbekannte online-Enzyklopädie, die dem Schwachsinn mit Artikeln, die einem die Tränen in die Augen treiben, Vorschub leistet. Ufologie ist hier eine Protowissenschaft (nb: proto-, nicht pseudo-). Im übrigen „wurden in den USA in der Tat in Schwarz gekleidete Regierungsbeamte unterschiedlicher Behörden dazu angehalten, vorgebliche Zeugen von Ufo-Sichtungen aufzusuchen, um möglicherweise unerlaubt gesammeltes, für die Öffentlichkeit ungeeignetes Material zu beschlagnahmen.“ Selbstverständlich ordnungsgemäß belegt mit Walter Hain: SAETI: Die Suche nach Artefakten von außerirdischen Intelligenzen. BoD, Norderstedt 2013, und Judit Laczkó: UFOs und Außerirdische: Eine religionssoziologische Untersuchung des Milieus um das Phänomen der fliegenden Untertassen. diplom.de-Verlag. --Vsop (Diskussion) 16:30, 24. Jan. 2016 (CET)

Das richtige englische Lemma ist en:Black helicopter, das deutsche Men in Black (Verschwörungstheorie) ist nichtssagend. Der ideologische Hintergrund beruht auf der amerikanischen Staatsfeindlichkeit und deren extremen Formen und der bei bestimmten Evangelikalen (Christlicher Zionismus) verbreiteten Ansicht, die in der Offenbarung des Johannes vorausgesagten Abläufe hätten mit der Gründung Israels 1948 begonnen und die Wiederkehr Christi und das Jüngste Gericht würde in unserer Gegenwart geschehen. Ein zugehörige Bestseller stammt von en:Hal Lindsey, en:The Late, Great Planet Earth und ist aus dem Jahr 1969. Orson Welles (ja, der!) verfilmte Lindseys Buch 1979. 'Black Helicopters' von en:Jim Keith war Bestseller in den 1990ern. Men in Black lief damals in den Kinos. Die Europäische Union wird dabei in die Offenbarungserzählung als kommenden Hure Babylon eingefügt, als völlig amoralischen Großmacht des Antichristen. (Köln wird bei Teaparty und CO grad riesig aufgeblasen und als Hexensabbat mit 1000 wildgewordenen Männern verkauft). Für die 80er Jahre wurde schon der ganz große Krieg vorhergesagt (war nicht mal so ohne). Für die USA sehen die entsprechenden Schriften eine Übernahme durch Abgesandte des Antichristen mit schwarzen Hubschraubern (und den Herren in Schwarz) vor, was den 'Heuschrecken aus dem Rauch' der Offenbarung entsprechen soll. Die Milizbewegung in den Vereinigten Staaten (die haben andere Schützenvereine als wir) stellt sich dagegen mit Waffengewalt auf. Aktuell reichts nur für einige hier auch medial wiedergegebene Landbesetzungen. Fazit: Der UFO Aspekt bei den Men in Black ist nur ein kleiner Teil einer milleniaristischen Gesamterzählung von Bedeutung in den USA, solide Studien u.a. bei Michael Hochgeschwender und dem Springerhandbuch Verschwörungstheorien. Polentarion DiskTebbiskala : Kritik 05:56, 25. Jan. 2016 (CET)

Hier ist eine Übersetzung notwendig. Die amerikanische Staatsfeindlichkeit heißt nur in Deutschland so. In Amerika wird das gerne Freiheit und Eigenverantwortlichkeit genannt. Ein in Deutschland fast unbekanntes Konzept. --84.149.232.44 10:39, 25. Jan. 2016 (CET)
Men in Black seit 1947
Black Helicopter ab etwa 1976
Hier amüsante Zusammenfassung mit vielen Stichworten. Ich wusste beispielsweise nicht, dass sie durch Matsch laufen können, ohne ihre Schuhe dreckig zu machen. Lotuseffekt?
BoD-Referenzen sollten aus Artikeln herausgenommen werden. Play It Again, SPAM (Diskussion) 10:44, 25. Jan. 2016 (CET)

Firmenmotorrad?

Laut Zulassungsstatistik [17] hat ein beliebtes Supersportmotorrad eine Quote von 44,2%, was die Zulassung auf juristische Personen angeht. Das ist außergewähnlich. Erstens, weil man sich nicht einfach ein "Geschäftsmotorrad" vom Arbeitgeber geben lasen kann, so wie es bei "Dienstwägen" der Fall ist. In nahezu allen Fällen ist so eine Idee aus Sicht der Finanzämter aussichtslos. Dementsprechend gering sind in der Regel auch die Zulassungen auf Firmen. Die bewegen sich üblicherweise im einstelligen Bereich. Und dort handelt es sich dann meist um Fahrschulmotorräder und Polizeimotorräder. Bei der angesprechenen BMW S 1000 RR verstehe ich aber gar nichts mehr. Fast die Hälfte der Zulassungen auf juristische Personen. Da es sich um eine absolute Supersportmaschine handelt, fallen Fahrschulen und die Polizei mit wenigen Ausnahmen aus. Es gibt auch wenig bis gar keine Motorradverleiher wie Sixt oder Europcar. Meine Frage lautet nun: Was könnten das für juristische Personen sein, die eine Supersportmaschine wie die S 1000 RR auf sich zugelassen bekommen? --95.115.139.36 15:43, 24. Jan. 2016 (CET)

Google BMW S 1000 RR gewerbliche zulassung, bei mir 7. und 8. Treffer. --Rôtkæppchen₆₈ 15:48, 24. Jan. 2016 (CET)
Konnte auf der ersten Seite nichts finden, was meine Frage beantwortet. --95.115.139.36 21:23, 24. Jan. 2016 (CET)
Meine Treffer fühten die hohen gewerblichen Zulassungen auf Händler- und Vorführzulassungen zurück. --Rôtkæppchen₆₈ 23:45, 24. Jan. 2016 (CET)
(BK) Guck dir doch mal an, wer die Maschinen fährt, dann kommst du selbst drauf. In Ehren ergraute und mit der eigenen Firma zu Geld gekommene, aber im Geiste jung gebliebene Herren mittleren (ähem..) Alters. --Snevern 15:50, 24. Jan. 2016 (CET)
Auch ist die Gesamtzahl der Zulassungen für dieses Modell ja nicht gerade enorm hoch. Da reicht es wenn eine Firma (warum auch immer) ein paar hundert Stück bestellt, um den Anteil ordentlich in die Höhe zu treiben. --MrBurns (Diskussion) 21:57, 24. Jan. 2016 (CET)
Hm, eine attraktive Sache. Bei straßentauglichen Rennmaschinen dürfte die Versicherung teuer sein. Die private Nutzung beträgt 1% des Neuwertes oder Erstattung der Kilometer nach Fahrtenbuch. Damit ist das gestellte Motorad netto mit Sicherheit attraktiv. --Hans Haase (有问题吗) 23:17, 24. Jan. 2016 (CET)
Ein anderer Grund für eine gewerbliche Zulassung eines Motorrades wären Leasingmotorräder. Manche Vereine und Unternehmen bieten ihren Mitarbeitern vergünstigte Leasingbedingungen an, wenn die Mitarbeiter ihr Fahrzeug über ihren Arbeitgeber leasen. In meinem Wohnort erkennt man diese Fahrzeuge am ektopen Kennzeichen des Leasingunternehmens. --Rôtkæppchen₆₈ 23:45, 24. Jan. 2016 (CET)

Werden die Motorräder auch für Rennen eingesetzt? Dann wäre der Besitzer doch auch oft eine juristische Person. --Pölkkyposkisolisti 00:45, 25. Jan. 2016 (CET)

Die meisten wohl eher nicht, aber die Versicherung ist vom Arbeitgeber getragen billiger als die Netto vom Lohn/Gehalt zu zahlen. --Hans Haase (有问题吗) 12:28, 25. Jan. 2016 (CET)

2012 (Film) - Glocke

Hallo! Obwohl ich den Film mehrfach gesehen habe, verstehe ich nicht den Sinn dieser Einrichtung. Gibt es dafür ein reales Vorbild, also wo Mönche eine Glocke bewachen um das Ende der Welt einzuläuten?Oliver S.Y. (Diskussion) 19:46, 24. Jan. 2016 (CET)

Ich bin kein Fachmann, aber Glocken scheinen im Buddhismus eine relativ große Bedeutung zu haben - siehe hier lt. diesem PDF ist die Glocke ein Sinnbild der Vergänglichkeit--An-d (Diskussion) 21:32, 24. Jan. 2016 (CET)
Bonshō
Es ist ein "starkes Bild" und wurde deshalb lange vor dem Film als Trailer verwendet: Uiiih Mönch! Uiih Glocke, Uiiih! Wasser.... Cool! (Hier eine kurze Auflistung von tibetanischen Klischees und ihre Verwendung in der Pop-Kultur. Also "irgendwas mit Mystik"...)
Hat aber bei den Buddhisten nicht so gut angeschlagen. Aber tibetanische Mönche waren sowieso nicht die Zielgruppe für den Film. Play It Again, SPAM (Diskussion) 08:02, 25. Jan. 2016 (CET)

Postcard Frankfurt

 

Could anyone date and place this postcard?Smiley.toerist (Diskussion) 10:55, 24. Jan. 2016 (CET)

That's "Zeil" viewed from "Hauptwache". At the site of Hotel Milani you will now find a department store "Kaufhof". The date is definitely after 1891 because the main post office (today "MyZeil") is already there. Cf. [18]. Dumbox (Diskussion) 12:00, 24. Jan. 2016 (CET)
Andere Versionen der gleichen Motivs: koloriert, Schriftzug "Zeil" weiter links Gruss --Nightflyer (Diskussion) 12:16, 24. Jan. 2016 (CET)
Oh, and the streetcars are not horse-drawn, but I can't spot any overhead wiring either, which was introduced in 1899. There were in-between experiments with steam-powered streetcars and wired tracks, so maybe a streetcar expert (not me!) could help you with a more exact date. Dumbox (Diskussion) 14:16, 24. Jan. 2016 (CET)
as shown on Straßenbahn Frankfurt am Main, there was some suse of steam driven and accumulator driven trams beside the normal ones with overhead wiring. - 188.195.182.67 16:20, 24. Jan. 2016 (CET)
There is overhead wiring if you look closely at the buildings on the rigth. This would put it at 1899 or later.Smiley.toerist (Diskussion) 21:28, 24. Jan. 2016 (CET)
Yup, that's what User:Rotkaeppche68 pointed out below in German. Also consider my last posting. So it's 1899-1902. Regards Dumbox (Diskussion) 22:01, 24. Jan. 2016 (CET)
Es handelt sich offenbar um Straßenbahnen der Frankfurter Baureihe A, siehe commons:Category:Frankfurt tram type A. Unsere Fotos davon sind von 1900/1901. Lyrabügel und Fahrdraht sind zu filigran, um auf der unscharfen Photographie überall zu sehen zu sein. Ein Teil der Oberleitung im Vordergrund wurde wegretuschiert, am rechten Bildrand über dem Schriftzug Augus… sind die Überreste noch erkennbar. Gemäß Fahrzeuge der Straßenbahn Frankfurt am Main#A und B, E lässt sich das Foto auf den Zeitraum 1899 bis 1923 datieren. --Rôtkæppchen₆₈ 16:47, 24. Jan. 2016 (CET) Differentialdiagnostisch dieses Bild von 1898 mit anderen Straßenlampen und Pferdebahn]]. Dieses Bild von 1901 mit den gleichen Straßenbahnwagen und sichtbarem Fahrdraht und Lyrabügel schließt Akku-, Dampf-, Natron- oder Stromschienenbetrieb aus. --Rôtkæppchen₆₈ 17:10, 24. Jan. 2016 (CET)
(BK) Just found out the Milani was torn down in 1902, so here's your terminus ante quem. :) @Rotkaeppchen: Wenn man's gesagt bekommt, ist es so offensichtlich mit den Drähten. Respekt! :) Dumbox (Diskussion) 17:13, 24. Jan. 2016 (CET)
Bitte was ist wohl der [rote] Wagen Nr. 60 links neben der Uhr? Sieht aus wie ein Fass. Freibier für alle oder Apfelwein?. Oder spritzt man nur die Strassen gegen Staub? -- Ilja (Diskussion) 23:34, 24. Jan. 2016 (CET)
Ich tippe auf Sprengwagen, denn das Tanklöschfahrzeug wurde erst Mitte des 20. Jahrhunderts erfunden. --Rôtkæppchen₆₈ 23:52, 24. Jan. 2016 (CET)
(BK)Keine Ahnung. Aber ein ähnliches Motiv auf einer Postkarte gibt es hier. Diesmal mit Wagennummer 18. Wenn die Wagen durchnummeriert sind und immer an der gleichen Stelle stehen, hatten sie vermutlich eine öffentliche Aufgabe. Von Schienen für eine Bahn ist noch nichts zu sehen. Trinkwasserversorgung? Gruss --Nightflyer (Diskussion) 00:00, 25. Jan. 2016 (CET)
Zusatzfrage: Ist die Zahl "60" wirklich real oder eine frühe Bildbearbeitung? Gruss --Nightflyer (Diskussion) 00:13, 25. Jan. 2016 (CET)
Stimmt, die Zahl ist zu hell. --Pölkkyposkisolisti 00:37, 25. Jan. 2016 (CET)
(BK)Du siehst die alte Straßenbeleuchtung und andere Beschriftung am Milani. Außerdem ist der Wagen Nr. 18 bespannt und es sitzt ein Fahrer drauf. Das heißt, er fährt demnächst weiter. Auf File:Frankfurt Am Main-Zeil mit Katharinenkirche-1901.jpg sieht man, dass die neuen Straßenlaternen gleichzeitig die Spanndrähte der der Oberleitung hielten. --Rôtkæppchen₆₈ 00:42, 25. Jan. 2016 (CET)
@Nightflyer: Auf Datei:Frankfurt Am Main-Max Junghaendel-FFMANDN-15-Zeil.jpg von 1898 siehst Du dieselbe Beschriftung am Milani, aber deutlich die Pferdebahnschienen. Demnach müssten die Schienen auch 1900 noch da gewesen sein. Meine Vermutung ist, dass die Schienen anlässlich der Umstellung auf elektrische Traktion nicht ausgetauscht wurden, aber eben die alten Straßenlampen gegen neue Masten mit Fahrdrahtspannmöglichkeit getauscht wurden. --Rôtkæppchen₆₈ 00:48, 25. Jan. 2016 (CET)
Ich habe noch einen weiteren Aspekt: Die Straßenbäume. Auf dem 1900er-Foto sind die Bäume kleiner als auf Benutzer:Smiley.toerists Foto. Demnach müsste das fragliche Foto nach 1900 aufgenommen sein. Kann jemand die Höhe der Bäume auf dem 1901er-Foto beurteilen? --Rôtkæppchen₆₈ 00:59, 25. Jan. 2016 (CET)
Auf dem 1901er-Foto steht vor der Uhr ein Dreirädriges Gestell mit einem ca. 6-7 Meter hohen Mast. Hat jemand eine Vermutung was das sein könnte? --Mauerquadrant (Diskussion) 01:54, 25. Jan. 2016 (CET)
Dem Schattenwurf nach würde ich auf eine Leiter nach Art der Feuerwehrdrehleiter auf Rädern tippen. --Rôtkæppchen₆₈ 03:10, 25. Jan. 2016 (CET)
Wenn man das Bild vergrößert sieht man deutlich, dass es sich um eine fahrbare, aber relativ kurze Schiebeleiter handelt, wie sie Handwerker und Versorgungsunternehmen hatten. Wartungsarbeiten an der Uhr oder deren Beleuchtung?--2003:75:AF0E:D00:BCA1:E8ED:F963:F763 08:30, 25. Jan. 2016 (CET)
Danke, jetzt erkenne ich es auch. --Mauerquadrant (Diskussion) 11:48, 25. Jan. 2016 (CET)
When was Café Milani also an hotel? The 1885 image doesnt show a hotel sign, but maybe already had some rooms upstairs.Smiley.toerist (Diskussion) 10:48, 25. Jan. 2016 (CET)
The earliest in books.google seems to be „Hotel Milani (1215), Zeil 72 (10 Zimmer).“ Leo Woerl: Kleiner Rheinführer 1900. --Vsop (Diskussion) 11:35, 25. Jan. 2016 (CET)

Rechtsform "städtische Gesellschaft" - was ist das?

auf der Website braunschweig.de habe ich gerade gelesen: << Die VHS Braunschweig ist von einem Verein zu einer städtischen Gesellschaft umgegründet worden. Das ist mehr als eine Änderung der Rechtsform... >>

Aber, was für eine Rechtsform ist das, "städtische Gesellschaft" ? Ich konnte nichts dazu finden, weder in der Wikipedia-Kategorie Gesellschaftsrecht_(Deutschland) noch anhand Google. --Daevl (Diskussion) 22:19, 23. Jan. 2016 (CET)

Wie ich das verstehe, ist die VHS jetzt eine Eigengesellschaft der Stadt Braunschweig, also privatwirtschaftlich organisiert (als GmbH, nehme ich an), mit der Stadt als 100-prozentigem Gesellschafter. 77.176.16.145 22:31, 23. Jan. 2016 (CET)
Google zufolge ist die VHS Braunschweig eine GmbH. Das ist rechtlich etwas anderes als ein Eigenbetrieb, auch wenn sie in kommunalem Besitz ist. --Rôtkæppchen₆₈ 08:09, 26. Jan. 2016 (CET)

Stab mit Papierstreifen und † oben im christlichen Kontext (kath.)

 

Wie heißt eigentlich der Stab mit dem beschrifteten Papierstreifen und dem † oben rechts im Bild? Danke --Mattes (Diskussion) 00:38, 22. Jan. 2016 (CET)

Wenn ich nach Kreuzstab mit Fahne googele steht beim 2. Treffer: Christus den Kreuzstab mit der Auferstehungsfahne in der Google Kurzbeschreibung. --Mauerquadrant (Diskussion) 01:04, 22. Jan. 2016 (CET)
Agnus Dei-Symbolik. Dort ist die Rede von Siegesfahne. --Pp.paul.4 (Diskussion) 01:16, 22. Jan. 2016 (CET)
+1 Wenn man reinzoomt, kann man von unten ECC lesen (Ecce Agnus Dei). Play It Again, SPAM (Diskussion) 08:43, 22. Jan. 2016 (CET)
Gut gesehen, auf der untersten Windung steht von links nach rechts EC(CE) in vorderseitiger Schrift, auf der zweituntersten Windung steht von rechts nach links AGNV(S) in rückseitiger Schrift usw. --Pp.paul.4 (Diskussion) 09:01, 22. Jan. 2016 (CET)
Und vor dem Stöckchen "D" - und dahinter "EI".
So, jetzt fehlt noch Auferstehungsfahne. Bin ich der einzige, der dieses Wort auch irgendwie komisch findet? Play It Again, SPAM (Diskussion) 09:20, 22. Jan. 2016 (CET)
Wieso, das ist das, was du hast, wenn du nach einer langen Nacht verkatert aufwachst. (Tötet Trichinen auch hinter Tresortüren) --RobTorgel 09:34, 22. Jan. 2016 (CET)
Kreuzstab fehlt auch noch. --Mauerquadrant (Diskussion) 10:00, 22. Jan. 2016 (CET)
Das ist die Ferula mit Siegesfahne. die nicht aus Papier ist, sondern aus Brokat sein dürfte. Auferstehungsfahne ist 33 Jahre VOR der Auferstehung sinnlos. Außerdem hätte die einen Querstab und wäre ein rotes Kreuz auf weißem Grund oder umgekehrt. --Heletz (Diskussion) 10:10, 22. Jan. 2016 (CET)
Die Sache mit „vor“ ist ikonographisch m. E. kein Gradmesser: daß der kindliche hl. Johannes der Täufer auf solchen Darstellungen oft ein Schaffell trägt oder auf dem Dach der Krippe ein Pfau sitzt, greift dem Zeitablauf auch vor, das heißt, ikonographisch verweisen solche Attribute auf die Zukunft.--Turris Davidica (Diskussion) 10:21, 22. Jan. 2016 (CET)
typologisch betrachtet verweist eigentlich so recht alles auf die Zukunft. --Edith Wahr (Diskussion) 13:04, 22. Jan. 2016 (CET)
Ich hab mal eine Weiterleitung von Kreuzstab nach Ferula (Kreuzstab) angelegt. --Mauerquadrant (Diskussion) 10:39, 22. Jan. 2016 (CET)
Benutzer Pp.paul.4 hat schon in der zweiten Antwort auf unseren Artikel Agnus Dei hingewiesen. Dort heißt es: Insbesondere in den johanneischen Schriften des Neuen Testaments spielt die Lamm-Gottes-Symbolik eine besondere Rolle. An zwei Stellen des Johannes-Evangeliums weist Johannes der Täufer auf Jesus Christus mit den Worten hin: „Seht, das Lamm Gottes, das die Sünde der Welt hinwegnimmt.“ (Joh 1,29 ELB, Joh 1,36 ELB). Die Deutung des "Johannes-Fähnchens" als "Auferstehungsfahne" (die zeigt gewöhnlich nur ein Kreuz) oder "Siegesfahne" halte ich für zu weit vorgegriffen; die Bedeutung ist m.E.: Schon der kleine Johannes kennt den kleinen Jesus als "Lamm Gottes" und als Gottessohn, nicht erst der erwachsene. Oder: Schon in den Kleinen steckt drin, was sie als Erwachsene tun werden. Den ganz jugendlichen Johannes als Propheten kennt auch das Lukasevangelium, siehe Mariä Heimsuchung, in diesem Artikel fehlt allerdings leider ein Hinweis auf den "hüpfenden Johannes" (Lk 1,41 ELB), genau wie in Johannes der Täufer. --Bremond (Diskussion) 15:37, 22. Jan. 2016 (CET)

Zusatzfrage

Das Madonnenbild ist aus dem 16. Jahrh.

Lässt sich die These erhärten, dass sich ein Jesuskind in frontaler Nacktheit fast ausschliesslich in Gemälden bis - sagen wir mal 1650 - findet? Dass später dann immer ein Tuch die Männlichkeit verdeckt oder das Kind im (Halb)Profil gezeigt wird?
Gibt es dazu kunsthistorische Untersuchungen? Play It Again, SPAM (Diskussion) 10:51, 22. Jan. 2016 (CET)
Wenn überhaupt, dann müßte man das nach Ländern getrennt betrachten (Italien, Deutschland, Frankreich ...). Mal gucken, ob ich was finde. --Heletz (Diskussion) 11:54, 22. Jan. 2016 (CET)
Interessante Beobachtung. Literatur habe ich jetzt keine, aber eine schnelle Durchsicht ergibt in der Tat: Den letzten sichtbaren Schniedel eines Jesuskindes finde ich bei Murillo um 1650-1660. Danach: Tücher, Schenkel, Kleider. --Jossi (Diskussion) 12:16, 22. Jan. 2016 (CET)
Womöglich eine allgemeine Zeit- und Sittenfrage, vgl. etwa die Heiligen und Propheten der sixtinischen Kapelle und die Tätigkeit Daniele da Volterras als „Hosenmaler“.--Turris Davidica (Diskussion) 12:31, 22. Jan. 2016 (CET)
 
Der vitruvianische (Renaissance)Mensch, Proportionsstudie nach Vitruv von Leonardo da Vinci (1492)
Siehe rechts, zeitlich etwas weiter vorn, 1641
 
man beachte die geschickte Plazierung des kleinen Skapuliers. ;)
.--Turris Davidica (Diskussion) 13:04, 22. Jan. 2016 (CET)
Bin drauf gekommen, weil in den meisten dieser Gemälde, jede Farbe, jede Handhaltung, jedes Utensil eine symbolische Bedeutung hatte.
Und auf genau das wird auch (obige Tatsache) hier referenziert. Wieder ein weisser Fleck weniger. Play It Again, SPAM (Diskussion) 13:13, 22. Jan. 2016 (CET)
...but to demonstrate Jesus' full humanity even as the Son of God. Interessant, um zu zeigen, dass er ein Mensch ist, bräuchte es, seinen Penis zu zeigen? Gibt es irgendwo eine Tradition, dass Gott keinen Penis hat? Wobei, (christlicher) Gott (ich meine jetzt als Gott-Vater) wird ja eher nie nackt abgebildet (beim Heiligen Geist bin ich mir nicht so sicher). --Alexmagnus Fragen? 19:23, 22. Jan. 2016 (CET)
gibt es denn eine Tradition, die besagt, dass Gott einen Penis habe? In der Bibel steht ja nix dergleichen, denn der Herrgott zeigt sich den Sterblichen ja so gut wie nie, und wenn, dann nur von hinten (2. Mose 33), und nicht mal dann erfährt man sachdienliche Hinweise, die zur Ergreifung des Täters führen könnten, sondern bspw. nur wie die "die Fläche unter seinen Füßen" aussah (nämlich mit Saphir ausgelegt, 2. Mose 24): In [biblical] accounts of God sightings, the gaze is averted from the face and front, parts of the anatomy that are critical to an identification of a body's sex. Not only is there no indication that this God has a penis, but we don't even know whether this being has secondary sexual characteristics such as facial hair […] People do think of God as a he without a male body. [19] --Edith Wahr (Diskussion) 18:00, 23. Jan. 2016 (CET)
Tradition ist mir nicht bekannt, aber um die Jahrtausendwende hat sich George Carlin mit der biologischen Austattung von Göttern allgemein auseinandergesetzt (steht in seinen "quotes"). Play It Again, SPAM (Diskussion) 17:30, 26. Jan. 2016 (CET)

„…but to demonstrate Jesus' full humanity even as the Son of God“ in diesem Satz gehts auch darum, daß der Sohn Gottes, wahrer Mensch und wahrer Gott, eben die Gestalt eines Sohnes angenommen hat.--Turris Davidica (Diskussion) 18:27, 23. Jan. 2016 (CET)

Wohin mit dem Testament?

Im letzten Jahr verstarb meine Frau. Wir haben keine Kinder. Als nächstes erbberechtigt sind ferne Verwandte von mir, die ich nicht unbedingt beerben will. Mein kleines Gespartes soll an wohltätige Organisationen gehen. Dies habe ich handschriftlich in meinem Testament festgehalten. Wie stelle ich sicher, dass dieser letzte Wille beachtet und gefunden wird, wennn sonst keiner da ist? --Guenther1950 (Diskussion) 20:00, 24. Jan. 2016 (CET)

einfach beim Nachlassgericht hinterlegen. Kostet pauschal 75 €. 188.195.182.67 20:02, 24. Jan. 2016 (CET)
Wie erfährt das Nachlassgericht von meinem Ableben? --Guenther1950 (Diskussion) 20:06, 24. Jan. 2016 (CET)
Durch den Standesbeamten, der die nötigen Formalitäten erledigt, z.B. die Sterbeurkunde ausstellt. --OnlyMe (Diskussion) 21:16, 24. Jan. 2016 (CET)
geht aus dem verlinkten Artikel hervor: Wer einen Erbschein haben will, muss den da beantragen. Ohne diesen kann er nicht rechtmäßig über den Nachlass verfügen. - 188.195.182.67 20:08, 24. Jan. 2016 (CET)
Das ist doch genau das Problem. Es wird wohl keiner den Erbschein beantragen. --Guenther1950 (Diskussion) 20:13, 24. Jan. 2016 (CET)
schon Erbschein gelesen ? - 188.195.182.67 20:14, 24. Jan. 2016 (CET)
Das habe ich. Es bringt mich hinsichtlich meines Problems nicht weiter. --Guenther1950 (Diskussion) 20:44, 24. Jan. 2016 (CET)
1. pflichtteil... so ganz kann man das wohl nich verhindern... 2. vllt kann man der „wohltätigen Organisation“ von dem Testament erzählen? --Heimschützenzentrum (?) 20:47, 24. Jan. 2016 (CET)
Pflichtteil dürfte bei "Fernen Verwandten" kaum eine Rolle spielen. -- Gerd (Diskussion) 20:57, 24. Jan. 2016 (CET)
wenn's keine näheren als die fernen gibt, dann sind die fernen doch die Nächsten? *heul* mein Babelfisch hat was an der Schwimmblase... --Heimschützenzentrum (?) 21:04, 24. Jan. 2016 (CET)
Bedeutet aber nicht, dass diese auch Pflichtteilsberechtigte sind, siehe § 2303 BGB. Der P. steht nur "Abkömmlingen" d.h. leibliche oder adoptierte Kinder, evtl. die Erben jener (wenn sie vorverstorben sind), dazu noch Ehegatten und Eltern. "Ferne" Verwandte gehen leer aus. --OnlyMe (Diskussion) 21:13, 24. Jan. 2016 (CET)
zu 1. Es gibt keine Verwandten mehr, die einen Pflichtteil bekommen.
zu 2. Da beißt sich die Katze in den Schwanz. Wie erfahren die wohltätigen Organisationen von meinem Ableben?
--Guenther1950 (Diskussion) 21:21, 24. Jan. 2016 (CET)
die wohltätigen Organisationen müssen ja nicht sofort davon erfahren... es reicht, dass da nur die Möglichkeit besteht, dass da irgendwann ein Kläger und dann auch ein Richter auftauchen könnten... diese Gefahr wird Erbschleicher abschrecken... --Heimschützenzentrum (?) 22:09, 24. Jan. 2016 (CET)
Die von dir im Testament bedachten Erben erfahren vom Erbfall (also von deinem Ableben) durch den Testamentsvollstrecker, den du auch gleich selbst bestimmen kannst.
Zur Formulierung: Du beerbst deine entfernten Verwandten, wenn die versterben und du der Erbe bist. Wenn dagegen du stirbst und deine entfernten Verwandten erben, dann wirst du von ihnen beerbt. --Snevern 22:17, 24. Jan. 2016 (CET)
und wie erfährt der Testamentsvollstrecker von dem Erbfall? --Heimschützenzentrum (?) 22:55, 24. Jan. 2016 (CET)
Durch die Testamentseröffnung. Sobald das Gericht vom Tod des Erblassers Kenntnis erlangt hat, hat es eine in seiner Verwahrung befindliche Verfügung von Todes wegen zu eröffnen. Das Gericht kann zur Eröffnung der Verfügung von Todes wegen einen Termin bestimmen und die gesetzlichen Erben sowie die sonstigen Beteiligten zum Termin laden. Das steht in § 348 FamFG. Zu den "sonstigen Beteiligten" gehört der benannte Testamentsvollstrecker. --Snevern 23:09, 24. Jan. 2016 (CET)
und wie erfährt das Gericht vom Tod des Erblassers? *kicher* --Heimschützenzentrum (?) 08:20, 26. Jan. 2016 (CET)
Die Sterbefallanzeige geht ans örtlich zuständige Standesamt, das Standesamt informiert das Nachlassgericht (bingo!) und die Bundesnotarkammer. Diese schaut ins Zentrale Testamentsregister und prüft, ob da was über ein Testament steht; falls ja, teilt sie das dem Nachlassgericht mit (nochmal bingo!). --Snevern 10:41, 26. Jan. 2016 (CET)
das hört sich ja mal richtig durchdacht an... soopah... :) --Heimschützenzentrum (?) 08:43, 27. Jan. 2016 (CET)
Na, solange der Tote nicht völlig spurlos verschwindet, wird ja jemand da sein, der ihn findet. Für den Fall kann der Erblasser einen Zettel à la "Bitte informieren Sie im Todesfälle die und die Person" bei sich tragen. Bei älteren Menschen ist das Ende nicht selten einige Zeit vorher absehbar; man befindet sich im Krankenhaus oder im Altenheim - da lassen sich Krankenschwestern, Betreuer, etc. vorher informieren, wen sie im Falle des Falles dann benachrichtigen sollen. --Proofreader (Diskussion) 23:14, 24. Jan. 2016 (CET)
Da wäre ich mir nicht so sicher. Gelegentlich liest man dann ja doch von Funden mumifizierter Leichen, wo die Nachbarn den Wohnungsinhaber im Urlaub oder Seniorenheim vermuteten. Ich habe da einen aktuellen Fall, wo der hochbetagte, kranke, schwerbehinderte, alleinstehende Herr fast 600 Kilometer von seinen Anverwandten wegwohnt. Ein Testament und eine Patientenverfügung existieren aber, ich weiß nur nicht wo. --Rôtkæppchen₆₈ 00:19, 25. Jan. 2016 (CET)
Ich glaube, wir können die Frage nicht beantworten, wie der Fragesteller Guenther1950 das organisieren kann. Guenther, ich fürchte, du musst dir selbst eine Person deines Vertrauens suchen, die regelmäßig guckt, ob du noch lebst, und im Falle deines Todes das Nachlassgericht verständigt! --MannMaus 00:06, 26. Jan. 2016 (CET)
Notar fragen. Der kennt sich bei der Verwaltung von Testamenten bestens aus und sorgt dafür, dass nichts wegkommt.--Giftzwerg 88 (Diskussion) 20:40, 26. Jan. 2016 (CET)

NPD im Landtag

Frage an Leute aus den Ländern wo sie drin ist oder war: Was haben die Typen da gemacht? Nur ihre dicken Gehälter eingeschoben oder haben die da Aktivität gezeigt bzw. lediglich getrollt? Wie sind die Politiker der anderen Parteien damit umgegangen?--Antemister (Diskussion) 22:55, 20. Jan. 2016 (CET)

Betrifft MeckPomm: Ich meine mich an einen Presseartikel (wohl Ostseezeitung oder NNN) zu erinnern, wonach die Landtagsabgeordneten der NPD mit großem Abstand die meisten Ermahnungen wegen Zwischenrufen und anderem störenden Verhalten bekommen haben. Außerdem wurde Udo Pastörs' Immunität mehrfach aufgehoben. Also, mein POV-Fazit: nur getrollt. Grüße, Grand-Duc (Diskussion) 23:06, 20. Jan. 2016 (CET)
NPD_Mecklenburg-Vorpommern#Landtagsfraktion --Buchling (Diskussion) 23:14, 20. Jan. 2016 (CET)
NNN? --85.212.24.128 00:35, 21. Jan. 2016 (CET)
In Sachsen haben sie teilweise Gelder veruntreut um Kameradschafts-Arbeit zu unterstützen. Fängt beim Kopierernutzen an, wo es aufhört wissen sie vermutlich nur selbst. Habe leider keine Quelle aus meine Zeitungsgedächtnis. -- 141.30.146.64 03:06, 21. Jan. 2016 (CET)

Die NPD kann so viele Anträge stellen wie sie will, die werden nur von einer der anderen Parteien beantwortet und immer abgelehnt. Dieses Schweriner Modell funktioniert. --Pölkkyposkisolisti 08:37, 21. Jan. 2016 (CET)

Der Status einer Partei wird ja für die NPD wohl gerade wieder einmal geprüft. Neben den bekannten Problemstellungen wird sicherlich auch geprüft, ob die überhaupt eine Partei nach dem Parteiengesetz darstellen oder eher nur eine Interessengemeinschaft: "..die politische Bildung anregen und vertiefen, die aktive Teilnahme der Bürger am politischen Leben fördern, zur Übernahme öffentlicher Verantwortung befähigte Bürger heranbilden, sich durch Aufstellung von Bewerbern an den Wahlen in Bund, Ländern und Gemeinden beteiligen, auf die politische Entwicklung in Parlament und Regierung Einfluß nehmen, die von ihnen erarbeiteten politischen Ziele in den Prozeß der staatlichen Willensbildung einführen und für eine ständige lebendige Verbindung zwischen dem Volk und den Staatsorganen sorgen..."--Wikiseidank (Diskussion) 10:18, 21. Jan. 2016 (CET)
Ich bezweifle, dass das "sicherlich geprüft" wird. --Eike (Diskussion) 10:50, 21. Jan. 2016 (CET)
Die Nationaldemokratische Partei Deutschlands wurde vor Inkrafttreten des Parteiengesetz (Deutschland) gegründet und ihre Eigenschaft als politische Partei wurde nie wirklich angezweifelt. Ich bezweifle deshalb, dass sich jemand die Mühe macht, gesondert zu prüfen, ob die offensichtlich vorliegenden Parteieigenschaften den möglicherweise nicht anwendbaren Bestimmungen des Parteiengesetzes genügen. --Rôtkæppchen₆₈ 11:34, 21. Jan. 2016 (CET)
Fuer Sachsen (leider ohne Quelle, ich hab mir die Finger wundgegoogelt, und steh jetz bestimmt unter Verfassungsschutz-Beobachtung): Die haben sich auch ordentlich selbst zertrollt. Nach dem Einzug in den Landtag hat wohl eine kleine Gruppe um Klaus Baier und Mirko Schmidt versucht, "konstruktive" Politik zu machen. Dafuer haben sie wohl vor allem Antraege der SPD gekapert und ausgearbeitet, an denen dann ploetzlich die SPD nicht mehr interessiert war. Auch die anderen Parteien wollten nicht mit der NPD reden, auch die Presse nicht (deswegen siehts auch so schlecht aus mit der Quellenlage). Der Rest der Partei ist mit den bekannten Flegeleien aufgefallen, wodurch mit denen erst recht keiner mehr reden wollte. Der Rest der Partei fand auch die "konstruktive" Arbeit nicht so toll, wodurch die wohl auch parteiintern auf die Muetze gekriegt und ihre "Konstruktivitaet" ziemlich bald eingestellt haben. --Nurmalschnell (Diskussion) 10:41, 21. Jan. 2016 (CET)
Als Quellen für Sachsen bieten sich vor allem die Plenarprotokolle der Landtagssitzungen an. Ich musste jetzt zwar auch etwas suchen, habe sie dann aber unter: [[20]] gefunden. Rechts oben kann man die gewünschte Legislaturperiode auswählen (die NPD war in der 4. und 5. Wahlperoide im Landtag) und dann links unter Plenarprotokolle/Sprecher die üblichen Verdächtigen raussuchen. Ich habe damals ab und zu die Protokolle gelesen - nicht wegen der NPD, sondern rein aus Interesse, was da so im Landtag besprochen wird. Wenn ich mich richtig erinnere, kann ich die Aussagen von Nurmalschnell nur bestätigen. Vor allem zu Beginn hatte sich die NPD gern mal Themen aus den Wahlprogrammen der anderen Parteien gesucht, die zu Ihnen passen und noch nicht eingebracht wurden. Das war besonders ärgerlich bei Themen, welche die Koalitionsfraktionen auch auf der Agenda hatten, aber noch nicht weit genug ausgearbeitet waren, denn damit war das Thema erstmal eine Zeitlang "verbrannt". Ansonsten kam von den NPD-Abgeordneten nicht viel - ein bisschen "trolliges" Verhalten mit vielen Zwischenrufen vielleicht noch, aber mehr war da nicht. Später war die NPD-Fraktion dann mehr damit beschäftigt sich selbst zu zerlegen...--84.173.214.15 01:04, 22. Jan. 2016 (CET)
Im Tragen von Szenekleidung war die sächsische NPD-Fraktion auch ganz stark, erheiterndes Video gibts auf YouTube: [21]. Yellowcard (D.) 01:19, 22. Jan. 2016 (CET)
Was ist denn daran erheiternd? Das ist vollkommen undemokratisch und reinster Machtmissbrauch. Ich mag die NPD auch nicht, aber das so etwas stattfinden kann ist 10 mal so gefährlich wie diese handvoll NPDler. --84.149.232.44 22:32, 23. Jan. 2016 (CET)
Bei der NSDAP waren die anderen Fraktionen nicht erst wegen Fehlverhalten, aber von vorne herein ausgeschlossen gewesen. Hitler hatte damals seine eigene Polizei geschaffen, die dann untätig geblieben wäre.
Ein anderer Politiker sagt es passend, treffend und nicht unrichtig und er fliegt nirgendwo raus.[22] Nach eigenen Angaben traf er sich mit jemand von der NPD im Gericht. Seine Anspielungen auf Mediokratien sind selbstverständlich unübersehbar in die Veranstaltung eingeplant worden. --Hans Haase (有问题吗) 01:01, 24. Jan. 2016 (CET)
Was ist daran denn Machtmissbrauch und undemokratisch? Sie wurden nach Fehlverhalten rausgeworfen. Sie wurden sogar höflich darum gebeten die unpassende Kleidung abzulegen. Sie haben sich jedoch sturr dagegegn geweigert. Sie wären nicht rausgeflogen, wenn sie sich einfach an die Bedingungen gehalten hätten anstatt aufzumucken. --HamburgerThure (Diskussion) 14:58, 25. Jan. 2016 (CET)
Vor allem haben sie ihr vorzügliches Deutsch zur Schau gestellt und Küche und Damentoilette gefunden - ganz alleine.--IP-Los (Diskussion) 19:55, 27. Jan. 2016 (CET)

Standesamt

Hallo! Wenn ich das oben mit dem Testament lese, die eigentlich simplen Fragen, a) werden Kinder einer Person bei deren Geburtsstandesamt irgendwie vermerkt? und b) Holt das Nachlassgericht regelmäßig eine Auskunft beim Geburtsstandesamt ein. (Standardfall in Deutschland). Oliver S.Y. (Diskussion) 21:36, 24. Jan. 2016 (CET)

Geburts- und Sterbefälle werden dem lokal zuständigen (örtlichen) Standesamt gemeldet. In der Geburtsurkunde sind die Eltern, soweit sie dies anerkennen, vermerkt. Bei der Entbindenden ist der Beweis erbracht, außer die wäre Leihmutter, was in Deutschland nicht zulässig ist. Anders sähe es aus, wenn die Leihmutterschaft im Ausland initiiert wurde. (Rechtliche Konsequenzen?) Der Vater kann das Kind anerkennen. Bei unverheirateten Eltern ist das die rechtliche Frage des Unterhalts. In der Geburtsurkunde werden die Eltern eindeutig – mit ihrem Geburtsort und -name – vermerkt. --Hans Haase (有问题吗) 21:52, 24. Jan. 2016 (CET)
Sind wir wieder mal nicht fähig, eine Frage zu lesen Hans? Dann für Dich extra langsam. Der Vater eines Kindes ist in Wismar geboren, und entsprechend dort im Standesamt eingetragen. Sein Kind wurde in Lübeck geboren, und dort ins Standesamt eingetragen. Meldet nun das Amt Lübeck ans Amt Wismar die Geburt, daß sie dort vermerkt wird? Wie gesagt, Standardfall Deutschland, keine Hintergedanken.Oliver S.Y. (Diskussion) 22:01, 24. Jan. 2016 (CET)
Der Sterbeurkunde durfte ich damals hinterhertelefonieren. Sterbeort war außerhalb. --Hans Haase (有问题吗) 23:17, 24. Jan. 2016 (CET)
Ich hoffe mal nicht. Da wir ja noch nicht völlig durchnummeriert sind, wäre ja auch eine Namensdopplung (gleicher Vor- und Zuname, gleicher Geburtstag in der gleichen Stadt) möglich. --Hachinger62 (Diskussion) 22:20, 24. Jan. 2016 (CET)
Geht um die "Handakte" zur Person. Ich weiß zB. das es für mich eine gibt, weil es zwei nachträgliche Änderungen gab. Kenne nur deren Inhalt nicht, und weiß auch angesichts der Digitalisierung nicht, wieviele Angaben da im Hintergrund hin und hergeschickt werden.Oliver S.Y. (Diskussion) 22:25, 24. Jan. 2016 (CET)
Sind wir ja doch mittlerweile, mit der Steuernummer. --Chricho ¹ ² ³ 14:24, 27. Jan. 2016 (CET)

Zentrales Testamentsregister. --Bubo 22:39, 24. Jan. 2016 (CET)

Aber da werden doch keine Verwandtschaftsbeziehungen gespeichert? --Chricho ¹ ² ³ 14:23, 27. Jan. 2016 (CET)
Das Zentrale Testamentsregister benachrichtigt das Nachlassgericht und das Nachlassgericht ist von Amts wegen verpflichtet, die Erben und Pflichtteilsberechtigten zu ermitteln und zu benachrichtigen. --Rôtkæppchen₆₈ 16:42, 28. Jan. 2016 (CET)

Das ist eine indirekte Frage (nur mein Beitrag wurde auf Wiki gelöscht!)

Ich sah eine Möglichkeit eine Maschine zu bauen in etwa wie Perpetuum Mobile wenn man die Natur als freundlich ansieht. Diese die in etwa aussieht wie das Cream Amnesia Logo http://www.22manchester.co.uk/news/the-cream-of-superlogos/ dreht sich wie ein Raupenantrieb und induziert eine Spannung in eine Spule dabei verdaut es organische oder chemische Substanzen und es wird thermische energie frei - zudem gelingt die Filterung...wenn man mit solch einer Maschine umzieht und sie dann weiter machen lässt ist sie quasi ewig..

--Marian Müller (Marc Miller / S*Wings Records Berlin) (nicht signierter Beitrag von 2A02:8109:9180:834:49BF:EE0D:9C3:D2C3 (Diskussion | Beiträge) 23:21, 23. Jan. 2016 (CET))

Das war ich. Dein Beitrag entsprach nicht WP:D, unserem Standard für Diskussionsseiten. Bitte tu dir selbst einen Gefallen und lies erst einmal durch, was ein Perpetuum Mobile ist, bevor du noch mehr Ideen spinnst. --Geist, der stets verneint (quatschen?|Fauler Sack?) 23:23, 23. Jan. 2016 (CET)
Das haste aus Nimmerklug in Sonnenstadt geklaut, nicht wahr? Da war's allerdings ein Erntemaschine, ziemlich cool das Teil. --87.148.91.233 23:45, 23. Jan. 2016 (CET)
Ich denke, der Energieerhaltungssatz wird manchmal nicht sofort richtig in seinen am jeweilige Beispiel existierenden Anteilen erkannt wird. YouTube ist voll davon. Und es ist stets das Arme-Leute-Märchen für die, denen Energiepreise einen höhen Anteil vom Einkommen wegnimmt. Not macht erfinderisch. Die Versuche sind gewagt, aber da die Not größer ist als das Wissen, wird auch das Scheitern später erkannt. --Hans Haase (有问题吗) 23:54, 23. Jan. 2016 (CET) Und nun los auf die Schulbank und Lehrgeld zahlen. Und nicht vergessen, den Müll zu sortieren, damit der Staat in gewinnbringend in der Müllverbrennung für sich verwerten kann.
Kann man die Schule eigentlich verklagen, wenn sie einem den Energieerhaltungssatz nicht richtig beibringt? Von wegen "Lehrgeld zahlen" ;) Wenn eine Maschine organische oder chemische Substanzen verdaut, um Energie zu gewinnen, dann ist sie kein Perpetuum mobile, sondern sie hat einen Stoffwechsel, der mir irgendwie bekannt vorkommt. Die "ewige" Energiegewinnung durch einzelne Exemplare scheitert aber in meinem Beispiel durch gewisse Abnützungserscheinungen. Trotz Wartung gibt es irgendwann Materialermüdung. --Regiomontanus (Diskussion) 00:25, 24. Jan. 2016 (CET)
Der hat doch deutlich geschrieben "in etwa wie Perpetuum Mobile" und sogar den Zusatzantrieb umrissen. Warum wird dann wie mit einem Perpetuum Mobile auf ihn geschossen?--2003:75:AF37:7900:C48A:26BB:F164:4E0 00:32, 24. Jan. 2016 (CET)
Ich lese im gelöschten Beitrag „als Perpetuum Mobile zu bauen“. Die vorgestellte Maschine hat aber mit einem perpetuum mobile nichts zu tun; noch weniger hat der Beitrag, wie der löschende Geist, der stets verneint korrekt beschieden hat, mit der Verbesserung des Artikels perpetuum mobile, wozu gemäß wp:DS nunmal die Seite Diskussion:perpetuum mobile da ist, zu tun. --Rôtkæppchen₆₈ 00:38, 24. Jan. 2016 (CET)
Lehrgeld? Natürlich Lehrgeld! Staatliche Schule taugt nichts (mehr). Es ist ideologische Zeitverschwendung mit Anwesendheitspflicht. Das könnte man auch „Vollzugsanstalt“ nennen. Zeit ist Geld. Wer sich eine Privatschule leisten kann, kann sich freikaufen und seine Zeit sinnvoll nutzen. Für die, die noch nicht präkarisiert sind, gibt es die Nachhilfe. Nachhilfe macht das, für was die Lehrer nicht bezahlt werden. Der Rest darf integriert stören und dazu benötigt die staatliche Schule die Kinder des Präkariats. Sie stören, weil sie nur so Aufmerksamkeit erhalten. „Milieugeschädigt“ ist auch ein Begriff dafür. Dadurch kommt die Klasse nicht vorwärts und des besteht der Zwang zur Nachhilfe. Nachhilfe boomt. Wenn wir Fachkräfte brauchen – aus Syrien kommen sie. Zuvor hatten wir das weltweit beste Duale Bildungssystem, das angeblich weltweit übernommen würde. Das Duale System gibt es auch bei der Mülltrennung: Schnellbrennbares von langsambrennbarem sortieren, damit in der Müllverbrennung die Temperatur durch Wahl des Behälters, aus dem nachgeschüttet wird, gehalten werden kann. Reicht es für die Nachhilfe nicht, für die Mülltrennung reicht es allemal. Wer bessere Noten hat, kann es mit Gedichtsinterpretationen versuchen.[23] --Hans Haase (有问题吗) 02:20, 24. Jan. 2016 (CET)
Mal wieder frei flottierende Assoziationen eines dafür bekannten Benutzers. Der erfahrungsgemäß Kritik an seinen Auslassungen gerne mal weglöscht. Aber irgendwie extraviel Good Faith für sein Verhalten reklamiert. Und erhält. Mannmannmann. 84.153.89.115 05:39, 24. Jan. 2016 (CET)
Schule ist in Deutschland kostenlos, weil die Kinder dazu herandressiert werden, andere Leute reich zu machen, aber nicht sich selbst. Mit der Berufsausbildung ist es dasselbe. Man beschäftigte sich stattdessen damit, dass Maler und Stukkateure, die bisher ihre Gerüste selbst aufbauten, den neu erfundenen Beruf des Gerüstbauers erlernen mussten. Dasselbe Märchen durften KFZ-Mechaniker anhören, die nun Mechatroniker werden mussten aber von Elektronik keine Ahnung hatten. Den Heizungsbauern, die an der Steuerung modernden Anlagen verzweifelten, genehmigte man dafür das Verlegen von Abwasserleitungen. Arbeitgeber und Politiker lamentierten über den Frachkräftemangel, den sie bis dahin selbst verursacht hatten. Auch ist den Kultusministerien die Gedichtsinterpretation wichtiger als Grundlagenwissen, kaufmännische Allgemeinbildung und Betriebswirtschaft. Danach erzählt man der arbeitenden Bevölkerung, dass sie Verantwortung für ihren Nachwuchs zu übernehmen hätten als ob sie das nicht von selbst tun würden und stellt sie nicht nur auf sich selbst, sondern zockt sie noch ab. Die Abgaben sind da vielseitig und die Steuern absichtlich kompliziert. Könnte man das selbst, so müsste Dir vertraut werden. Belege musst Du eh bringen und Du könntest etwas werden. Will man aber nicht, denn die Kommunisten sagten auch: „Vertrauen ist gut, Kontrolle ist besser“ und nicht nur Steve Jobs war da ganz anderer Meinung. Ohne Studium wird es nichts. Dieses Ungeschriebene Gesetz beweist seine Existenz täglich neu. Ob Anteil der akademischen Gefängnisinsassen, bevorzugte Auswahl von Bewerbern zum KFZ-Mechatroniker. Das einzige, an was sich das staatliche Schulsystem noch klammert: Es will keine Kinder von Extremistischen Eltern an deren ideologischen Hausunterricht abgeben. Nur dürfte der Schaden am Lehrplan überwiegen. --Hans Haase (有问题吗) 09:56, 24. Jan. 2016 (CET)
Vor wie vielen Jahrzehnten hast du das letze Mal eine Schule betreten? --194.95.142.180 11:50, 25. Jan. 2016 (CET)
Die Symptome belegt die Studie der Bertelsmannstiftung, über die jüngst berichtet wurde und die Zahl der Gymnasiasten, die Nachhilfe nimmt ist im Verhältnis recht aufschlussreich. --Hans Haase (有问题吗) 09:21, 28. Jan. 2016 (CET)
Also ich bin erst seit einigen Jahren aus der Schule raus... und kann deinen behauptungen so garnicht beipflichten. Vermutlich war ich wohl an einer Ausnahmeschule ohne Ideologie...--HamburgerThure (Diskussion) 11:53, 31. Jan. 2016 (CET)
Ich dachte auch, dass meine Schulzeit ohne Ideologie war. Mit der Zeit erkennt man mehr. Interessant wird es wenn Journalisten aus dem Ausland berichten und dortige Zustände schildern. Und dann stellst Du fest, dass hier derselbe Unfug gemacht wird und nur einen anderen Namen hat. Es muss keine Weltreise sein, aber möglicherweise solltest Du mehrere außereuropäischen Länder kennen lernen. Urlaub reicht da nicht. --Hans Haase (有问题吗) 01:01, 3. Feb. 2016 (CET)